You are on page 1of 248

M O D U L A R S Y S T E M

Class 8
GEOMETRY

www.zambak.com
Copyright © Zambak Yayýncýlýk ve
Eðitim Gereçleri A.Þ.
All rights reserved.
No part of this book may be reproduced,
stored in a retrieval system or
transmitted in any form without the prior
written permission of the publisher.
Digital Assembly
Zambak Typesetting & Design
Page Design
Serdar YILDIRIM
Aydýn ÇETÝN
Proofreader
Zoe Barnett
Publisher
Zambak Yayýncýlýk ve
Eðitim Gereçleri A.Þ.
Printed by
Çaðlayan A.Þ.
Gaziemir / zmir, August 2012
Tel: +90-232 252 22 85
+90-232 522 20 96 / 97
ISBN: 978-605-112-467-4
Printed in Turkey
DISTRIBUTION
Zambak Yayýncýlýk ve
Eðitim Gereçleri A.Þ.
Mahmutbey Merkez Mah.
Souksu Cad. No. 31 Tek-er Merkezi
Baclar / ÝSTANBUL
_______________________
Tel.: +90-212 604 21 00
Fax: +90-212 604 21 12
http://book.zambak.com
To the Teacher,
Analytic Analysis of Lines and Circles is designed to provide students with the
analytic geometry background needed for further college-level geometry
courses. Analytic geometry can be defined as algebraic analysis applied to
geometrical concepts and figures, or the use of geometrical
concepts and figures to illustrate algebraic forms.
Analytic geometry has many applications in different
branches of science and makes it easier to solve a wide
variety of problems. The goal of this text is to help students
develop the skills necessary for solving analytic geometry
problems, and then help students apply these skills. By the
end of the book, students will have a good understanding
of the analytic approach to solving problems. In addition,
we have provided many systematic explanations throughout
the text that will help instructors to reach the goals that
they have set for their students. As always, we have taken
particular care to create a book that students can read,
understand, and enjoy, and that will help students gain
confidence in their ability to use analytic geometry.

To the Student,
This book consists of two chapters, which cover analytical analysis of lines and
circles respectively. Each chapter begins with basic definitions, theorems, and
explanations which are necessary for understanding the subsequent chapter
material. In addition, each chapter is divided into subsections so that students
can follow the material easily.

Every subsection includes self-test Check Yourself problem sections followed by basic
examples illustrating the relevant definition, theorem, rule, or property. Teachers
should encourage their students to solve Check Yourself problems themselves
because these problems are fundemental to understanding and learning the related
subjects or sections. The answers to most Check Yourself problems are given directly
after the problems, so that students have immediate feedback on their progress.
Answers to some Check Yourself problems are not included in the answer key, as they
are basic problems which are covered in detail in the preceding text or examples.
Giving answers to such problems would effectively make the problems redundant,
so we have chosen to omit them, and leave students to find the basic answers
themselves.

At the end of every section there are exercises categorized according to the
structure and subject matter of the section. Exercises are graded in order,
from easy (at the beginning) to difficult (at the end).
Exercises which involve more ability and effort are
denoted by one or two stars. In addition, exercises which
deal with more than one subject are included in a
separate bank of mixed problems at the end of the
section. This organization allows the instructor to deal
with only part of a section if necessary and to easily determine which exercises
are appropriate to assign.

Every chapter ends with three important sections.

The Chapter Summary is a list of important concepts and


formulas covered in the chapter that students can use
easily to get direct information whenever needed.

A Concept Check section contains questions about the


main concepts of the subjects
covered, especially about the definitions, theorems or
derived formulas.

Finally, a Chapter Review Test section consists of three tests, each with sixteen
carefully-selected problems. The first test covers
primitive and basic problems. The second and third tests
include more complex problems. These tests help
students assess their ability in understanding the
coverage of the chapter.

The answers to the exercises and the tests are given at the end of the book so
that students can compare their solution with the correct answer.

Each chapter also includes some subjects which are denoted as optional. These
subjects complement the topic and give some additional
information. However, completion of optional sections is
left to the discretion of the teacher, who can take into
account regional curriculum requirements.
CHAPTER 1 I. FURTHER APPLICATIONS . . . . . . . . . . . .89
1. Menelaus’ Theorem . . . . . . . . . . . . . . . . . . . . . . .89
SECTION : QUADRILATERALS
2. Auxiliary Elements of a Triangle . . . . . . . . . . . . . .93

A. QUADRILATERALS AND THEIR BASIC

PROPERTIES . . . . . . . . . . . . . . . . . . . . .10
1. Definitions . . . . . . . . . . . . . . . . . . . . . . . . . . . . . . .10
2. Basic Properties of a Quadrilateral . . . . . . . . . . .11 CHAPTER 2
3. Inscribed and Circumscribed Quadrilaterals . . . .17
SECTION : PYTHAGOREAN THEOREM
B. PARALLELOGRAM . . . . . . . . . . . . . . . . .22
1. Definition . . . . . . . . . . . . . . . . . . . . . . . . . . . . . . . .22 A. TRIGONOMETRIC RATIOS OF
2. Properties of a Parallelogram . . . . . . . . . . . . . . . .23
3. Proving that a Quadrilateral Is a Parallelogram . .33 ACUTE ANGLES . . . . . . . . . . . . . . . . .136

C. RECTANGLE . . . . . . . . . . . . . . . . . . . . .38 B. RECIPROCAL TRIGONOMETRIC


1. Definition . . . . . . . . . . . . . . . . . . . . . . . . . . . . . . . .38
2. Properties of a Rectangle . . . . . . . . . . . . . . . . . . .38 RATIOS . . . . . . . . . . . . . . . . . . . . . . . .138

D. RHOMBUS . . . . . . . . . . . . . . . . . . . . . .46 C. TRIGONOMETRIC RATIOS OF


1. Definition . . . . . . . . . . . . . . . . . . . . . . . . . . . . . . . .46
COMPLEMENTARY ANGLES . . . . . . . .139
2. Properties of a Rhombus . . . . . . . . . . . . . . . . . . .46

E. SQUARE . . . . . . . . . . . . . . . . . . . . . . . .52 D. BASIC TRIGONOMETRIC IDENTITIES . .140


1. Definition . . . . . . . . . . . . . . . . . . . . . . . . . . . . . . . .52
E. FINDING A TRIGONOMETRIC RATIO
2. Properties of a Square . . . . . . . . . . . . . . . . . . . . .52

FROM A GIVEN RATIO . . . . . . . . . . . .146


F. TRAPEZOID . . . . . . . . . . . . . . . . . . . . . .57
1. Definition . . . . . . . . . . . . . . . . . . . . . . . . . . . . . . . .57
G. RATIOS IN A 45°-45°-90° TRIANGLE . .150
2. Properties of a Trapezoid . . . . . . . . . . . . . . . . . . .57
3. Isosceles Trapezoids . . . . . . . . . . . . . . . . . . . . . .67 F. RATIOS IN A 30°-60°-90° TRIANGLE . .150
4. Right Trapezoids . . . . . . . . . . . . . . . . . . . . . . . . . .74 1. Distance Between Two Points . . . . . . . . . . . . . .154
2. Midpoint of a Line Segment . . . . . . . . . . . . . . .156
G. KITE . . . . . . . . . . . . . . . . . . . . . . . . . . .78
1. Definition . . . . . . . . . . . . . . . . . . . . . . . . . . . . . . . .78 H. EUCLIDEAN RELATIONS . . . . . . . . . . .161
2. Properties of a Kite . . . . . . . . . . . . . . . . . . . . . . . .78

H. THE TRIANGLE PROPORTIONALITY THEOREM

AND THALES’ THEOREM . . . . . . . . . . . .84


1. The Triangle Proportionality Theorem . . . . . . . . .84
2. Thales’ Theorem of Parallel Lines . . . . . . . . . . . .87
CHAPTER 3

SECTION : AREAS OF QUADRILATERALS

A. THE CONCEPT OF AREA . . . . . . . . . .172


1. Basic Definitions . . . . . . . . . . . . . . . . . . . . . . . . .172
2. Area of a Rectangle . . . . . . . . . . . . . . . . . . . . . .173

B. AREA OF A TRIANGLE . . . . . . . . . . . . .176

C. PROPERTIES OF THE AREA OF A TRIANGLE189

D. AREA OF A QUADRILATERAL . . . . . . . .213

E. AREA OF A PARALLELOGRAM . . . . . . .218

F. AREA OF A RHOMBUS . . . . . . . . . . . .225

G. AREA OF A SQUARE . . . . . . . . . . . . . .228

H. AREA OF A TRAPEZOID . . . . . . . . . . . .231

I. AREA OF A KITE . . . . . . . . . . . . . . . . .241


If you look around you, you will see many things
which have four lines for sides. A book, a door, the
spaces between the bars at a window, a slice of
bread and the floor of a square room are all examples
of a closed figure bounded by four line segments.
A figure like this is called a quadrilateral. In other
words, a quadrilateral is a geometrical figure which
has four sides. In this section we will study
quadrilaterals and their properties.

A. QUADRILATERALS AND THEIR BASIC PROPERTIES


1. Definitions

Definition quadrilateral
A quadrilateral is a polygon which has four sides.

In each of the quadrilaterals ABCD shown c


C
D
opposite, points A, B, C and D are the vertices
and the line segments AB, BC, CD and DA are b
d
the sides of the quadrilateral. ABC, BCD,
CDA and DAB are the interior angles of A B
a
the quadrilateral. {A, C} and {B, D} are two a convex quadrilateral
examples of pairs of opposite vertices. The
D
pairs of sides {AB, CD} and {BC, DA} are
opposite sides. {A, C} and {B, D} are
d c
two pairs of opposite angles.
B
Since a quadrilateral is a polygon, it also has a b
consecutive vertices, sides and angles.
A C
In both figures, AC and BD are the diagonals a concave quadrilateral
of the quadrilateral. Notice that in a concave
quadrilateral, one of the diagonals lies in its exterior region. In a convex quadrilateral, the
diagonals always lie in its interior region.
The perimeter of a quadrilateral is the sum of the lengths of all of its sides. In other words,
How many quadrilaterals
can you see?
the perimeter of a quadrilateral ABCD is AB + BC + CD + DA.

8 Geometry 8
EXAMPLE 1 State the opposite sides, opposite angles and the diagonals in each quadrilateral.

a. P b. L
c.
T Z
K
N S
K Y

M X
T

Solution a. opposite sides: {KM, PN} and {KP, MN}


opposite angles: {M, P} and {K, N}
diagonals: KN and MP

Questions b and c are left as an exercise for you.

2. Basic Properties of a Quadrilateral

a. Angles of a quadrilateral
We have already seen that the sum of the ÐD¢
D
measures of the interior angles of an n-sided
ÐD
polygon is (n – 2) · 180°. Since a quadrilateral C
ÐC
has four sides we have n = 4, and ÐC¢

(4 – 2)  180° = 2  180° = 360°. So the sum ÐA¢ ÐA ÐB


of the measures of the interior angles of a B
A ÐB¢
quadrilateral is 360°.
The sum of the measures of the exterior
angles of a quadrilateral is also 360° (since this is true for all polygons).
In conclusion, for any quadrilateral ABCD we have m(A) + m(B) + m(C) + m(D) = 360°
and m(A) + m(B) + m(C) + m(D) = 360°.

Quadrilaterals 9
EXAMPLE 2 In the figure,
F
m(EAD) = 50°, 75° C

m(EBF) = 60° and D


?
m(DCF) = 75°.
Find the measure of ADC.
50° 60°
E A B

Solution m(DAB) + m(DAE) = 180° (supplementary angles)


m(DAB) = 180° – m(DAE) = 180° – 50° = 130°
m(BCD) + m(DCF) = 180° (supplementary angles)
m(BCD) = 180° – m(DCF) = 180° – 75° = 105°
In quadrilateral ABCD,
m(ADC) + m(DAB) + m(ABC) + m(BCD) = 360° (sum of the interior angles)
m(ADC) + 130° + 60° + 105° = 360°
m(ADC) = 360° – 295°
= 65°.

EXAMPLE 3 In the figure,


F
m(BAE) = x,
D y
m(ABF) = m, C
n
m(FCD) = y,
m(CDE) = n and
m
A
x + y = 105°. x
B

Calculate m + n. E

Solution m(DAB) + x = 180°; m(DAB) = 180° – x (supplementary angles)


m(BCD) + y = 180°; m(BCD) = 180° – y (supplementary angles)
In quadrilateral ABCD,
m(CDA) + m(DAB) +m(ABC) + m(BCD) = 360° (sum of interior angles)
n + (180° – x) + m + (180° – y) = 360° (substitution)
m + n = 360° – 360° + x + y; m + n = x + y = 105°. (x + y = 105° is given)

10 Geometry 8
Property
In a quadrilateral, the measure of the angle
D
formed by the bisectors of two consecutive
interior angles equals the half the sum of the
K C
measures of the other two angles.

In the figure, ABCD is a quadrilateral and AK


and BK are the bisectors of A and B
A B
respectively. So by Property 2,
m( C )+ m( D)
m( AKB) = .
2

EXAMPLE 4 In the figure, DF and CE are bisectors of D C


and C respectively. Given that
D
m(A) = 85° and m(B) = 75°, find the
measure of CEF.
?
E
85° F 75°
A B
Solution 1 D and C are consecutive interior angles.
By Property 2,
m( A )+ m( B) 85° +75°
m( DEC ) = = = 80°.
2 2
m(CEF) + m(DEC) = 180° (supplementary angles)
m(CEF) = 180° – m(DEC)
= 180° – 80°
= 100°
Solution 2 In quadrilateral ABCD,
m(A) + m(B) + m(C) + m(D) = 360° (sum of interior angles)
m(C) + m(D) = 360° – 85° – 75° = 200°.
In CDE,
m( CDA ) m( DCB)
m( CDE) = and m( DCE) = (DE and CE are bisectors)
2 2
m(CEF) = m(CDE) + m(DCE) (exterior angle property of a triangle)
m( CDA ) m( DCB)
= +
2 2
m( DCB)+ m( CDA ) 200°
= = =100°.
2 2

Quadrilaterals 11
Property
In a quadrilateral, the measure of the acute
C
angle formed by the bisectors of opposite
angles is half the absolute value of the
D
difference between the measures of the other
K
two angles. For example, in the figure,
| m( D ) – m( B)| P
m( AKP ) = .
2 A B

EXAMPLE 5 In the figure, DK and BE are bisectors of D, C


B, respectively. Given that D

m(C) > m(A), m(C) = 90° and


m(DKE) = 5°, find m(A).

E
Solution 1 By Property 3, K
?
| m( C ) – m( A)|
m( DKE) = A B
2
90 – m( A)
5 = . (m(C) > m(A))
2
So m(A) = 80°.
Solution 2 m(DKE) + m(DKB) = 180° (supplementary angles)
m(DKB) = 180° – m(DKE)
= 180° – 5° = 175°
In quadrilateral BCDK,
m(C) + m(CDK) + m(CBK) + m(BKD) = 360° (sum of interior angles)
m(CDK) + m(CBK) = 360° – 90° – 175° = 95°
m( D ) m( B)
= +
2 2
m( D )+ m( B)
=
2
m( D )+ m( B)
 95°
2
m( D )+ m( B) =190°.
In quadrilateral ABCD,
m(A) + m(B) + m(C) + m(D) = 360° (sum of interior angles)
m(A) = 360° – 90° – 190° = 80°.

12 Geometry 8
EXAMPLE 6 In the figure, ABCD is a concave quadrilateral A
with m(BAD) = a,
a
m(ABC) = b,
C
m(BCD) = c and b
c
m(CDA) = d. B d

Show that c = a + b + d. D

Solution Let us extend the line segment BC so that it A


intersects side AD at point K. a
BKD is an exterior angle of ABK. K
C
So m(BKD) = a + b. b a+b
c
B d
Also, BCD is an exterior angle of CKD.
So m(BCD) = m(BKD) + d = a + b + d. D

So c = a + b + d.

EXAMPLE 7 In the figure, AK and CK bisect EAB and D


BCF respectively.
50°
If m(B) = 160° and m(D) = 50°,
C
find m(AKC). A 160°

E B F

Solution In quadrilateral ABCD,


m(A) + m(B) + m(C) + m(D) = 360° (sum of interior angles)
m(A) + m(C) = 360° – 50° – 160° = 150°

m(BAE) + m(BAD) = 180° (supplementary angles)


m(BAE) = 180° – m(BAD)

m(BCF) + m(BCD) = 180° (supplementary angles)


m(BCF) = 180° – m(BCD).

Quadrilaterals 13
b. Sides of a quadrilateral
Property
If the diagonals of a quadrilateral are
D
perpendicular to each other, the sums of the
squares of the lengths of opposite sides of the d c
quadrilateral are equal.

In the figure, ABCD is a quadrilateral and A C


E
the diagonals AC and BD are perpendicular
a b
to each other. So by Property 4,
B
AB2 + DC2 = AD2 + BC2.

EXAMPLE 8 The figure shows a quadrilateral ABCD D


whose diagonals are perpendicular. 5 6
E
AB = 8 cm, A C

AD = 5 cm and
8
DC = 6 cm are given.
Find the length of side BC.
B

Solution By Property 4 we can write AB2 + DC2 = AD2 + BC2.


Substituting the given values into this equation gives
82 + 62 = 52 + BC2
BC 2 = 64 + 36 – 25
BC 2 = 75
BC = 5ñ3 cm.

EXAMPLE 9 In a quadrilateral ABCD, m(A) = m(C) = 90°. Show that AB2 + AD2 = BC2 + CD2.

D
Solution Look at the figure. Drawing the diagonal BD C
creates two right triangles DAB and DCB.
By the Pythagorean Theorem,
A
AB2 + AD2 = BD2 and BC2 + CD2 = BD2.
B
So AB2 + AD2 = BC2 + CD2, as required.

14 Geometry 8
EXAMPLE 10 The diagonals of a quadrilateral ABCD are perpendicular, with AB = 4 cm, AD = 6 cm and
DC = 10 cm. Point E is the intersection point of the diagonals, and point K is on side BC such
that that BK = KC. What is the length of EK?

Solution We begin by drawing the figure. D

Since the diagonals are perpendicular, 6 10


2 2 2 2 E
AB + DC = AD + BC (by Property 4) A C

42 + 102 = 62 + BC2 4 K
2
BC = 116 – 36 B
BC2 = ò80
In a right triangle, the BC = 4ñ5 cm.
length of the median
drawn to the hypotenuse EK is a median of the right triangle BEC, and since the length of the median drawn to the
is half the length of
hypotenuse. BC 4 5
hypotenuse is half the length of the hypotenuse, EK = = = 2 5 cm.
2 2

Check Yourself C
5
1. In the figure, m(C) = m(A) = 90°. Given AD = 3 cm,
D 12
DC = 5 cm and CB = 12 cm, find the length of AB.
3

A ? B
2. The diagonals of a quadrilateral are perpendicular to each other. The lengths of two
opposite sides are 8 cm and 4 cm, and the ratio of the lengths of the other two opposite
sides is 1:2. Find the lengths of the unknown sides.
Answers
1. 4ò10 cm 2. 4 cm and 8 cm

3. Inscribed and Circumscribed Quadrilaterals


Definition inscribed quadrilateral, cyclic quadrilateral
A quadrilateral is called an inscribed quadrilateral (or cyclic quadrilateral) if all of its vertices
lie on the same circle. This circle is called the circumscribed circle (or circumcircle) of the
quadrilateral. D

C
In the figure, ABCD is an inscribed quadrilateral.

Quadrilaterals 15
Property
The sum of the measures of either pair of D
opposite angles of an inscribed quadrilateral C
is 180°.
In the figure, ABCD is an inscribed
quadrilateral. So by Property 5, A

m(A) + m(C) = 180° and


m(B) + m(D) = 180º.
B
In fact, if the sum of any pair of opposite angles
in a quadrilateral is equal to 180° then the quadrilateral is always an inscribed quadrilateral.

EXAMPLE 11 In the figure, ABCD is an inscribed B


quadrilateral with A
5x+10°
m(A) = 4x – 5°, 4x–5°

m(B) = 5x + 10° and C ?

m(D) = 3x + 10°. 3x+10°


Find m(C).
D

Solution Since ABCD is an inscribed quadrilateral, by Property 5 its opposite angles are supplementary. So
m(B) + m(D) = 180°
5x + 10°+ 3x + 10° = 180°
8x = 160°
x = 20°, and
m(A) + m(C) = 180°
(4  20°) – 5° + m(C) = 180°
75° + m(C) = 180°
m(C) = 105°.

Definition circumscribed quadrilateral


A quadrilateral is called a circumscribed quadrilateral if all of its sides are tangent to the
same circle. This circle is called the inscribed circle of the quadrilateral.

A T D
In the figure, ABCD is a circumscribed
S
quadrilateral.
P

B E C

16 Geometry 8
Property
The sums of the lengths of the opposite sides
C
of a circumscribed quadrilateral are equal.
N
D
In the figure, ABCD is a circumscribed
M
quadrilateral. So by Property 6, K

AB + CD = BC + AD.
In fact, if the sums of the lengths of the
opposite sides of a quadrilateral are equal then A L B
the quadrilateral is always a circumscribed
quadrilateral.

EXAMPLE 12 Three consecutive sides of a circumscribed quadrilateral measure 9 cm, 12 cm and 13 cm


respectively. Find the length of the fourth side.

Solution Look at the figure. Let AB = 9 cm, BC = 12 cm B

and CD = 13 cm. 9 12
A
Since ABCD is a circumscribed quadrilateral,
C
by Property 6 we have ?
13
AB + CD = BC + AD D
9 + 13 = 12 + AD
AD = 10 cm.

Check Yourself C

1. In the figure, m(A) = 2y – 40°, m(B) = 3x – 25°, 3y


D B
2x+5° 3x–25°
m(D) = 2x + 5° and m(C) = 3y.
Find the values of x and y. 2y–40°

2. Three consecutive sides of a circumscribed quadrilateral measure 6 cm, 8 cm and 9 cm


respectively. Find the length of the fourth side.
Answers
1. x = 40°, y = 44° 2. 7 cm

Quadrilaterals 17
EXERCISES 1 .1
A. Quadrilaterals and Their Basic 4. Each figure shows the bisectors of opposite angles
Properties of a quatrilateral. Find the measure of angle x in
each case.
1. State the pairs of opposite and N M
consecutive sides and angles a. D b. D C
C
and the diagonals in the 110° x
polygon opposite. K
165°
K
x 130°
T A B A B
S
c. D

70°
K
A
160° C
2. Find the measure of angle x in each figure. B x

a. C b. C
60° 40° x
D D
x 60° 70°
70° 105°
A B
A B 5. In the figure, C
P
D
c. D 130°
m(C) = 90°, AP and BP ?
x C
A
are respectively bisectors 30°
100°
70° of DAB and CBE, A B E
B
and m(P) = 30°. Find
m(ADC).

3. Find the measure of angle x in each figure.


a. C b. C 6. Find the length x in each figure.
D
115° D x a. 5 x b.
K x 8
75° K 2x
50°
A B A B 6 4 x
c. D
4
80° c.
9
A x 7
75° C

x
B 4

18 Geometry 8
7. Find the value of x in each figure. 12. In the figure, AC and BD are D
 C
a. b. diagonals of the quadrilateral P F
5 x ABCD. Points P and K are A
10 E K
8
respectively the midpoints of
B
4 sides AD and BC, and points F
6 x
2x and E are respectively the midpoints of diagonals
AC and BD. Show that P(EKFP) = AB + DC.

D
13. In the figure, AF, BF, CE

and DE are respectively the
8. In the figure, AC  BD and D M F
6 bisectors of A, B, C C
AD  AB, m(ADB) = 60°, 60° A
E K
A C and D. Prove that the
AB = 4ñ3 cm and DC = 6 cm. E
quadrilateral EKFM is an B
Find the length of BC.
inscribed quadrilateral.
4ñ3 ?

B 14. In the figure, ABCD is a C


 D
convex quadrilateral and
O
points E and F are respectively E F
the midpoints of diagonals AC
9. In the figure, m(A) = 90°, D and BD. A B
 Prove that
m(B) = 150°, m(C) = 60°, ?
AD = 8 cm and BC = 5 cm. 8 AB2 + BC2 + CD2 + DA2 = AC2 + BD2 + (4  EF2).
Find the length of DC. 60° C
150° (Hint: Use the property of the length of a median
5
A B in BDA, BCD and AFC.)

15. In the quadrilateral ABCD D


opposite,
10. ABCD is an inscribed quadrilateral with m(A) = 70° AE = EC = 10 cm, 20 20
and m(B) = 100°. Find m(C) and m(D).
BF = FD = 10 7 cm, F
2 10 10 C
A
BC = 10 cm, E

DC = AD = 20 cm and 10ñ3 10

AB = 10ñ3 cm. B
11. A quadrilateral ABCD is a circumscribed
Find the length of EF.
quadrilateral with AB = 9 cm, BC = 7 cm and
CD = 10 cm. Find the length of side AD. (Hint: Use the formula given in question 14.)

Quadrilaterals
19
There are many different types of quadrilateral, but they all have several things in common:
all of them have four sides and two diagonals, and the sum of the measures of their interior
angles is 360°. This how they are alike, but what makes them different?
The figure shows some special types of quadrilateral and the relationships between them. In
this section we will look at the properties of each of these special quadrilaterals in turn.

no pairs of
parallel sides
Quadrilateral

two pairs of Kite


parallel sides one pair of
Parallelogram parallel sides

Trapezoid
Rectangle Rhombus

Isosceles Right
Square Trapezoid
Trapezoid

B. PARALLELOGRAM

1. Definition

Definition parallelogram
A parallelogram is a quadrilateral which has two pairs of opposite parallel sides.

D C
In the figure, AB  DC and BC  AD.
So quadrilateral ABCD is a parallelogram by
definition.

A B

20 Geometry 8
2. Properties of a Parallelogram
Theorem 5

Opposite sides of a parallelogram are congruent.

Proof Look at the figure. Given that ABCD is a a


D C
parallelogram, we need to show AB  CD and 2
1
BC  DA. b b
Let us use a flow chart proof. 3
4
A a B

AB  CD 2  4

By definition Alternate angles


ASA means the Angle AB  CD
Side Angle postulate: If ABCD is a parallelogram. BC  AD 1  3 ABD CDB
BC  DA
two angles and their
common side in a triangle Given By definition Alternate angles ASA
are congruent to two
angles and their common
BD  BD
side in another triangle,
then the triangles are Common side
congruent.
So opposite sides of a parallelogram are congruent, as required.
Notice that as a result of Theorem 5, the perimeter of a parallelogram is twice the sum of any
two consecutive sides:
P(ABCD) = 2  (AB + BC).

EXAMPLE 13 In the figure, ABCD is a parallelogram with A 2x + 1 D


AB = y + 1,
BC = 5x – 8,
y+1 3y – 7
CD = 3y – 7 and AD = 2x + 1.
Find the lengths of the sides of the parallelogram.
B 5x – 8 C

Solution Since the lengths of opposite sides of a parallelogram are equal, AB = CD and BC = AD.
So y + 1 = 3y – 7, and 2x + 1 = 5x – 8
2y = 8 3x = 9
y = 4 cm x = 3 cm.
So AB = CD = 5 cm and BC = AD = 7 cm.

Quadrilaterals 21
EXAMPLE 14 In the figure, ABCD is a parallelogram with
D C
m(A) = 2m + 10° and m(C) = 3m – 15°.
3m – 15°
Find the measures of the interior angles of
the parallelogram.
2m + 10°
A B

Solution The measures of opposite angles in a parallelogram are equal:


m(A) = m(C)
2m + 10° = 3m – 15°
m = 25°.
So m(A) = m(C) = 60°.
Since consecutive angles in a parallelogram are supplementary, we have
m(A) + m(D) = 180°
60° + m(D) = 180°
m(D) = 120°

m(B) = m(D) = 120°. (opposite angles)

EXAMPLE 15 Show that the measure of the angle formed by the bisectors of any two consecutive angles in
a parallelogram is 90°.

D C
Solution In the figure, point E is the intersection point
of the bisectors of A and B. We need to E

show m(E) = 90°. We know


m(A) + m(B) = 180°, (supplementary angles)
A B
m( A )
m( EAB) = and,
2
m( B)
m( EBA ) = . Adding these last two equations side by side gives us
2
m( A ) m( B) m( A )+ m( B) 180°
m( EAB)+ m( EBA ) = + = = = 90°.
2 2 2 2
In AEB,
m(EAB) + m(EBA) + m(E) = 180° (sum of interior angles)
90° + m( ) = 180°
m(E) = 90°.

22 Geometry 8
EXAMPLE 16 ABCD is a parallelogram with AB > AD. Point E is on the side DC such that BE = BC, AE = AB
and m(DAE) = 15°. Find the measures of the interior angles of the parallelogram.
D E C
x x
Solution We begin by drawing the figure. Let m(C) = x. x

Then 15°
m(A) = m(C) = x (opposite angles) x – 15° x
m(EAB) = m(C) – m(DAE) = x – 15° A B

m(BCE) = m(CEB) = x (base angles in isosceles triangle BEC)


m(ABE) = m(CEB) = x (alternate interior angles, DC  AB)
m(BEA) = m(ABE) = x. (base angles in isosceles triangle ABE)
In ABE,
m(EAB) + m(ABE) + m(BEA) = 180° (sum of interior angles)
x – 15° + x + x = 180°
3x = 195°
x = 65°.
So m(A) = m(C) = 65°.
Since consecutive angles in a parallelogram are supplementary, we have
m(A) + m(B) = 180°
65° + m(B) = 180°
m(B) = 115°.
Since B and D are opposite angles, m(D) = m(B) = 115°.

Check Yourself
1. In the figure, ABCD is a parallelogram with AD = 2y – 1, D x+5 C
AB = 2x – 3, BC = y + 1 and CD = x + 5. Find the
perimeter of the parallelogram. 2y–1 y+1

A 2x–3 B

2. The measure of the angle between one side of a parallelogram and the altitude drawn
from one of its obtuse angles is 35°. Find the measures of the interior angles of the
parallelogram.
Answers
1. 32 2. 55° and 125°

Quadrilaterals 23
Theorem 8

The diagonals of a parallelogram bisect each other.


Proof D a C
Look at the figure.
Given that ABCD is a parallelogram, we need O
to show AO  OC and BO  OD. b b

Let us prove it with a two-column proof.


A a B

Statements Reasons
1. ABCD is a parallelogram. Given
2. AB  DC Definition of a parallelogram
3. OBA  ODC and OAB OCD Alternate interior angles
4. AB  DC Opposite sides are congruent.
5. OBA  ODC ASA by 3 and 4
6. AO  OC and BO OD Corresponding sides of congruent triangles

EXAMPLE 17 In the figure, ABCD is a parallelogram and


D C
point O is the intersection of diagonals AC and 2x
1
+
3y–
DB. AO = 3x + 5, OC = 3y – 1, BO = y + 2 1

and OD = 2x + 1 are given. +5 O

y+
3x

2
Find the lengths of the diagonals AC and BD.
A B

Solution The diagonals of a parallelogram bisect each other, so


AO = OC and BO = OD.
This gives the system
3x+5 = 3y – 1 3x – 3y = –6
 ; 
 y + 2 = 2 x +1 y = 2 x – 1.
Substitute y = 2x – 1 in the first equation:
3x – 3(2x – 1) = –6
3x – 6x + 3 = –6
–3x = –9
x = 3.
For x = 3, y = 2x – 1
y=23–1
y = 5.
So x = 3 and y = 5, and
AC = 2  AO = 2(3x + 5) = 2(3  3 + 5) = 28,
BD = 2  BO = 2(y + 2) = 2(5 + 2) = 14.
24 Geometry 8
EXAMPLE 18 In the figure, ABCD is a parallelogram and AE D E 2 C
is the bisector of A. Given that BC = 5 cm
and EC = 2 cm, find the perimeter of ABCD.
5

A B

D 5 E 2 C
Solution The lengths of opposite sides of a parallelogram
are equal, so BC = AD = 5 cm.
5 5
Opposite sides of a parallelogram are
parallel, so DC  AB. Now we can write
A 7 B

m(EAB) = m(AED) (alternate interior angles)


ADE is isosceles (two congruent angles in ADE)
AD = DE = 5 cm (legs of isosceles ADE)
DC = DE + EC
DC = 5 + 2
DC = 7 cm
AB = DC = 7 cm. (opposite sides)
P(ABCD) = 2  (7 + 5) = 24 cm.

EXAMPLE 19 In a parallelogram ABCD, DC = 12 cm and point O is the intersection point of the diagonals
AC and DB. The perimeter of COB is 24 cm and the perimeter of AOB is 28 cm. Find the
perimeter of ABCD.

Solution AB = DC = 12 cm because opposite sides of a parallelogram are congruent.


The diagonals of a parallelogram bisect each other, so let AO = OC = x and DO = OB = y. Then
P(AOB) = AO + OB + AB
28 = x + y + 12 D C

x + y = 16 cm, and y
x
P(COB) = CO + OB + BC O y
24 = x + y + BC x

24 = 16 + BC A B
BC = 8 cm.
So the perimeter of ABCD is 2(AB + BC) = 2(12 + 8) = 40 cm.

Quadrilaterals 25
EXAMPLE 20 In the figure, ABCD is a parallelogram. Point
D C
E is the midpoint of side AB and point F is the
intersection of line segments EC and DB. 6
Given FB = 2 cm and FC = 6 cm, find the
lengths of DF and EF. F 2

A E B

Solution AE = EB since point E is the midpoint of AB. D 2x


C
Let us write AE = EB = x, so AB = 2x.
6
The lengths of opposite sides of a parallelogram
are equal, so CD = AB = 2x. F 2

Opposite sides of a parallelogram are parallel, A E B


x x
so DC  AB. Also,
m(DFC) = m(BFE) (vertical angles)
m(FBE) = m(FDC). (alternate interior angles)
Angle Angle (AA) So FEB  FCD by the Angle Angle similarity postulate.
similarity postulate:
If two angles of one If the triangles are similar, then the lengths of their corresponding sides are proportional, so
triangle are congruent
to two angles of another FE EB FB FE x 2
triangle, then the
= = , = = .
FC CD FD 6 2x FD
triangles are similar.
FE x
Since  , by simplification and cross multiplication we get EF = FE = 3 cm.
6 2x
x 2
Similarly,  gives us DF = FD = 4 cm.
2 x FD

EXAMPLE 21 In the figure, ABCD is a parallelogram. Point E


D E
C
is the midpoint of side DC and point K is on
2 F
side BC such that KC = 3  KB. Point F is the
3x
intersection of line segments AE and DK.
6
DF = 2 cm and AF = 6 cm are given. Find K
x
the lengths of FK and EF. A B

26 Geometry 8
Solution Let KB = x. So KC = 3x. D E
C
The lengths of opposite sides of a parallelogram 2 F 3x
2
are equal, so CB = AD = 4x. 4x P 3x
6
Let P be a point on the line segment DK and K
x
let us draw the line segment EP such that A B
EP  AD.

So EP  BC, because AD  BC.


Triangle proportionality
theorem: A line parallel
Since point E is the midpoint of DC and EP  BC, by the triangle proportionality theorem we
to one side of a triangle can say that point P is the midpoint of DK and so EP is a midsegment of CDK.
which intersects the
other two sides divides the KC 3x
So EP = = . Also,
two sides proportionally. 2 2
A
m(FAD) = m(FEP) (alternate interior angles)

m(EFP) = m(AFD). (vertical angles)


D E
So FEP  FAD by the Angle Angle similarity postulate.

B C If the triangles are similar then the lengths of their corresponding sides are proportional, i.e.
AD AE 3x
DE  BC  = FE EP FP FE FP
DB EC
= = ; = 2 = .
FA AD FD 6 4x 2

3x
FE
So  2 , which gives us FE = 9 cm .
6 4x 4
3x
2  FP 3
Similarly, which gives us FP  cm. Now
4x 2 4
DP = DF + FP

4 1
DP = 2+ = 3 cm
3 3
1
PK = DP = 3 cm, and so finally (point P is the midpoint of DK)
3

FK = PK + FP

1 4 2
=3  4 cm.
3 3 3

Quadrilaterals 27
EXAMPLE 22 In the figure, ABCD is a parallelogram. Points D C
E and F are the midpoints of sides AD and AB
respectively, and point K is the intersection of E
EF and AC. Point N is the intersection of EB N

and AC. If KN = 1 cm, find the length of AC. K


A F B

D C
Solution Let point O be the intersection of the
diagonals AC and DB. Since the diagonals of E O
N
a parallelogram bisect each other, AO = OC
1
and DO = OB. Also, EF  BD since EF is a K
midsegment of ADB. A F B

OD BO
So EK is the midsegment of ADO, which
means EK  , i.e. EK (since OD = BO).
2 2
Now

m(NEK) = m(NBO) (alternate interior angles)


m(ENK) = m(BNO). (vertical angles)
So ENK  BNO by the Angle Angle similarity postulate.
If two triangles are similar then their corresponding sides are proportional:
OB
EN EK KN NK 1
= = , = 2 = .
BN BO NO BN BO NO
OB
1
So 2  , which gives us NO = 2 cm. Also,
BO NO
KO = KN + NO = 1 + 2 = 3 cm,
AO = AK + KO = 2KO = 6 cm. (point K is the midpoint of AO)
So AC = 2AO = 12 cm.

EXAMPLE 23 ABCD is a parallelogram and BH and BE are altitudes from vertex B to the sides DC and AD
respectively. The measure of the angle between BH and BE is 60°, DE = 2 cm and DH = 6 cm.
Find the lengths of sides AB and AD.

28 Geometry 8
Solution We begin by drawing a figure with the D 6 H x C
information in the question: 2 60°
E
m(EBH) = 60° and 30° 2x
60°
BE  AD and BH  DC (since AB  DC and 60° 30°
BH is an altitude). From the figure, A B

m(ABE) = m(ABH) – m(EBH)


= 90° – 60°
= 30°.
In the right triangle ABE,
m(A) + m(ABE) + m(BEA) = 180° (sum of interior angles)
m(A) + 30° + 90° = 180°
m(A) = 60°
m(A) = m(C) = 60°. (opposite angles of a parallelogram)
In the right triangle CHB,
m(C) + m(CHB) + m(HBC) = 180° (sum of interior angles)
60° + 90° + m(HBC) = 180°
m(HBC) = 30°.
Now let HC = x. Then
BC = 2x (side opposite 30° is half of the hypotenuse)
AB = DC = 6 + x (opposite sides of a parallelogram)
Remember:
AB 6+ x
In a 30° - 60° - 90° AE = = (side opposite 30° is half of the hypotenuse)
triangle, the length of 2 2
the side opposite 30° is AD = BC (opposite sides of a parallelogram)
half the length of the
hypotenuse. 6+ x
2+ = 2x
2
4+6+ x = 4 x

3x =10
10
x= cm.
3
28 20
So AB = 6+ x = cm and BC = 2 x = cm.
3 3

Quadrilaterals 29
Theorem 9

In a parallelogram, the sum of the squares of the lengths of the diagonals is equal to twice
the sum of the squares of the lengths of two consecutive sides.

Proof Let a, b and e, f be the lengths of the sides and D a C


diagonals of a parallelogram, respectively. f
e
Then we need to prove that 2 2
b O b
e2 + f 2 = 2(a2 + b2). f e
2 2
Remember the theorem which relates the
C median of a triangle and its sides: if a, b and A a B
a
b K c are sides of a triangle and Va is the median
Va
a2
A c B to side a, then 2Va2 + = b2 + c2 .
2
In ABC,
In the figure above, ABCD is a parallelogram.
a2
2Va 2 + = b2 + c2 .
2 Let us apply the median theorem to DAC:
AC 2
2  DO2 + = AD 2 + DC 2
2
2 2
e f
2  + = b 2 + a2
2
  2
e2 f 2
2 + = b 2 + a2
4 2
e2 + f 2 = 2( b2 + a2 ). This is the required result.

EXAMPLE 24 The diagonals of a parallelogram measure 8 cm and 4ñ6 cm, and the shorter side of the
parallelogram measures half the length of its longer side. Find the perimeter of this parallelogram.

Solution Let x be the length of the shorter side and y be the length of the longer side of the
parallelogram. Then from the question, y = 2x.
Let e and f be the lengths of the diagonals. Then
e2 + f 2 = 2(a2 + b2) (by Theorem 9)
82 + (4ñ6)2 = 2  (x2 + (2x)2)
64 + 16  6 = 10x2
10x2 = 160
x2 = 16
x = 4 cm.
So the sides of the parallelogram measure 4 cm and 8 cm, and the perimeter of the
parallelogram is 2  (4 + 8) = 24 cm.

30 Geometry 8
Check Yourself
1. In the figure, ABCD is a parallelogram. Point O is the intersection D 9 C

point of diagonals AC and DB, and DC = 9 cm and BC = 6 cm.


O
Given that P(AOD) = 17 cm, find P(AOB). 6

A B

2. In the figure, ABCD is a parallelogram. D 6 C


120°
Find the length of the altitude BH if DC = 6 cm. H

A B
3. In the figure, ABC is a triangle and quadrilaterals AFHE and A
DBHE are parallelograms. If DG = 2GE and AB = 12 cm,
F
find the length of EH.

G
D E
Answers
1. 20 cm 2. 3ñ3 cm 3. 3 cm B H C

3. Proving that a Quadrilateral Is a Parallelogram


As we have seen, if both pairs of opposite sides of a quadrilateral are parallel then by definition
the quadrilateral is a parallelogram. Here are some more theorems which help us to prove
that a quadrilateral is a parallelogram:

Theorem 10

If the diagonals of a quadrilateral bisect each other then the quadrilateral is a parallelogram.

Theorem 11

If both pairs of opposite sides of a quadrilateral are congruent then the quadrilateral is a
parallelogram.

Theorem 12

If any two opposite sides of a quadrilateral are parallel and congruent then the
quadrilateral is a parallelogram.

Theorem 13

If both pairs of opposite angles of a quadrilateral are congruent then the quadrilateral is a
parallelogram.

We now have five ways to prove that a quadrilateral is a parallelogram: we can use the
definition or one of the four theorems above.

Quadrilaterals 31
EXAMPLE 25 Write a two-column proof of Theorem 10: if the diagonals of a quadrilateral bisect each other
then the quadrilateral is a parallelogram.

Solution Look at the figure. Given that the diagonals D C


of quadrilateral ABCD bisect each other, we
need to prove that ABCD is a parallelogram. In
O
other words, we need to show that both pairs
of opposite sides of quadrilateral ABCD are
A B
parallel.

Statements Reasons
1. AO = OC and BO = OD Given
2. BOC  DOA Vertical angles
Side Angle Side (SAS) 3. BOC  DOA SAS postulate by 1 and 2
postulate: If two sides
and their interior angle 4. DOC  BOA Vertical angles
in a triangle are congruent
5. OAD  OCB and OBC  ODA Corresponding angles of congruent triangles
to two sides and their
interior angle in another 6. AD  BC By 5
triangle, then the
triangles are congruent. 7. BOA  DOC SAS postulate by 1 and 4
8. OAB  OCD and OBA  ODC Corresponding angles of congruent triangles
9. AB  DC By 8
10. ABCD is a parallelogram. By 6 and 9

EXAMPLE 26 In a parallelogram ABCD, point O is the intersection of diagonals AC and BD, and points M and
N are midpoints of DO and BO respectively. Show that the quadrilateral ANCM is a parallelogram.

Solution Look at the figure. ABCD is a parallelogram. D C


The diagonals of a parallelogram bisect each M
other, so BO = OD and AO = OC. O

DO
M is the midpoint of DO, so DM = MO = . N
2
A B
BO
N is the midpoint of BO, so BN = NO = .
2
Since DO = BO we have MO = NO.
MN and AC are diagonals of the quadrilateral ANCM and they bisect each other. So by
Theorem 10, ANCM is a parallelogram.

EXAMPLE 27 Show that the quadrilateral which is formed by joining the midpoints of the sides of any
quadrilateral is a parallelogram.

32 Geometry 8
Solution Look at the figure. ABCD is a quadrilateral, C
S
and points K, P, T and S are midpoints of the D
sides DA, AB, BC and CD respectively. We
T
have to show that KPTS is a parallelogram. In K
other words, we have to prove that both pairs
of opposite sides of the quadrilateral KPTS A P B
are parallel.
In BDA, KP  BD. (KP is a midsegment)
In BCD, ST  BD. (ST is a midsegment)
So KP  ST. (lines parallel to the same line are parallel)
In DAC, KS  AC. (KS is a midsegment)
In ABC, PT  AC. (PT is a midsegment)
So KS  PT. (lines parallel to the same line are parallel)
So KPTS is a parallelogram, since both pairs of its opposite sides are parallel.

Theorem 14

Let ABCD be a parallelogram, and let d be a D C


line in the same plane. Then the following
statements are true:
d
B
a. If line d is perpendicular to each of AA1, A C1
BB1, CC1 and DD1 as in the figure, then B1
D1
AA1+ CC1 = DD1 + BB1.
A1

D C
b. If line d cuts the parallelogram ABCD such d

that line d is perpendicular to each of AA1, C1


B1
BB1, CC1 and DD1 as in the figure, then A B
D1
AA1 + CC1 = DD1 – BB1.
A1

c. If point O is the intersection point of the D C


diagonals of the parallelogram ABCD and
O
line d does not cut the parallelogram, and
d
if line d is perpendicular to each of AA1, B
A C1
BB1, CC1, DD1 and OO1 as in the figure,
O B1
then D1 1
AA1 + BB1 + CC1 + DD1 = 4  OO1. A1

Quadrilaterals 33
EXAMPLE 28 The figure shows a parallelogram ABCD.
K
d
Line d does not intersect ABCD and d is
P x+4
perpendicular to each of AS, BP, CK and DM. M
Given AS = 4x – 1, BP = 3x + 3, CK = x + 4 2
D
S C
3x+3
and MD = 2 cm, find the value of x.
4x–1

Solution By part a of Theorem 14 we can write A B


AS + CK = BP + MD.
Substituting the given values into this equation gives us
4x – 1 + x + 4 = 3x + 3 + 2
2x = 2
x = 1 cm.
Theorem 15

In the figure, ABCD is a parallelogram. S


If points A, K, T, S and B, C, S are
respectively collinear and if BD is a diagonal
D T
of the parallelogram, then C
K
AK2 = KT  KS.

A B

Proof We will use a two-column proof.

Statements Reasons
1. KAB   KTD Alternate interior angles
2. BKA  DKT Vertical angles
3. KAB  KTD AA similarity postulate
KA KB
4. = Corresponding sides of similar triangles are proportional.
KT KD
5. DAK  BSK Alternate interior angles
6. AKD  SKB Vertical angles
7. KBS  KDA AA similarity postulate
KS KB
8. = Corresponding sides of similar triangles are proportional.
KA KD
KA KS
9. = By 4 and 8
KT KA
10. KA2 = KS  KT By 9

34 Geometry 8
EXAMPLE 29 In the figure, ABCD is a parallelogram, AC is
N
5
its diagonal and points B, T, K, N and A, D, N
D K C
are respectively collinear. If BT = 6 cm and
T
KN = 5 cm, find the length of TK.
6

A B
Solution By Theorem 15, BT 2 = TK  TN.
Substituting the given values gives us the equality
62 = TK  (TK + 5).
Let TK = x.
Then 62 = x  (x + 5)
36 = x2 + 5x
x2 + 5x – 36 = 0
(x – 4)(x + 9) = 0 (by factoring)
x = 4 or x = –9.
Since x = –9 is not a posssible length, TK = 4 cm.

Note
Since opposite angles of a parallelogram do not need to be D C
supplementary and the sums of the lengths of opposite sides are
not necessarily equal, a parallelogram cannot usually be inscribed A B
or circumscribed.
ABCD cannot be inscribed
or circumscribed

Check Yourself
1. In the figure opposite, line d does not intersect parallelogram A1 D
1
ABCD and d is perpendicular to each of AA1, BB1, CC1 and B1 C
1 d
5
DD1. If AA1 = 13 cm, DD1 = 5 cm and BB1 = 11 cm, find the 13 D
C
length of CC1. 11

A B

2. In the figure opposite, ABCD is a parallelogram. Points D, E, F D C


and G are collinear, and point E is the intersection of DG and E
the diagonal AC. If FG = 5 cm and EF = 4 cm, find the length 4 F
5
of DE.
A B G
Answers
1. 3 cm 2. 6 cm

Quadrilaterals 35
C. RECTANGLE
1. Definition
Definition rectangle
A rectangle is a parallelogram which has four right angles.
We can also define a rectangle as a D a C
parallelogram with one right angle, since if
one of the angles of a parallelogram is a
b b
right angle then the other three angles will
also be right angles.
A a B
In the figure, ABCD is a
parallelogram with right angles
m(A) = m(B) = m(C) = m(D) = 90°.
A banknote is a common So ABCD is a rectangle.
example of a rectangle.

2. Properties of a Rectangle
Since a rectangle is a type of parallelogram, it has all the properties of a parallelogram. It also
has some additional properties.

Theorem 16

The diagonals of a rectangle are congruent.

Proof Look at the figure. Given that ABCD is a


D a C
rectangle, we need to prove AC  BD.
O
b b

ABCD is a rectangle, so it is a parallelogram. A a B

AD  BC (opposite sides of a parallelogram are congruent)


AB is a common side of DAB and CBA.
DAB  CBA (both right angles by definition of a rectangle)
DAB  CBA (by SAS congruence postulate)
AC  BD (corresponding sides of congruent triangles)
Moreover, since the rectangle is a parallelogram, its diagonals bisect each other:
AO  OC  BO  OD.

So the diagonals of a rectangle are congruent and bisect each other. It can also be proven that
if the diagonals of a parallelogram are the same length then this parallelogram is a rectangle.

36 Geometry 8
EXAMPLE 30 The bisector of angle A of a rectangle ABCD intersects side DC at a point K such that
DK : KC = 4 : 3. Given that DK =16 cm, find the lengths of all sides of ABCD and its perimeter.

Solution Let x be the constant of proportionality. D 4x K 3x C

Since DK : KC = 4 : 3 we can write DK = 4x


and KC = 3x. Also, DK = 16 cm so 4x = 16;
x = 4 cm.
A B

AB  DC (opposite sides of a rectangle are parallel)


m(BAK) = m(DKA) (alternate interior angles)
DAK is isosceles (two congruent angles in DAK)
AD = DK = 16 cm (congruent legs of an isosceles triangle)
DC = DK + KC = 7x = 28 cm
AD = BC = 16 cm (opposite sides)
DC = AB = 28 cm (opposite sides)
So the perimeter of ABCD is 2  (16 + 28) = 88 cm.

EXAMPLE 31 In the figure, ABCD is a rectangle and point D E


24°
C
O is the intersection of diagonals AC and BD.
Point E is on the side DC and DO = DE. O
Given m(EOC) = 24°, calculate m(ODE).
A B

Solution ABCD is a rectangle. So the diagonals are D E C


x x+24° x
equal and bisect each other. So DO = OC. 24°

A B
Let m(OCD) = x, then
m(ODE) = m(OCD) = x (base angles in isosceles triangle DOC)
m(DEO) = x + 24° (exterior angle of triangle OCE)
m(DEO) = m(DOE) = x + 24° (base angles in isosceles triangle DOE)
In triangle DOE,
m(ODE) + m(DEO) + (DOE) = 180° (sum of interior angles)
x + x + 24° + x + 24° = 180°
3x = 132°
x = 44°.
So m(ODE) = 44°.

Quadrilaterals 37
EXAMPLE 32 A rectangle ABCD has side lengths 8 cm and 16 cm, and point O is the intersection point of
diagonals AC and BD. Find the distances from O to two consecutive sides of the rectangle.

Solution We begin by drawing the figure. The question D 16 C


asks us to find the lengths OH and ON. O
8 N

A H B

Since ON  BC, it follows that ON  AB. (lines perpendicular to the same line BC)
The diagonals bisect each other, so AO = OC. It follows that BN = NC.
AB
So ON is a midsegment of ACB and ON = ; ON = 8 cm.
2
BC
In a similar way we can show that OH is also a midsegment of ABC and OH = ; OH = 4 cm.
2
So ON = 8 cm and OH = 4 cm.

EXAMPLE 33 In the figure, ABCD is a rectangle and point D 4 E ?


C
E is on the side DC. Line segments AE and
BE are perpendicular to each other. Given 6
DE = 4 cm and BC = 6 cm, find the length
of line segment EC. A B

Solution Let us draw a line EH which is perpendicular to side AB. Then AHED and HBCE are also
rectangles. D 4 E x
C
Let EC = x, then HB = x. Also,
DE = AH = 4 cm (given), and 6 6
BC = EH = 6 cm (also given).
First Euclidean theorem: By the first Euclidean theorem, A 4 H x B

C EH = AH  HB
2

62 = 4  x
36 = 4x
A H B x = 9 cm.
In ABC, if m(C) = 90° So EC = 9 cm.
and CH  AB then The Parthenon in Athens is an
example of the architectural use of a
CH = AH  HB.
2
shape known as the golden rectangle.
The golden rectangle is thought to be
the geometric form that is most
pleasing to the human eye.

38 Geometry 8
Check Yourself
1. In the figure, ABCD is a rectangle and point E is the D C
intersection of diagonals AC and BD. If EH  AB,
EH = 3 cm and HB = 4 cm, find the length of AC. E

3
2. The bisector of angle C of a rectangle ABCD intersects
side AD at point F such that DF : FA = 3 : 2. Find the A H 4 B

perimeter of this rectangle if the length of side AB is 9 cm.


3. In the figure, ABCD is a rectangle and points A, B and D C
E are collinear. If AC = BE and m(CAE) = 36°, find
m(AED).

36°
Answers A B E

1. 10 cm 2. 48 cm 3. 18°

Theorem 17

In the figure, ABCD is a rectangle. If P is any


D C
point in or on the rectangle then P

PA2 + PC2 = PB2 + PD2.

A B

Proof Let us draw NK through point P so that it is D N C


perpendicular to both sides AB and DC as in
the figure. P

In right triangles PAK and PKB, A K B

PK2 = PA2 – KA2 and PK2 = PB2 – KB2. (Pythagorean Theorem)


2 2 2 2
So PA – KA = PB – KB . (1)
In right triangles PNC and PND,
PN2 = PC2 – NC2 and PN2 = PD2 – ND2. (Pythagorean Theorem)
So PC2 – NC2 = PD2 – ND2. (2)
Adding equalities (1) and (2) side by side gives
PA2 – KA2 + PC2 – NC2 = PB2 – KB2 + PD2 – ND2, (KA = ND, KB = NC)
which means PA2 + PC2 = PB2 + PD2 as required.

Quadrilaterals 39
Note P
D C
Theorem 17 also holds if point P lies outside the rectangle: in the
figure opposite,
PA2 + PC2 = PB2 + PD2.
A B

EXAMPLE 34 In the figure, point P is an interior point of T K

the rectangle MNKT. Given PM = 4 cm, ? 6


PN = 5 cm and PK = 6 cm, find the length P
of line segment PT. 4 5

M N

Solution By Theorem 17 we can write PT 2 + PN 2 = PM 2 + PK 2.


Let us substitute the given values in the equality:
PT 2 + 52 = 42 + 62
PT 2 + 25 = 16 + 36
PT 2 = 16 + 36 – 25
PT 2 = 27
PT = 3ñ3 cm.

EXAMPLE 35 In the figure, point P lies outside rectangle


8
2
P
ABCD and points A, C and P are collinear. If D C
PC = 2 cm, AC = 8 cm and PD = 8 cm, find
8 ?
the length of line segment PB.

Solution By Theorem 17 we can write A B

PB2 + PD2 = PA2 + PC2.


Let us substitute the given values:
PB2 + 82 = 102 + 22
PB2 + 64 = 100 + 4
PB2 = 104 – 64
PB2 = 40
PB = 2ò10 cm.

40 Geometry 8
EXAMPLE 36 ABCD is a rectangle and point E is on side DC with DE < EC. Point F is the midpoint of side
DA. Given FE  BE, FE = 12 cm and m(EBA) = 30°, find the perimeter of the rectangle.

Solution Let us draw the figure. From it we can conclude


m(ABE) = m(BEC) = 30° (DC  AB, alternate interior angles)
m(CED) = 180° (straight angle)
m(FED) = m(CED) – m(FEB) – m(BEC)
= 180° – 90° – 30° D 60° E C
30°
= 60°. 12
F
In the right triangle EDF,
30°
DF A B
sin60°  (sine ratio)
FE

DF sin60°  FE
Drivers’ licenses, credit
3
cards and membership DF   12  6 3 cm,
cards are all rectangular. 2
DE
cos60°  (cosine ratio)
FE

DE cos60°  FE

1
DE   12 6 cm.
2
So BC = DA = 2  DF = 12ñ3 cm.
In the right triangle BCE,
BC BC 12 3
tan 30  = ; EC = ; EC  ; EC = 12ñ3  ñ3 = 36 cm, so
EC tan 30  1
3
DC = EC + DE = 36 + 6 = 42 cm.
So the perimeter of ABCD is 2(12ñ3 + 42) = (84 + 24ñ3) cm.

EXAMPLE 37 In the figure, ABCD is a rectangle and point D C


E is the midpoint of side AB. AC is a diagonal
11
of the rectangle, AC = 11 cm and EC = ò46 cm. ò46
Find the lengths of the sides of the rectangle.
A E B

Quadrilaterals 41
Solution Let EB = x and BC = y. D C

So AB = 2x. 11 y
ò46
Also, B is a right angle.
A x E x B
In the right triangle BEC,
y2 + x2 = (ò46)2; y2 = 46 – x2. (1) (Pythagorean Theorem)
In the right triangle ABC,
y2 + (2x)2 = 112; y2 + 4x2 = 121. (2) (Pythagorean Theorem)
Substituting (1) in (2) gives
46 – x2 + 4x2 = 121
3x2 = 75
x2 = 25
x = 5, and

y2 = 46 – x2
y2 = 21
y = ò21.
So AD = BC = ò21 cm and DC = AB = 10 cm.

EXAMPLE 38 In the figure, ABCD is a rectangle. Point H is D 20 C


on the diagonal AC and DH is perpendicular
to AC with AD = 15 cm and DC = 20 cm.
15 H
Find the length of line segment HB.
?

A B

Solution In the right triangle ADC, D 20 C

9
AC2 = 152 + 202 (Pythagorean Theorem) 12
E
AC = 25 cm 15 7 15
Second Euclidean 12
theorem: AD2 = AH  AC (second Euclidean theorem) 9 H
C
15 = AH  25
2

A 20 B
AH = 9 cm.
A H B In the right triangle AHD,
In ABC, if m(C) = 90° DH2 = 152 – 92 (Pythagorean Theorem)
and CH  AB then
AC2 = AH  AB. DH = 12.

42 Geometry 8
Now let us construct BE such that BE  AC. Then we have
DAH  BCE (alternate angles)
ADH  CBE (third angles in right triangles)
BC  AD. (opposite sides of a rectangle)
So AHD  CEB, by the SAS congruence postulate.
So AH = EC = 9 cm and DH = BE = 12 cm, and
HE = AC – AH – EC
HE = 25 – 2  9 = 7 cm.
Finally, in the right triangle HEB,
HB2 = EB2 + EH2 (Pythagorean Theorem)
HB2 = 122 + 72
HB = ó193 cm.

Note D C
Since opposite angles of a rectangle are supplementary, we can always
draw the circumscribed circle of a rectangle. However, it is not
generally possible to construct its inscribed circle. A B

ABCD can be inscribed


but not circumscribed.

Check Yourself
D C
1. In the figure, ABCD is a rectangle and P is a point in 10 8
its interior. If PA = 9 cm, PC = 8 cm and PD = 10 cm, P
find the length of segment PB. 9 ?

A B

2. In the figure, ABCD is a rectangle, point E is the D C


midpoint of side AB and EC is the bisector of angle C.
If EC = 6ñ2 cm, find P(ABCD). 6ñ2

A E B

3. In the figure, ABCD is a rectangle and points E and F are D C


on the sides AB and DA respectively. Given CE  FE, ?
4
m(CEB) = 45°, AE = 2 cm and DF = 4 cm, find the
F
length of segment CF.
45°
Answers A 2 E B

1. 3ñ5 cm 2. 36 cm 3. 4ñ5 cm

Quadrilaterals 43
D. RHOMBUS
1. Definition
Definition rhombus
A rhombus is a parallelogram whose sides are all congruent.
D a
C

In the figure, ABCD is a parallelogram and


AB  BC  CD  DA. a a

So ABCD is a rhombus.
A a B

Many objects that need to change in shape are built in the shape
of a rhombus. The most useful property of a rhombus is that since
the lengths of the sides are the same, opposite sides remain parallel
as you change the measures of the angles. In addition, as you
change the measures of the angles, the vertices slide along the
lines of the diagonals and the diagonals remain perpendicular.

2. Properties of a Rhombus
The plural form of Since a rhombus is a parallelogram, it has all the properties of a parallelogram. It also has
rhombus is rhombi. some additional properties that are not true for all parallelograms.

Theorem 18

Each diagonal of a rhombus bisects two angles of the rhombus.

EXAMPLE 39 In the figure, ABCD is a rhombus and D


52°
C
m(CDB) = 52°. Find m(DAB).

?
Solution We know that ABCD is a rhombus and that its A B
diagonal bisects two angles (by Theorem 18).
So m(ADB) = m(CDB) = 52°
m(CDA) = 104°
m(CDA) + m(BAD) = 180° (consecutive angles in a parallelogram are supplementary)
m(DAB) = 180° – 104°
= 76°.

44 Geometry 8
EXAMPLE 40 In the figure, ABCD is a rhombus, D C
AB = BE and points D, B and E are collinear.
If m(A) = 64°, find m(BCE). ?

64°
A B

Solution ABCD is a rhombus and AB = BC = BE.


So BEC is isosceles. E
Let m(BCE) = x. Then
m(BEC) = m(BCE) = x (base angles in isosceles triangle BCE).
m(CBD) = 2x (exterior angle of BEC)
m(DAB) + m(ABC) = 180° (supplementary angles in a parallelogram)
m(ABC) = 180° – 64° = 116°
m( ABC )
m( CBD ) = (diagonal BD is the bisector of ABC)
2
116°
2x = ; 2 x = 58°; x = 29°.
2
So m(BCE) = 29°.
Rhombi protect us.

EXAMPLE 41 In the figure, ABCD is a rhombus, AC is its D


C
diagonal, DH is perpendicular to AB and
AK = KD. Find m(AKH).
K
?

A H B

Solution ABCD is a rhombus so its diagonal bisects its vertex angles. So m(HAK) = m(DAK) = x
and m(DAK) = m(ADK) = x (base angles in isosceles triangle AKD)
m(AKH) = 2x. (exterior angle of AKH)
In AKH,
m(HAK) + m(AKH) + m(AHK) = 180° (sum of interior angles)
x + 2x + 90° = 180°
3x = 90°
x = 30°.
So m(AKH) = 2x = 60°.

Quadrilaterals 45
Theorem 19

The diagonals of a rhombus are perpendicular.


Proof Look at the figure. Since ABCD is a rhombus, D
the diagonals bisect each other. So AO = OC
and BO = OD.
Triangles ABD, ABC, BCD and DAC are A
O
C
isosceles because the sides of a rhombus are
congruent by definition. In an isosceles
B
triangle, the median to the base is perpendicular
to the base and also bisects the vertex angle. So
AO  BD and m(BAO) = m(OAD)
BO  AC and m(ABO) = m(OBC)
CO  BD and m(BCO) = m(OCD)
DO  AC and m(ADO) = m(ODC).
So AC and DB are the bisectors of each pair of vertex angles, and also the diagonals are
perpendicular to each other.
It can also be shown that if either the diagonals of a parallelogram are perpendicular to each other, or
if one of the diagonals bisects two angles of the parallelogram, then the parallelogram is a rhombus.

Theorem 20

In a rhombus, the sum of the squares of the lengths of the diagonals is equal to four times
the square of the length of one side.
D

Proof In the figure, ABCD is a rhombus. AC and


BD are the diagonals and point O is the
intersection of the diagonals.
We need to prove that DB2 + AC2 = 4  AD2. A C
O
Since the diagonals bisect each other, DO = OB
and AO = OC.
DB AC
So DO = and AO = .
2 2 B

In AOD, O is a right angle. (diagonals are perpendicular)


2 2 2
So AO + DO = AD (Pythagorean Theorem)
2 2
 DB   AC  2
  +  = AD
 2   2 
DB2 AC 2
+ = AD2 . Multiplying both sides by 4 gives us
4 4
DB2 + AC2 = 4  AD2, which is the desired result.

46 Geometry 8
EXAMPLE 42 Find the perimeter of a rhombus whose diagonals measure 10 cm and 24 cm.

Solution 1 Let ABCD be the rhombus. AC and BD are D


the diagonals, AC = 10 cm and BD = 24 cm.
By Theorem 20, we have
BD2 + AC2 = 4AB2 5
A C
2 2
10 + 24 = 4AB 2 O
12
4AB2 = 676
AB2 = 169
B
AB = 13 cm.
So the perimeter of the rhombus is 4  AB = 4  13 = 52 cm.
AC BD
Solution 2 AO = and BO = (diagonals bisect each other)
2 2
So AO = 5 cm and BO = 12 cm.
Also, m(AOB) = 90°. (diagonals are perpendicular)
In the right triangle AOB,
AB2 = AO2 + OB2 (Pythagorean Theorem)
2 2 2
AB = 5 + 12
AB2 = 169
AB = 13 cm.
So the perimeter of the rhombus is 4  AB = 4  13 = 52 cm.

EXAMPLE 43 In the figure, ABCD is a rhombus, DH is D C


perpendicular to AB and AH = 3 cm,
HB = 2 cm. Find the lengths of the
diagonals of this rhombus.

A 3 H 2 B

Solution ABCD is a rhombus, so its sides are D 5 C


congruent:
5
AD = AB = AH + HB 4 5
AD = 3 + 2 = 5 cm.
A 3 H 2 B
In the right triangle AHD,
DH2 = AD2 – AH2 (Pythagorean Theorem)
DH2 = 52 – 32
DH2 = 16
DH = 4 cm.

Quadrilaterals 47
Let us construct the diagonal BD. Then The arms of the lifting platform
shown in the picture make
in the right triangle DHB, rhombus shapes. When the
DB2 = DH2 + HB2 lift is operating, the lengths
of the diagonals change but
DB2 = 42 + 22 the lengths of the sides do not
change. Can you imagine
DB2 = 20 how this lift would look and
DB = 2ñ5 cm. work if its arms formed
parallelograms that were not
By Theorem 20, DB2 + AC2 = 4AD2 rhombi?
20 + AC2 = 4  52
AC2 = 80
AC = 4ñ5 cm.
So the diagonals measure 2ñ5 cm and 4ñ5 cm.

44
C
EXAMPLE In the figure, ABC is a right triangle and
DFBE is a rhombus. Given that AB = 8 cm
and AC = 6 cm, find the length of one side 6 D E
of the rhombus.

A F B
8

Solution In the right triangle ABC,


C
BC2 = AB2 + AC2 (Pythagorean Theorem)
10 – x 10
BC2 = 82 + 62
6 D E
BC2 = 100
x
BC = 10.
A F B
Let the length of one side of the rhombus be
8
x, so EB = x and CE = 10 – x. Then
DE  FB (opposite sides of the rhombus)
m(CDE) = m(A). (corresponding angles)
m(C) is also a common angle of BCA and ECD, so CDE  CAB by the AA similarity
postulate. Therefore,
CD DE CE x 10  x
= = ; = (lengths of corresponding sides are proportional)
CA AB CB 8 10
10x = 80 – 8x
40
18 x = 80; x = cm.
9
40
So one side of the rhombus measures cm.
9

48 Geometry 8
Note
D C
Since opposite angles of a rhombus are not supplementary it is not
possible to construct the circumscribed circle of a rhombus, but it is
possible to construct the inscribed circle of a rhombus. In the figure,
ABCD is a rhombus. A B

ABCD can be circumscribed


Activity but not inscribed.

There are many simple things you can do to improve your creative thinking ability. Everyone knows that solving
puzzles is a good way to develop your creative thinking and problem solving skills. These skills are not just useful
for math: they can help you understand the world around you, too.
Here are two puzzles that you can try to solve using matchsticks or toothpicks. Good luck, and enjoy!

1. Move two matchsticks in the pattern to make 2. Move six of the matchsticks below to make a new
one rhombus and one equilateral triangle. figure made up of six congruent rhombi.

Check Yourself
1. ABCD is a rhombus and point E is on side DC such that m(BEC) = 55°. If m(A) = 100°,
find m(DBE). D

2. In the figure, ABCD is a rhombus, E is a point on side AB and CE


is perpendicular to side AB. If m(A) = 120° and EC = 2ò21, find
the perimeter of the rhombus. A 120° C

2ò21
E

F
3. In the figure, ABCD is a rhombus and points C, E, F 3
and B, A, F are respectively collinear. BA = 6 cm and A

AF = 3 cm are given. Find the length of the line 6


E
segment ED. ?
This lamp stays perpendicular to the
wall as it moves into the room. Can you B D
explain why, using your knowledge of
rhombi?

Quadrilaterals 49
4. In the figure, ABCD is a rhombus and point E is on the D C
diagonal AC. EC = 8 cm, AE = 28 cm and AB = 30 cm are 8
?
given. Find the length of DE. E

28
Answers
1. 15° 2. 16ñ7 cm 3. 4 cm 4. 26 cm A 30 B

E. SQUARE
1. Definition
Definition square
A square is a rectangle whose sides are all congruent.
In the figure, ABCD is a square since it is a D a C
rectangle and all the sides are congruent:
AB = BC = CD = DA = a.
a a
We can also define a square as a rhombus
with four right angles. In ABCD,
A chessboard is a square
board which is divided m(A) = m(B) = m(C) = m(D) = 90°. A a B
into smaller squares of
two contrasting colors.
Do you know how many 2. Properties of a Square
squares there are on a
chessboard? We can say that a square is both a rectangle and a rhombus. So it has all the properties of a
square and a rhombus, i.e.: D a C
1. Its diagonals have the same length. e
2 e
2. Its diagonals are perpendicular. 2
a a
3. Its diagonals bisect each other. e
2 e
4. Each diagonal bisects two interior angles. 2
A a B

EXAMPLE 45 In the figure, ABCD is a square and points E D C


and F are on the sides BC and AB respectively. 70°
FB is congruent to BE and m(ADE) = 70°.
Find m(DEF). ? E

A F B

50 Geometry 8
Solution m(ADE) = m(CED) = 70° (AD  BC, alternate interior angles)
Also, m(BFE) = m(BEF) and (base angles in isosceles triangle BEF)
90° D C
m( BEF ) = ; m( BEF ) = 45 
2 70°
m(BEC) = 180° (straight angle) 70°
E
m(DEF) = 180° – m(CED) – m(BEF)
45°
= 180° – 70° – 45°
= 65°. A F B

EXAMPLE 46 In the figure, ABCD is a square and BEC is D C

equilateral. Find the measure of AEB.


E
?

A B

Solution AB = BC (ABCD is a square)


BC = BE (BEC is equilateral)
So AB = BE and ABE is isosceles.
D C
Also, m(EBC) = 60° (equilateral triangle)
m(ABE) = m(B) – m(EBC) E
= 90° – 60° x
x 60°
= 30°. 30°
A B
In ABE,
m(BAE) = m(BEA) = x (base angles in ABE)
m(BAE) + m(AEB) + m(ABE) = 180° (sum of interior angles)
x + x + 30° = 180°
2x = 150°
x = 75°.
So m(AEB) = 75°.

Quadrilaterals 51
EXAMPLE 47 In the figure, ABCD is a square and point E is C B
on the diagonal DB such that CE = x + 5 and
EA = 3x – 13. What is the value of x? x+5

3x –
E 13

D A

Solution 1. CD  DA (sides of a square are congruent)


2. DE  DE (common side of CDE and ADE)
3. CDE  ADE (diagonal DB is the bisector of D)
4. CDE  ADE (by SAS congruence postulate)
5. CE  AE (corresponding sides of congruent triangles)
6. CE = AE (congruent sides have equal lengths)
7. x + 5 = 3x – 13
2x = 18
x = 9 cm

EXAMPLE 48 In the figure, ABCD is a square and PBKE is D 6 C


a rectangle. Point E is on the diagonal AC. If
DC = 6 cm and the length of EK is half of E x
K
the length of EP, find the length of EK.
2x

A P B

Solution AC bisects DAB because it is the diagonal of a square. So m(CAD) = 45°.


In APE, m(APE) = 90° and
m(PEA) = 180° – (90° + 45°) (sum of interior angles)
m(PEA) = 45°.
So AP = PE.
Also, EK = PB (opposite sides of a rectangle) D 6 C
and AP = 2  EK. (given)
AB = AP + PB; AB = 3  EK E K

AB = DC; 3  EK = 6 cm 45°

So EK = 2 cm. 45°
A P B

52 Geometry 8
Property 7

If the length of one side of a square is b then length of its diagonal is bñ2.

EXAMPLE 49 In the figure, ABCD is a square and point E is D


15°
C

on the diagonal AC such that m(CDE) = 15°. 2ñ3 E


Find the perimeter of the square if
DE = 2ñ3 cm.

A B

Solution Let us draw the diagonal DB, so


DB  AC. D C
15° 2ñ3
30°
Point O is the intersection of diagonals, and 60° E
3 ñ3
m(EDO) = m(CDO) – m(CDE) 3ñ2
O
= 45° – 15°
3
= 30°.
A B
In the right triangle DOE, 3ñ2

DO 3
cos 30  = ; DO = DE  cos 30 = 2 3  = 3 cm.
DE 2
Also, DO = OB (diagonals bisect each other)
DB = DO + OB = 6 cm.
In a square, since the length of the diagonal is ñ2 times the length of one side, we get
DB = ñ2  AB
DB 6
AB = = = 3 2 cm.
2 2

EXAMPLE 50 In the figure, ABCD is a square. Points A, C P


and P are collinear, AC = BP and the length D C
of one side of the square is 4 cm. Find the
length of line segment CP.

A 4 B

Quadrilaterals 53
Solution Let us draw the diagonal DB, so DB  AC. Point O is the intersection of the diagonals. So
BD = AC = BP = 4ñ2
OB = OD = OC = OA = 2ñ2 (diagonals bisect each other)
POB is a right triangle. (diagonals are perpendicular)
In POB,
PO2 + OB2 = PB2 (Pythagorean Theorem)
2 2 2
PO + (2ñ2) = (4ñ2) P
2
PO + 8 = 32

6
D 4 C


2
PO = 24; PO = 2ñ6 cm.


2

2
4ñ2
Finally, PC = PO – CO 4
O


2
= 2ñ6 – 2ñ2
A 4 B
= 2(ñ6 – ñ2) cm.

Note D C

Since opposite angles in a square are supplementary and


the sum of lengths of opposite sides is equal to the sum of
the lengths of the other two opposite sides, a square is both
A B
an inscribed and circumscribed quadrilateral. In the figure,
ABCD is inscribed and
ABCD is a square. circumscribed.
How many squares?
How many circles?
Check Yourself 14
How many triangles?
1. In the figure, ABCD is a square and BD is a diagonal of the B A
20°
square. Point E is on BD and m(BAE) = 20°. Find E
m(AEC).

C D

2. AC is a diagonal of a square ABCD. Points E and F are on the sides AC and AB respectively,
and FE is perpendicular to AC. Find the length of EC if AF = 4ñ2 cm and FB = 2ñ2 cm.

3. In the figure, ABCD is a square and points P, D and B are P


1
collinear. If PD = 1 cm and BD = 6 cm, find the length of D C
the line segment PC.
Answers 6

1. 130° 2. 8 cm 3. 5 cm
A B

54 Geometry 8
Activity
Copy the shapes opposite onto a piece of paper and cut
them out. Cut along the dotted lines to make four pieces
from each shape. Then try to make a quadrilateral from
each set of four pieces.

F. TRAPEZOID
1. Definition
Definition trapezoid, base, leg, base angles, altitude, height
A trazepoid is a quadrilateral which has exactly one pair of parallel sides.
The parallel sides of the trapezoid are called base
D C
the bases of the trapezoid. The other sides
base
are the legs. leg angles leg
Two angles that share a base of the trapezoid base angles
are called base angles.
A base B
In the top figure at the right, ABCD is a
quadrilateral, DC  AB and AD is not parallel
D base P C
to BC. So by the definition, ABCD is a trapezoid.
Sides DC and AB are the bases, and sides AD
altitude
and BC are the legs.
A perpendicular line segment drawn from
A H K B
any point on one of the bases to any point on
base
the other base is called an altitude of the
trapezoid. The length of any altitude is called the height of the trapezoid.
In the figure opposite, DH and PK are two altitudes of the trapezoid.

2. Properties of a Trapezoid
Theorem 21

In a trapezoid, two interior angles that share the same leg are supplementary.

Proof In the figure, ABCD is a trapezoid with DC  AB. E


We need to prove that C
D
m(A) + m(D) = 180° and
m(B) + m(C) = 180°.
If we extend AD so that points A, D and E are
A B
collinear, we get

Quadrilaterals 55
m(A) = m(EDC) (corresponding angles)
m(D) + m(EDC) = 180°. (supplementary angles)
So m(D) + m(A) = 180°, as required.
In a similar way, we can prove that m(B) + m(C) = 180°.

EXAMPLE 51 In the figure, ABCD is a trapezoid, points C,


D and E are collinear, and DC  AB.
E D C

70°
m(ABC) = 50° and m(ADE) = 70° are given.
Find the measures of all the interior angles of 50°
the trapezoid. A B

Solution m(ABC) + m(BCD) = 180° (by Theorem 21)


50° + m(BCD) = 180°
m(BCD) = 180° – 50°
m(BCD) = 130°
m(DAB) = m(ADE) = 70° (alternate interior angles)
m(ADC) + m(ADE) = 180° (supplementary angles)
m(ADC) = 180° – 70°
m(ADC) = 110°
So m(B) = 50°, m(C) = 130°, m(A) = 70° and m(D) = 110°.

EXAMPLE 52 In the figure, ABCD is a trapezoid. Point E


D C
lies inside the trapezoid and DC  AB. Given y
2y
m(EBC) = 2  m(EBA) = 2x and ?
E
m(ECB) = 2  m(ECD) = 2y, find 2x
m(CEB). x
A B

Solution m(B) + m(C) = 180° (two angles that share the same leg are supplementary)
3x + 3y = 180°
x + y = 60°
In CEB,
m(CEB) + 2x + 2y = 180° (sum of the measures of interior angles)
m(CEB) + 2(x + y) = 180°
m(CEB) = 180° – 2  60°
m(CEB) = 60°.

56 Geometry 8
Definition median of a trapezoid
The median of a trapezoid is the line segment that joins the midpoints of the legs.

In the figure, points E and F are midpoints of


D base C
the legs AD and BC respectively. So line
segment EF is the median of trapezoid ABCD. median
E F

base
A B

Theorem 22

The median of a trapezoid is parallel to the bases and its length is half of the sum of the
lengths of the bases.
Proof In the figure, ABCD is a trapezoid, DC  AB D C
and EF is the median of the trapezoid.
E F
We have to prove that EF  AB  DC and median
AB + CD
EF = . A B G
2
Let us begin by drawing DF to intersect line
AB at point G, and continue with a two-column proof:
Statements Reasons
1. DFC  GFB Vertical angles
2. DCB  GBF Alternate interior angles
3. CF = FB Definition of median
4. DFC  GFB ASA congruence postulate by 1, 2 and 3
5. DF = GF Corresponding sides of equal triangles
6. DC = BG Corresponding sides of equal triangles
7. AG = AB + BG Addition of line segments
8. AG = AB + DC By 6 and 7
9. AE = ED Definition of median
10. EF is the midline of ADG By 5 and 9
AG
11. EF  AG and EF = By 10
2
AB + CD
12. EF  AB  DC and EF = Bases are parallel, and combining 8 and 11
2

Quadrilaterals 57
EXAMPLE 53 In trapezoid ABCD in the figure, DC  AB and D x+3 C

EF is the median of the trapezoid.


8
E F
EF = 8 cm, AB = x + 1 and DC = x + 3 are
given. Find the lengths of AB and DC.
A x+1 B
AB + CD
Solution EF = since EF is the median. So
2
x +1+ x + 3
8=
2
2x + 4 = 16; 2x = 12; x = 6 cm.
So AB = 7 cm and DC = 10 cm.

54 In trapezoid ABCD in the figure, AK = KE = ED


c
EXAMPLE D C

and BP = PF = FC. E 6 F
Given AB = a, DC = c, KP = 9 cm and 9
K P
EF = 6 cm, find a and c.
A a B

Solution Since AK = KE = ED and BP = PF = FC, by Thales’ theorem we obtain DC  EF  KP  AB.


So quadrilaterals ABFE and KPCD are trapezoids.
In trapezoid ABFE,
AB + EF a +6
KP = ; 9= (KP is the median of the trapezoid)
Thales’ theorem of 2 2
parallel lines: If three or
a + 6 = 18
more parallel lines
intersect two a = 12 cm.
transversals, then they In trapezoid KPCD,
divide the transversals
proportionally. KP + DC c +9
EF = ; 6= (EF is the median of the trapezoid)
2 2
c + 9 = 12
c = 3 cm.
So a = 12 cm and c = 3 cm.

EXAMPLE 55 In trapezoid ABCD in the figure, AD  BC and C


AE and BE are the bisectors of angles A and E
B respectively. 10
D ?
Given AD = 6 cm and BC = 10 cm,
6
a. find m(AEB). A ? B
b. show that DE = EC.
c. find the length of AB.

58 Geometry 8
Solution a. m(A) + m(B) = 180° C
(supplementary angles) E
m( A ) m( B) 10
D
m( EAB) = , m( EBA ) =
2 2
6
m( A )+ m( B)
m( EAB)+ m( EBA ) = A F B
2
180
= = 90 
2
In AEB,
m(EAB) + m(EBA) + m(AEB) = 180° (sum of the measures of interior angles)
90° + m(AEB) = 180°
m(AEB) = 90°.
b. Let us draw line segment EF parallel to the bases: EF  DA  CB. Then
m(CBE) = m(FEB) (alternate interior angles)
EF = FB (congruent angles in EFB)
m(DAE) = m(AEF) (alternate interior angles)
AF = EF. (congruent angles in AFE)
So EF = AF = FB.
Since EF  AD  BC and EF bisects AB, by Thales’ theorem of parallel lines we can
conclude that EF bisects side DC.
So DE = EC, and EF is the median of the trapezoid.
AD + BC
c. Since EF is the median, EF =
2
6+10
= = 8 cm.
2
AEB is a right triangle and EF is the median to the hypotenuse of the triangle. In a right
triangle, the length of the median to the hypotenuse is half of the length of hypotenuse.
So AB = 16 cm.

EXAMPLE 56 In the figure, ABCD is a trapezoid, AB  DC, D C


and AE and DE are the bisectors of A and 6
D respectively. E
8
If CH  AB, AE = 8 cm and DE = 6 cm, find
A B
the height CH. H

Quadrilaterals 59
Solution Since AB  DC, we know from Example 83 that D P C
m(AED) = 90°. 6
N
In AED, 10
E
2 2 2
AD = DE + AE (Pythagorean Theorem) 8
2 2 2 A B
AD = 6 + 8 F H

AD2 = 100; AD = 10 cm.


Let us draw the perpendiculars EP, EF and EN so
NE = EF (AE is a bisector)
Remember:
NE = EP. (DE is a bisector)
Any point on the bisector
of an angle is equidistant So PE = EF and points P, E and F are collinear.
from the two sides of the Also, CH = PF = PE + EF; CH = 2  PE.
angle.
In AED,
AD  NE = DE  AE (Euclidean theorem)
10  NE = 6  8
24
NE =
5
48
So CH = 2  EP = 2  NE = cm. (NE = EP = EF)
5

Theorem 23

The length of the segment of the median of a trapezoid which lies between the diagonals of
the trapezoid is half the difference of the lengths of the bases.

D c C
In the figure, EF is the median of trapezoid
ABCD. AC and BD are diagonals of the
K L
trapezoid and they intersect median EF at E
a–c
F
points K and L. So by Theorem 23, 2
ac
KL  . A a B
2

EXAMPLE 57 In trapezoid ABCD in the figure, EF is the D C

median and AC and BD are diagonals. Given


that EK = 2 cm and AB = 10 cm, find the E 2 F
K L
lengths of DC and KL.

A 10 B

60 Geometry 8
Solution 1 EF  AB  DC (EF is the median of the trapezoid)
Since EF  AB and point E is the midpoint of AD, then by the triangle proportionality theorem,
point K is the midpoint of AC.
So EK is the midsegment of ACD, and
DC
EK =
2
DC = 2  EK
DC = 2  2 = 4 cm
AB – DC
KL = (by Theorem 23)
2
10 – 4
KL =
2
KL = 3 cm.
So DC = 4 cm and KL = 3 cm.
AB
Solution 2 EL is the midline of ABD. So EL = = 5 cm.
2
Also, KL = EL – EK, so KL = 5 – 2 = 3 cm.
Since EK is the midsegment of ACD, DC = 2  EK. So DC = 4 cm and KL = 3 cm.

EXAMPLE 58 ABCD is a trapezoid with AB  CD, m(D) = 130°, m(B) = 65°, AD = 10 cm and DC = 5 cm.
Find the length of AB.

Solution We begin by drawing the figure, then draw CK parallel to AD, intersecting side AB at point K
as shown below. Then quadrilateral ADCK is a parallelogram, since DC  AK and AD  CK. Also,
DC = AK = 5 cm and AD = KC = 10 cm (opposite sides of a parallelogram)
m(D) = m(CKA) = 130° (opposite angles of a parallelogram)
m(CKA) = m(B) + m(BCK) (exterior angle property of a triangle)
m(BCK) = 130° – 65°
= 65°.
KBC is isosceles, so KC = KB = 10 cm. (m(BCK) = m(B))
AB = AK + KB = 5 + 10
D 5 C
= 15 cm.
130° 65°
10 10

130° 65°
A 5 K 10 B

Quadrilaterals 61
EXAMPLE 59 In the figure, ABCD is a trapezoid with AB  DC and point E is the midpoint of AD.
Given AB = 20 cm, DC = 6 cm and EC = 12 cm, find the length of BC.

Solution Let us draw EF parallel to the bases and D 6 C


intersecting side BC at point F, so EF  AB  DC. 12

Then EF is the median of the trapezoid, E F

AB + DC
CF = FB and EF = A
2 20 B

20+6
= =13 cm.
2
In the right triangle ECF,
CF2 = EF2 – EC2 (Pythagorean Theorem)
2 2
= 13 – 12
= 169 – 144
= 25
CF = 5 cm.
So CB = 2  CF = 10 cm.

EXAMPLE 60 In the figure, ABCD is a trapezoid with AB  DC. D 12 C


AD = 10ñ3 cm, DC = 12 cm, m(A) = 30° 60°
10ñ3
and m(C) = 60° are given. Find the length
of AB. 30°
A ? B

Solution Let us draw DH and BE such that DH  AB E


D 7 5 C
and BE  DC, as shown in the figure. Then 60°
10ñ3 60°
DH = BE. (altitudes of a trapezoid) 5ñ3 5ñ3
30°
In the right triangle AHD, 30°
A 15 H 7 B
AH
cos 30  = ; AH  AD  cos 30  22
AD
3
 10 3  =15 cm, and
2
DH
sin 30  = ; DH = AD  sin 30 
AD
1
=10 3  = 5 3 cm.
2

62 Geometry 8
So EB = DH = 5ñ3 cm.
In the right triangle CEB,
EB EB
tan60   ; EC =
EC tan60 
5 3
= = 5 cm.
3
Finally,
DE = DC – EC = 12 – 5 = 7 cm and HB = DE = 7 cm. (opposite sides of rectangle HBED)
Also, AB = AH + HB = 15 + 7 = 22 cm, so AB = 22 cm.

EXAMPLE 61 In the figure, ABCD is a trapezoid with AB  DC. D c C


Point O is the intersection point of the
K N
diagonals AC and BD, and AB = a and DC = c. O
If KN is parallel to the bases then prove that
2ac
a. KO = ON. b. KN = .
a+ c A a B

Solution a. AKO  ADC by the AA similarity postulate.


Corresponding sides of similar triangles are proportional, so
AK KO
= . (1)
AD DC
Similarly, DKO  DAB by the AA similarity postulate, so
DK KO
= . (2)
AD AB
Adding equations (1) and (2) side by side gives
AK + DK KO KO
= +
AD DC AB
1 1
1= KO( + ) (since AK + DK = AD)
DC AB
1 1 1
= + . (3)
KO DC AB
In a similar way, by using the similarities BON  BDC and CON  CAB we obtain the
equation
1 1 1
= + . (4)
ON DC AB

Quadrilaterals 63
Since the right sides of equations (3) and (4) are equal, the left sides are equal, too.
So KO = ON, as required.
b. From equation (3) in part a we have
1 1 1 1 1 1
= + ; = + (finding the common denominator)
KO DC AB KO c a
1 a+ c
=
KO ac
ac
KO = .
a+ c
Since KN = KO + ON we have KN 2  KO (from part a).
2ac
So KN = , as required.
a+c

Check Yourself 15
1. In the figure, ABCD is a trapezoid with DC  AB. D C
35°
Given m(DCA) = 35° and AC = AB, find m(B).

?
A B

2. In the figure, point E is on side BC of a trapezoid ABCD with 8


D C
AB  DC. AE and DE are bisectors of angles A and D
respectively, AB = 20 cm and DC = 8 cm. Find the length
? E
of AD.

A 20 B

3. ABCD is a trapezoid with bases AB and CD. Given AB = 15 cm, BC = 6 cm, CD = 5 cm


and DA = 8 cm, find the height of the trapezoid.

4. ABCD in the figure is a trapezoid with AB  DC. Point E is D 6 C


the midpoint of AD, AB = 14 cm, DC = 6 cm and EC = 8 cm.
Find the length of BC. E
8 ?

Answers A 14 B
24
1. 72.5° 2. 28 cm 3. cm 4. 12 cm
5

64 Geometry 8
3. Isosceles Trapezoids
a. Definition

Definition isosceles trapezoid


An isosceles trapezoid is a trapezoid whose legs are congruent.

In the figure, AB  DC and AD = BC.


D base C
So ABCD is an isosceles trapezoid.
b b

A base B

b. Properties of an isosceles trapezoid


An isosceles trapezoid has all the properties of a regular trapezoid. It also has some additional
properties. Let us look at them in turn.

Theorem 24

The base angles of an isosceles trapezoid are congruent.

Proof In the figure, ABCD is an isosceles trapezoid D C

with AB  DC and AD = BC.


We have to prove A  B and C  D.
Let us draw CE so that CE  DA and point E
A E B
is on AB. Then
1. ADCE is a parallelogram (AB  DC and CE  AD)
2. AD = CE. (opposite sides of a parallelogram)
3. Since AD = BC, we have AD = BC = CE and
4. CEB  B (BC = CE)
5. A  CEB (corresponding angles)
6. A  B.
7. Since two interior angles that share the same leg are
supplementary, it follows that
m(A) + m(D) = 180° and
m(B) + m(C) = 180°, i.e.
m(D) = m(C) (m(A) = m(B))
8. D  C, which completes the proof.
It can also be shown that if the base angles in a trapezoid are congruent then the trapezoid
is an isosceles trapezoid.

Quadrilaterals 65
EXAMPLE 62 In the figure, ABCD is an isosceles trapezoid D C
with AB  DC and AH  BC. ? H
If AH is the bisector of A, find m(C).

A B

Solution m(A) = m(B) (base angles of an isosceles trapezoid)


In the right triangle ABH,
m( A )
+ m(B) + 90° = 180° (sum of interior angles of a triangle)
2
m( A )
+ m(A) + 90° = 180°
2
m(A) = 60°.
So m(A) = m(B) = 60°
m(B) + m(C) = 180°
m(C) = 180° – 60°
= 120°.

Theorem 25

The perpendicular projections of the legs of an isosceles trapezoid are congruent and the
length of each leg equals half the difference of the lengths of the bases.
Proof In the figure, ABCD is an isosceles trapezoid D C
with AB  DC and AB > DC. DH and CE are
two altitudes and AH and EB are projections
of the legs AD and BC respectively.
AB – DC A H E B
We need to show that AH = EB = .
2
1. ADH  BCE (SAS congruence postulate)
2. AH  EB (corresponding sides of congruent triangles)
3. DC = HE (opposite sides of a rectangle)
4. AH + EB = AB – HE
2  AH = AB – DC
AB – DC
AH =
2
AB – DC
So AH = EB = .
2

66 Geometry 8
EXAMPLE 63 ABCD is an isosceles trapezoid with AB  DC. Given AB = 22 cm, DC = 12 cm and BC = 13 cm,
find the height of the trapezoid.

Solution Let us draw the altitude CH  AB as shown D 12 C


in the figure.
13
AB – CD
Then HB = by Theorem 25, i.e.
2
A H5 B
22 – 12
HB = = 5 cm. 22
2

In the right triangle BHC,

CH2 + HB2 = CB2 (Pythagorean Theorem)

CH2 = 132 – 52

= 169 – 25

= 144

CH = 12 cm.

So the height of the trapezoid is 12 cm.

Theorem 26

The diagonals of an isosceles trapezoid are congruent.

Proof In the figure, ABCD is an isosceles trapezoid D C


with AB  DC and AD = BC.

We have to prove AC  BD.


A B

Statements Reasons
1. A  B Base angles of an isosceles trapezoid
2. AD = BC Legs of an isosceles trapezoid
3. AB = AB Common side of ABC and BAD
4. ABC  BAD SAS congruence postulate
Corresponding sides of congruent triangles
5. AC  BD
are congruent.

We can also conclude that if the diagonals of a trapezoid are congruent then this trapezoid
is an isosceles trapezoid.

Quadrilaterals 67
EXAMPLE 64 ABCD is an isosceles trapezoid with AB  DC. D C
If m(BAC) = 45°, m(CAD) = 15° and

2

AC = 6ñ2 cm, find the lengths of the sides of

°
15
the trapezoid.
45°
A B

Solution Let us draw the altitude CH  AB, as in the D C

figure. Then AHC is an isosceles right triangle, 45°

30°
2

and AH = HC. Also,
6

°
15
CH2 + AH2 = AC2 (Pythagorean Theorem) 45° 60°
A 6 H B
2  CH = (6ñ2)
2 2

2  CH2 = 72
CH2 = 36, CH = 6 cm. So AH = 6 cm.
We also know m(A) = m(B) = 60°, since these are base angles of an isosceles trapezoid.
So in CHB,
HB
cot 60  = ; HB = CH  cot 60 
CH
3
HB = 6  = 2 3 cm, and
3
HB HB
cos60  = ; CB =
CB cos60 
2 3
CB = = 4 3 cm.
0,5
Finally, AB = AH + HB = (6 + 2ñ3) cm, and
AB – CD
HB = ; CD = AB – 2  HB
2
= 6 + 2ñ3 – 4ñ3
= (6 – 2ñ3) cm. So the sides measure (6 + 2ñ3) cm and (6 – 2ñ3) cm.

Note
In an isoceles trapezoid, the perpendicular drawn from the midpoint c c
2 P 2
of one base bisects the other base and passes through the intersection D C

point of the diagonals. This line is called the axis of symmetry of the
O
trapezoid. In the figure, ABCD is an isosceles trapezoid with AB  DC,
and PH is its axis of symmetry.
A a H a B
2 2

68 Geometry 8
Theorem 27

If the diagonals of an isosceles trapezoid are perpendicular to each other then the height of
the trapezoid is equal to half the sum of the lengths of the bases.
In the isosceles trapezoid ABCD in the figure, D C
AB  DC, AD = BC and AC  DB. O

AB + DC
So by Theorem 27, CH = .
2
A H B

EXAMPLE 65 An isosceles trapezoid has diagonals which are perpendicular to each other. Given that the
bases measure 8 cm and 16 cm, find the height of this trapezoid.

8
Solution The figure shows an isosceles trapezoid ABCD D C
with AB  DC. O

Let us draw the altitude CH so CH  AB.


Since AC  DB, by Theorem 27 we can write
A B
AB + DC H
CH =
2 16

16+8
= =12 cm.
2
This is the height of the trapezoid.

Theorem 28

If the diagonals of an isosceles trapezoid are


D C
perpendicular to the legs then the height of
the trapezoid is half the square root of the
difference of the squares of the bases.

A E H B
In the figure, ABCD is an isosceles trapezoid
with AB  DC and AC  BC.
AB2 – DC 2
So by Theorem 28, CH = .
2

EXAMPLE 66 ABCD is an isosceles trapezoid with AB  DC. Given AC  BC, AB = 10 cm and DC = 6 cm,
find the height of this trapezoid.

Quadrilaterals 69
Solution 1 Let us draw the altitude CH, so CH  AB. 6
D C
Since the diagonal AC is perpendicular to BC,
by Theorem 28 we can write
AB2 – DC 2
CH = A 8 H 2 B
2
10
10 2 – 6 2 64
= = = 4 cm.
2 2
AB – DC 10 – 6
Solution 2 HB = = = 2 cm
2 2
AH = AB – HB
= 10 – 2 = 8 cm
In the right triangle ACB,
CH2 = AH  BH (first Euclidean theorem)
=82
= 16; CH = 4 cm.

Theorem 29

In an isosceles trapezoid, the difference of the squares of the lengths of a diagonal and a leg
is equal to the product of the lengths of bases.
In the figure, ABCD is an isosceles trapezoid
c
with AB  DC, AB = a, DC = c, BC = b and D C
AC = e. b e b
So by Theorem 29, e – b = a  c.
2 2

A B
a

Proof Let us draw the altitude CH = h, as shown in


c
the figure. D C

a–c b e b
By Theorem 25, HB = . h
2
a–c
Also, AH = AB – HB so AH = a – . A H B
2
a+ c a
So AH = .
2
Applying the Pythagorean Theorem to AHC and CHB gives us
2 2
 a+c  a– c 
e2 = h2 +   and b2 = h2 +   .
 2   2 

70 Geometry 8
Subtracting the second equation from the first, we get
2 2
 a+ c   a – c 
e 2 – b2 =   – 
 2   2 
 a+ c a – c   a+c a – c 
= +   – 
 2 2   2 2 
 a+ c+ a – c   a+c – a+c 
=   
 2   2 
e2 – b2 = a  c, as required.

EXAMPLE 67 The bases of an isosceles trapezoid measure 8 cm and 12 cm respectively. If the diagonal of
the trapezoid measures 14 cm, find the length of its legs.

Solution 1 By Theorem 29 we have 8


D C
AC2 – CB2 = AB  CD
14
CB2 = 142 – 12  8
= 196 – 96 A H B
= 100; CB = 10 cm. 12

Solution 2 Let us draw the altitude CH, so CH  AB. Then


AB – DC 12 – 8
HB = = = 2 cm.
2 2
Also, AH = AB – HB
= 12 – 2 = 10 cm.
In the right triangle ACH,
CH2 = AC2 – AH2 (Pythagorean Theorem)
2 2
= 14 – 10
= 96; CH = ò96 cm.
In the right triangle BCH,
BC 2 = CH 2 + BH 2 (Pythagorean Theorem)
2
= 96 + 2
= 100; BC = 10 cm. This is the length of the leg. Expensive trapezoids

Check Yourself 16 D C
?
1. In the figure, ABCD is an isosceles trapezoid with DC  AB, 72°
and the diagonal AC bisects A. If m(BCA) = 72°, find
m(DCA).
A B

Quadrilaterals 71
D ? C
2. In the figure, ABCD is an isosceles trapezoid with AB  DC,
CH  AB, AH = 7 cm and HB = 3 cm. Find the length of DC.

3. ABCD is an isosceles trapezoid with AB  DC, AB = 20 cm, A 7 H 3 B

BC = 10 cm and CD = 8 cm. Find the length of BD.


Answers
1. 36° 2. 4 cm 3. 2ò65 cm

4. Right Trapezoids
a. Definition

Definition right trapezoid


A right trapezoid is trapezoid which contains a right angle.

In the figure, AB  DC and m(A) = m(D) = 90°.


D c C
So by the definition, ABCD is a right trapezoid.
d b

A a B

b. Properties of a right trapezoid


A right trapezoid has all the properties of an ordinary trapezoid, and also some additional
properties. Let us look at one important property.

Theorem 30

If the diagonals of a right trapezoid are perpendicular to each other then the height is equal
to the square root of the product of the lengths of the bases.

Proof In the figure, ABCD is a right trapezoid with


D C
AB  DC, AD  AB and AC  DB.
We need to prove that AD = AB  DC .

Let us draw DH from point D such that


AC  DH and H is a point on the extension of H A B

AB.

72 Geometry 8
Then, since DH  AC and AC  DB we have DH  DB.
If one of two parallel lines Also, DC = HA since ACDH is a parallelogram.
is perpendicular to a line
l then the other parallel In the right triangle HDB,
line is also perpendicular
to l. DA2 = HA  AB (second Euclidean theorem)

DA = HA  AB = DC  AB. (HA = DC)

So DA = DC  AB as required.

EXAMPLE 68 A right trapezoid has perpendicular diagonals and base lengths 4 cm and 9 cm. Find the
height of this trapezoid.

Solution In the figure, ABCD is a right trapezoid with D 4 C

AB  DC, AD  AB and AC  DB.


By Theorem 30, DA = DC  AB ?
= 49
= 6 cm.
A 9 B

EXAMPLE 69 In the figure, ABCD is a right trapezoid with


D 4 C
AB  DC and AC  BC. Given AB = 13 cm and
DC = 4 cm, find the height of the trapezoid.
?

A 13 B

Solution Let us draw the altitude CH. Then D 4 C

AH = DC = 4 cm (opposite sides of a rectangle)


HB = AB – AH 4 9
A H B
= 13 – 4 = 9 cm.
In the right triangle BCA, 13

CH2 = HA  HB (first Euclidean theorem)


=49
= 36
CH = 6 cm. This is the height of the trapezoid.

Quadrilaterals 73
70
D 10 C
EXAMPLE In the figure, ABCD is a right trapezoid with
AB  DC. AC is the bisector of A, BC = 8 cm 8
and DC = 10 cm.
Find the length of AB. A B

D 10 C

Solution m(BAC) = m(ACD) because they are 10


alternate interior angles. So ADC is isosceles 8

and AD = DC = 10 cm.
A x – 10 H 10 B
Let us draw the altitude DH. Then x

DH = BC = 8 cm (opposite sides of rectangle HBCD)


HB = DC = 10 cm (opposite sides of rectangle HBCD)
AH = AB – HB.
Let AB = x with x > 10. So AH = x – 10.
In the right triangle AHD,
AD2 = DH2 + AH2 (Pythagorean Theorem)
2 2 2
10 = (x – 10) + 8
100 = x2 – 20x + 100 + 64
x2 – 20x + 64 = 0
(x – 4)(x – 16) = 0 (factorize)
x = 4 or x = 16.
x = 4 cannot be a solution because x > 10. So AB = x = 16 cm.

EXAMPLE 71 In a right trapezoid ABCD, AD  BC and AD  AB. AD = 8 cm, BC = 15 cm and AB = 23 cm


are given. Additionally, point H is the midpoint of side DC and point P is on the side AB such
that PH  DC. Find the length of PB.

Solution Remember that in an isosceles triangle, the C


median to the base is also the altitude to the H
base. D
15
Let us draw DP and PC as in the figure. Then 8
DPC is an isosceles triangle, because PH  DC
A 23 – x P x B
and DH = HC.
So DP = PC.
Also, AP = AB – PB.
Let PB = x, then AP = 23 – x.

74 Geometry 8
In the right triangles PAD and PBC,
PD2 = AD2 + AP2 and PC2 = PB2 + BC2 (Pythagorean Theorem)
AD2 + AP2 = PB2 + BC2 (PD = PC)
82 + (23 – x)2 = x2 + 152
64 + 529 – 46x + x2 = x2 + 225
46x = 593 – 225
46x = 368
x = 8 cm.
So PB = 8 cm.

Note
A trapezoid can be both an inscribed and circumscribed quadrilateral. An isosceles trapezoid is
always an inscribed quadrilateral but not always a circumscribed quadrilateral. A right trapezoid
is never an inscribed quadrilateral, but it may be a circumscribed quadrilateral.
In each figure, ABCD is a trapezoid.
D C D C
D C

A B
A B
A B

inscribed and inscribed but circumscribed but


circumscribed not circumscribed not inscribed

Check Yourself 17
1. The bases of a right trapezoid measure 5 cm and 8 cm respectively. If the height of the
trapezoid is 4 cm, find the lengths of its legs.
2. In the figure, ABCD is a right trapezoid, H is a point on the D 9 C
diagonal DB and AH  DB. AB = 20 cm, BC = 12 cm and
H
DC = 9 cm are given. Find the length of AH.
12
?

A 20 B
3. In the figure, ABCD is a right trapezoid with AB = BC, 3
D C
CD = 3 cm and AD = 3ñ3 cm. Find m(ABC).

3ñ3

Answers
?
1. 4 cm and 5 cm 2. 16 cm 3. 60° A B

Quadrilaterals 75
G. KITE
1. Definition

Definition kite
A kite is a quadrilateral with two pairs of consecutive congruent sides and two non-congruent
opposite sides.

In the figure, AB  AD and BC  CD, and also A


AB  CD. So the quadrilateral ABCD is a kite.

As we can see, a kite consists of two isosceles


triangles with a common base BD.
A square and a rhombus can also be divided
Not all flying kites are in B D
E
the shape of a kite! into two isosceles triangles with a common
base. Therefore, the properties of a kite are
C
similar to some of the properties of a rhombus
and a square.

2. Properties of a Kite
Theorem 31

The two angles formed by the non-congruent sides of a kite are congruent angles.

Proof In the figure, ABCD is a kite, AB = AD and


A
CB = CD.
We need to show that B  D.
SSS (Side Side Side) B D
postulate: If the three Let us draw the diagonal AC. Since
sides of one triangle can AB = AD, (given)
be paired with the three
sides of another triangle CB = CD (given)
such that the sides in
and AC = AC, C
each pair are congruent,
then the triangles are then by the SSS postulate, ABC  ADC.
congruent.
So B  D, as required.

76 Geometry 8
Theorem 32

The diagonals of a kite are perpendicular.


Proof In the figure, ABCD is a kite, AB = AD and
D
CB = CD.
We need to show that AC  BD.
A O C
Let us draw the diagonals AC and BD. Then
ABC  ADC by the SSS postulate.
Therefore, B

DCA  BCA (corresponding angles of congruent triangles)


CDB  CBD (base angles of the isosceles triangle DCB)
m(CDB) + m(CBD) + m(BCD) = 180° (sum of interior angles)
2  m(CDB) + 2  m(OCD) = 180°
m(CDB) + m(OCD) = 90°.
In DOC,
m(CDB) + m(OCD) + m(DOC) = 180° (sum of interior angles)
90° + m(DOC) = 180°
m(DOC) = 90°.
So AC  BD, which is the required result.
Notice also that since DCB is isosceles and DB  AC, DO = OB.

EXAMPLE 72 In the figure opposite, ABCD is a kite with


E
AB = AD and CB = CD. D 55°
If m(A) = 4  m(C) – 10° and m(EDC) = 55°,
find the measure of each interior angle of the A C

kite.
B

Quadrilaterals 77
Solution m(ADC) + m(EDC) = 180° (supplementary angles)
m(ADC) = 180° – 55°
= 125°
m(ADC) = m(B) = 125°
In quadrilateral ABCD,
m(ADC) + m(B) + m(A) + m(C) = 360° (sum of interior angles)
2  125° + 4  m(C) – 10° + m(C) = 360°
5  m(C) = 120°
m(C) = 24°.
Finally, m(A) = 4  m(C) – 10° (given)
= 4  24° – 10°
= 86°.

EXAMPLE 73 In a kite ABCD, AB = AD, CB = CD and E is the intersection point of the diagonals. If
m(EAD) = 30° and BE = 3 cm, find the length of AD.

Solution In the figure, BD  AC and BE = ED. A


30°
So ED = 3 cm.
?
In the right triangle AED, side ED is
opposite the 30° angle and we know from 3
B D
trigonometry that the length of the side E
opposite 30° is half the length of hypotenuse.
So AD = 2ED = 6 cm. C

EXAMPLE 74 In the figure, ABCD is a kite, AB = AD and D


CB = CD. Points E and F are the midpoints of
E
sides AD and CB respectively. Find the perimeter 5
A O C
of the kite if EO = 5 cm and OF = 3 cm. 3
F
Solution The diagonals of a kite are perpendicular to B
each other, so BD  AC.
Also, OE and OF are medians of the right triangles AOD and COB. In a right triangle, the
length of the median to the hypotenuse is equal to half the length of the hypotenuse.
So AE = ED = EO = 5 cm, and AD = 10 cm.
Similarly, BF = FC = OF = 3 cm and so BC = 6 cm.
So P(ABCD) = AB + AD + CB + CD
= 2AB + 2BC (AB = AD and BC = CD)
= 2(10 + 6)
= 32 cm.

78 Geometry 8
EXAMPLE 75 In the figure, ABCD is a kite, AB = AD,
CB = CD and BEC is an equilateral triangle.
D
?
If m(A) = 70° and m(ABE) = 40°, find E
A 70° C
m(ADE).
40°

Solution DC = BC = EC = EB (sides of an equilateral triangle)


m(EBC) = m(BCE) = 60° (interior angles of an equilateral triangle)
m(ABC) = m(ABE) + m(CBE) D

= 40° + 60° 25° 75°


30°
= 100° A 70° E C
60°
m(B) = m(D) = 100° 40° 60°

In quadrilateral ABCD, B

m(A) + m(B) + m(C) + m(D) = 360° (sum of interior angles)


2  100° + m(C) + 70° = 360°
m(C) = 90°
m(DCE) = m(DCB) – m(ECB)
= 90° – 60°
= 30°.
DCE is isosceles because EC = CD, and so
m(DEC) + m(EDC) + m(ECD) = 180° (sum of interior angles)
2  m(EDC) + 30° = 180°
m(EDC) = 75°.
Finally, m(ADE) = m(ADC) – m(EDC)
= 100° – 75°
= 25°.

EXAMPLE 76 In the figure, ABCE is a kite with AE = EC. A


6
Given AE = 6 cm, ED = 9 cm, CD = 5 cm E
x
and AB = x, find the value of x. 9

B C 5 D

Quadrilaterals 79
Solution We know AB = BC = x since ABCE is a kite. A
6
Let us draw the diagonal BE. x
E
9
Then BE bisects ABC and CEA.

So by the angle bisector theorem in ABD we B x C 5 D


Angle bisector theorem:
have
If a line segment bisects
an angle of a triangle AB AE
then it divides the
=
BD ED
opposite side into segments
proportional to the other x 6
two sides of the triangle. =
x +5 9
A 9x = 6x + 30
3x = 30
x = 10 cm.

B N C The pattern of the Piazza del Campidoglio in


Rome includes many kite shapes.
BN AB
=
NC AC

Note
D
A kite is always a circumscribed
D
quadrilateral, but it is not always an
inscribed quadrilateral because
A C A C
opposite angles of a kite may not be
supplementary. In the figures, ABCD B
is a kite.
B
circumscribed but inscribed and
not inscribed circumscribed

Activity
Cut four congruent right triangles from a piece of paper. Show how the four
triangles can be put together to make each quadrilateral.
1. a rhombus
2. a rectangle
3. a parallelogram that is neither a rhombus nor a rectangle
4. a trapezoid
5. a kite

80 Geometry 8
Check Yourself 18
1. In a kite ABCD, AB = AD and CB = CD. If m(B) = 40° and m(C) = 110°, find m(D).
2. In the figure, ABCD is a kite, AB = AD and m(BDC) = 60°. A
If CD = 12 cm and DA = 3ñ7 cm, find the length of AC.
3ñ7
E
B D
60°

12

C
3. In the figure, ABCD is a trapezoid with AB  DC, and ABCE D 17 C
is a kite with EA = AB. If CD = 17 cm and DE = 7 cm, find
7
the length of AB. E

A ? B

4. In the figure, ABC is a triangle and AFED is a kite with FE = ED. C


Points F, E and D are on the sides AB, BC and CA respectively, 7 8
and FB = 4 cm, BE = 6 cm, EC = 8 cm and CD = 7 cm are D
E
given. Find the length of AF.
6

Answers A ? F 4 B

1. 40° 2. 9ñ3 cm 3. 10 cm 4. 5 cm

Quadrilaterals 81
3. In the figure, A
BC = 10 cm and DC = 4 cm. D
A( DEC )
Find the value of the ratio . 4
A( ABC )
B E C
10
4. What is the area of quadrilateral FGED in the figure? A

3
D E
2
F G
1
B C
A( ABC ) 8
5. Prove Property 9.3: if ABC  DEF then = k2 .
A( DEF )
Answers
4 32
1. 9 2. 18 cm 3. 4.
25 3

H. THE TRIANGLE PROPORTIONALITY THEOREM AND


THALES’ THEOREM
1. The Triangle Proportionality Theorem
Theorem Triangle Proportionality Theorem
A line parallel to one side of a triangle which intersects the other two sides of the triangle at
different points divides these two sides proportionally. In other words, in the figure below,
m x
= .
n y
A
Proof Look at the figure.
Given: DE  BC
m x
AD AE
Prove: =
DB EC D E
ABC  ADE (AA Similarity Postulate)
n y
AB AC
So = .
AD AE
B C
Let AD = m, DB = n, AE = x and EC = y.
m + n x+ y
Then =
m x
n y
1+ =1+
m x
n y DB EC AD AE
= . So = , and so = , as required.
m x AD AE DB EC

82 Geometry 8
Conclusion
Using the properties of ratio in the previous figure, we can conclude that if DE is parallel to
AD AE AB AC AB AC
BC then = , = and = .
DB EC DB EC AD AE

Theorem Converse of the Triangle Proportionality Theorem


If a line divides two sides of a triangle proportionally then it is parallel to the third side of the
triangle.
A
Proof Look at the figure.
AD AE
Given: =
DB EC
D E
Prove: DE  BC

B C

AD AE DB AD EC AE AB AC
= + = + =


DB EC AD AD AE AE AD AE
Given Properties of Simplification
proportion (using the figure) D  B
ABC  ADE DE  BC
E  C

SAS Similarity Definition of Converse of the


Theorem similarity Corresponding
BAC  DAE Angles Theorem

Common angle

EXAMPLE 77 In the figure, TS  AC, BT = 6 cm,


T
A

BS = 9 cm, AB = 2x + 4 and
BC = 5x. Find SC. 6
B
Solution Since TS  AC, by the Triangle Proportionality 9
Theorem we can write S
C
BT BS
=
BA BC
6 9
=
2 x + 4 5x
10 x = 6 x +12; 4 x =12; x = 3 cm.
So BC = 5x = 5  3 = 15 cm and
SC = BC – BS = 15 – 9 = 6 cm.

Quadrilaterals 83
EXAMPLE 78 In the figure, AK = 12 cm, KB = 4 cm, A
AC = 20 cm, NC = 5 cm, BC = 24 cm
and MC = 6 cm. Show that KN  BC and 12
MN  AB. 20
K N
Solution To show that the lines are parallel, it is
4 5
sufficient by the Converse of the Triangle
Proportionality Theroem to show that B M 6 C

AK AN BM AN 24
= and = .
KB NC MC NC
Since AN = AC – NC,
AN = 20 – 5 = 15 cm.
Similarly, BM = 18 cm.
AK 12 AN 15 AK AN
So = = 3 and = = 3. So = , and so by the Converse of the Triangle
KB 4 NC 5 KB NC
Proportionality Theorem, KN  BC.

BM 18 AN 15 BM AN
Also, = = 3 and = = 3, so = and so by the same theorem, MN  AB.
MC 6 NC 5 MC NC

EXAMPLE 79 In the triangle ABC at the right,


3
A

EF  AC, AK = KD, BD = 2DC and AE = 3 cm. E

Find the length of segment EB. ?


K

B D F C

DK DF
Solution In ADC, = =1. (Triangle Proportionality Theorem and AK = KD)
AK FC
So DF = FC.

Let DF = FC = y, then BD = 2CD = 4y.


AE FC
So in ABC we have = (Triangle Proportionality Theorem)
EB BF
3 y
=
EB 5y
EB =15 cm.

84 Geometry 8
EXAMPLE 80 In ABC at the right, AF = FE, DB = 5 cm, A
BE = 4 cm and EC = 6 cm. Find the length
?
of AD.
D
F
Solution First we find a point K on AB such that 5
DC  KE. Then in DBC,
B 4 E 6 C
BE BK
= . (Triangle Proportionality Theorem)
EC KD
4 BK
So = A
6 5 – BK
6  BK = 20 – 4  BK
10  BK = 20 D
F
BK = 2 cm.
K
Hence KD = BD – BK = 5 – 2 = 3 cm.
B 4 E 6 C
On the other hand, in AKE we have
AF AD
= =1. (Triangle Proportionality Theorem and AF = FE)
FE DK
So AD = 3 cm.

2. Thales’ Theorem of Parallel Lines


Theorem Thales’ Theorem
If two transversals intersect three or more parallel lines then the parallel lines divide the
transversals proportionally. This theorem is known as Thales’ Theorem.

A D
Proof Look at the figure.
Given: AD  BE  CF B K E
AB DE
Can you see the
Prove: =
BC EF C L F
proportional lengths?
First we draw a line which is parallel to AC and
passes through D. Let us label the intersection
A D points K and L of this new line with BE and CF.
Then BKDA and CLKB are parallelograms, since if the opposite sides of a quadrilateral are
parallel then the quadrilateral is a parallelogram. So DK = AB and KL = BC. (1)

B E DK DE
Since KE  LF, by the Triangle Proportionality Theorem in DLF we have = . (2)
KL EF
AB DE
C F Substituting (1) into (2) gives us = , as required.
BC EF

Quadrilaterals 85
EXAMPLE 81 In the figure, AS  BR  CM  DN.
4
A
m
S

Find the lengths m, n, x and y using the B K R 3


information in the figure. x 3 4
C P M
Solution Since AS  BR  CM  DN and AD, AN and y n
5
NS are tranversals, we can apply Thales’ D N
Theorem:
AK SR m 3 9 KP RM 3 4 15
= ; = ; m = and = ; = ; n= .
KP RM 3 4 4 PN MN n 5 4
AB SR 4 3 16 AB SR 4 3 20
  ; = ; x= and ; = ; y= .
BC RM x 4 3 CD MN y 5 3

Check Yourself 7
1. Find the value of m  n in the figure. A

m 6
D E
7 n

B C
2. In the figure, DE  AB. A
x+1
Find the value of x.
D
x–2

B x+5 E x C

3. In the figure, DE  BC, DF  BE, A


AD = 12, DB = 4 and
AF = 6. Find the lengths x and y.
6
12
F
x
D E
4 y

B C

4. Find the value of x in the figure. A


3
5 K
x

E D
F
4
C
B

86 Geometry 8
5. In the figure, DC  EF  AB.
DE = 50 cm, EA = 70 cm, CF = x and D C
and FB = x + 20 cm are given. Find
the value of x.

E
F

A
B

Answers
8 12
1. 42 2. 5 3. x = 2, y = 4. 5. 50 cm
3 5

I. FURTHER APPLICATIONS
1. Menelaus’ Theorem
Theorem Menelaus’ Theorem
Let ABC be a triangle. If a line d intersects
A
the two sides AB and AC and the extension d
S
of the third side BC of ABC at points R, S R
PB CS AR
and P respectively, then   =1. P
PC SA RB B C

A
Proof Let us draw the line k through point B and
d
parallel to side AC (Parallel Postulate), and let S

T be the intersection point of lines k and d. R


T
Then PBT  PCS by the AA Similarity P
Postulate. B C
k
Menelaus of Alexandria
(c. 40-140 AD) was a
So PB = BT . (1)
PC CS
Greek mathematician
and astronomer. He was Moreover, BRT  ARS by the AA Similarity Postulate.
the first mathematician
to describe a spherical So BR = BT . (2)
triangle, and proved the AR AS PB BT
theorem described here
Dividing (1) by (2) side by side gives PC = CS ; PB  AR = AS ; PB  AR  CS =1.
in his book Sphaerica, BR BT PC BR CS PC BR AS
which is the only book he PB CS AR
wrote that has survived. So   =1, as required. AR AS
PC SA RB

Quadrilaterals 87
3. Look at the figure.
A D
Given: ABC  DEF, and AH and DP
are the altitudes to sides BC and EF
respectively.
Prove: AH = DP
B H C E P F
BAH  EDP (B  E and
BHA  EPD = 90°)
AB  DE (CPCTC)
B  E (CPCTC)
ABC  DEF (ASA Congruence Theorem)
So AH = DP because CPCTC.

Theorem
If a line parallel to one side of a triangle bisects another side of the triangle, it also bisects the
third side.

A
Proof Let us draw an appropriate figure.
Given: Line d bisects AB and d  BC.
Prove: Line d bisects AC. N
d

Pasch's Postulate  Since line d bisects AB and is parallel to


If a line intersects one BC, by Pasch’s Postulate it cuts side AC of
side of a triangle, then it B C
must also intersect one the triangle. Let K be the point of
of the other two sides. intersection with AC.
Now we have to show that AK = KC.
Let us draw a line through K which is parallel to AB and cuts BC at the point T, as shown
in the figure below.
 Since parallel line segments between two A
parallel lines are congruent, KT = NB.
 KT = AN because AN = NB. N K
d
 KTC  ABC because they are
corresponding angles.
 CKT  KAN because they are also B T C

corresponding angles.
 So by the ASA Congruence Theorem, KCT  AKN.
 AK  KC because CPCTC. So AK = KC.

88 Geometry 8
Activity Paper Folding - Midsegments in a Triangle
 Cut out a triangle and label its vertices K, M and N. K

A B
 Fold M onto K to find the midpoint A of KM.
Similarly, fold N to K to find the midpoint B of KN.
M N

A B

M K N
 Fold K to MN on AB. Then fold M to K and fold N
a b
to K. A B

What can you say about the lines AB and MN in K

relation to each other? How do their lengths M a a b b N


compare? Repeat the activity with a different
a b
triangle. Are the same things true? A B

K
M N
a b

Theorem Triangle Midsegment Theorem


The line segment which joins the midpoints of two sides of a triangle is called a midsegment
of the triangle. It is parallel to the third side and its length is equal to half the length of the
third side.

Proof Let us draw an appropriate figure. A

Given: AE = EB and AF = FC
E F
BC
Prove: EF  BC and EF =
2
Let us begin by drawing a line through E
B C
parallel to the line BC.

Quadrilaterals 89
By the previous theorem, this line will pass A

through the midpoint F of the side AC.


E F
So EF  BC.
Now let us draw a line parallel to AB which
passes through F.
B T C
By the previous theorem, it passes through
the midpoint T of the side BC. Now,
BAC  TFC (Corresponding angles)
AF  FC (Given)
AFE  FCT. (Corresponding angles)
So by the ASA Congruence Theorem, EAF  TFC, and so EF = TC because CPCTC.
BC
Also, since T is the midpoint of BC, TC = .
2
BC
So EF = TC = , which completes the proof.
2

EXAMPLE 82 In a triangle ABC, P and R are the midpoints of AB and BC, respectively. AC = 3x – 1 and
PR = x + 2 are given. Find PR.

A
1
Solution  PR = AC (Triangle
2 Midsegment
P
Theorem) 3x – 1
1
 x + 2 =  (3x – 1) (Substitute) x+2
2
 x=5 (Simplify) B R C
So PR = 5 + 2 = 7.

Theorem Angle Bisector Theorem


The distances from a point lying on the bisector of an angle to each side of the angle are
congruent.

Proof Look at the figure. B


H
Given: BOC  AOC,
C
PH  OB and P
The distance from a PN  OA
point A to a line m is
the length of the line Prove: PH  PN
segment AB such that O N A
B  m and AB  m.

90 Geometry 8
EXAMPLE 83 Write whether each statement is true or false
according to the figure opposite. D B
A
a. Point T is in the interior of DFE. K
M
b. M  ext BDE F E
T
c. ADF  BED =  P C
d. ext FDE  int FCE = FCE
e. Points T and K are in the exterior of DFE.

Solution a. false b. true c. false d. false e. true

Check Yourself 2
Answer according to the figure. A

a. Name five points which are on the triangle. L


b. Name three points which are not on the triangle. J
N S
c. Name two points which are in the exterior of the triangle.
d. What is the intersection of the line ST and the triangle
B T C
ABC?
e. What is the intersection of the segment NS and the exterior of the triangle ABC?
Answers
A physical model of a a. points A, B, C, T and S b. points J, L and N c. points J and L d. points S and T
triangle with its interior
region e. 

2. Auxiliary Elements of a Triangle


Auxiliary elements are Three special line segments in a triangle can often help us to solve triangle problems. These
extra or additional
segments are the median, the altitude and the bisector of a triangle.
elements.
a. Median
Definition median
In a triangle, a line segment whose endpoints are a vertex and the midpoint of the side
opposite the vertex is called a median of the triangle.
In the figure, the median to side BC is the A
line segment AD. It includes the vertex A and
the midpoint of BC.
Va

B D C

Quadrilaterals 91
We usually use the capital letter V to indicate A
the length of a median. Accordingly, the
lengths of the medians from the vertices of a Va
triangle ABC to each side a, b and c are Vb Vc
written as Va, Vb and Vc, respectively. As we
can see, every triangle has three medians. B D C

EXAMPLE 84 Name the median indicated in each triangle and indicate its length.
a. b. c.
K S P

D E

L M T V
R F N

Solution a. median MD, length Vm


b. median TE, length Vt
c. median PF, length Vp

Activity Paper Folding - Medians


1. Follow the steps to construct the median of a triangle by paper folding.

Take a triangular piece of paper Fold the paper again from the DM is the median of EF.
and fold one vertex to another midpoint to the opposite vertex.
vertex. This locates the
midpoint of a side.
2. Cut out three different triangles. Fold the triangles carefully to construct the three
medians of each triangle. Do you notice anything about how the medians of a triangle
intersect each other?

92 Geometry 8
Definition centroid of a triangle
The medians of a triangle are concurrent. Their common point is called the centroid of the
triangle.
The centroid of a triangle is the center of gravity of the triangle. In other words, a triangular
model of uniform thickness and density will balance on a support placed at the centroid of
Concurrent lines are lines
which all pass through a
the triangle. The two figures below show a triangular model which balances on the tip of a
common point. pencil placed at its centroid.

b. Angle bisector
Definition triangle angle bisector
An angle bisector of a triangle is a line segment which bisects an angle of the triangle and
which has an endpoint on the side opposite the angle.

In the figure, AN is the angle bisector which A


divides BAC into two congruent parts.
a a
We call this the bisector of angle A
because it extends from the vertex A. nA
Since AN is an angle bisector, we can write
m(BAN) = m(NAC).
B N C

We usually use the letter n to indicate the A


length of an angle bisector in a triangle.
Hence the lengths of the angle bisectors of a nA
triangle ABC from vertices A, B and C are
written nA, nB and nC, respectively. As we can nB nC
see, every triangle has three angle bisectors.
B N C

Quadrilaterals 93
Activity Paper Folding - Angle Bisectors
Follow the steps to explore angle bisectors in a triangle.
1. Cut out three different triangles.
2. Fold the three angle bisectors of each triangle as shown below.
3. What can you say about the intersection of the angle bisectors in a triangle?

Folding the angle bisector of A. AN is the angle bisector of A. BM is the angle bisector of B.

Definition incenter of a triangle


The angle bisectors in a triangle are concurrent A
and their intersection point is called the
incenter of the triangle. The incenter of a tri- L K
O
angle is the center of the inscribed circle of
The inscribed circle of a
triangle is a circle which the triangle.
is tangent to all sides of
the triangle. B N C
O is the incenter of DABC

As an exercise, try drawing a circle centered at the incenter of each of your triangles from
the previous activity. Are your circles inscribed circles?
We have seen that nA, nB and nC are the bisectors of the interior angles of a triangle ABC. We
can call these bisectors interior angle bisectors. Additionally, the lengths of the bisectors of
the exterior angles A, B and C are
written as nA, nB and nC respectively. These
K
bisectors are called the exterior angle
bisectors of the triangle. nK¢
In the figure at the right, segment KN is the
exterior angle bisector of the angle K in
N M T
KMT and its length is nK.

94 Geometry 8
Definition excenter of a triangle
The bisectors of any two exterior angles of a
B
triangle are concurrent. Their intersection is
called an excenter of the triangle.

In the figure, ABC is a triangle and the bisectors A V C

of the exterior angles A and C intersect at


T S
the point O. So O is an excenter of ABC. In O
addition, O is the center of a circle which is
tangent to side AC of the triangle and the
extensions of sides AB and BC of the triangle.
An escribed circle of a This circle is called an escribed circle of
triangle is a circle which is
tangent to one side of the ABC.
triangle and the extensions
of the other two sides. As we can see, a triangle has three excenters and three corresponding escribed circles.

EXAMPLE 85 Find all the excenters of KMN in the figure K

by construction.
N
M

Solution To find the excenters, we first construct the


bisector of each exterior angle using the
E2
method we learned in Chapter 1. Then we
K
use a straightedge to extend the bisectors
until they intersect each other. E1

The intersection points E1, E2 and E3 are the


N
excenters of KMN. M

E3

Quadrilaterals 95
c. Altitude
Definition altitude of a triangle
An altitude of a triangle is a perpendicular line segment from a vertex of the triangle to the
line containing the opposite side of the triangle.

In the figure, AH is the altitude to side A


BC because AH is perpendicular to BC.

ha

B H C

In a triangle, the length of an altitude is called a height of the triangle.

The heights from sides a, b and c of a triangle


A
ABC are usually written as ha, hb and hc,
respectively. As we can see, every triangle has
three altitudes. ha
hb hc

B H C

EXAMPLE 86 Name all the drawn altitudes of all the B


triangles in the figure.

A C
K

Solution There are eight triangles in the figure. Let us look at them one by one and name the drawn
altitudes in each.

96 Geometry 8
B B
B B

C A K
C A K K
A
K

D altitudes
altitude BK altitude AK altitudes BK and CK AK and BK
B

K
K
K C A
A C K C

D D
D D altitudes
altitude DK altitude CK altitudes CK and DK AK and DK

Activity Paper Folding - Altitudes


To fold an altitude, we fold a triangle so that a side matches up with itself and the fold
contains the vertex opposite the side.

Cut out three different triangles. Fold them carefully to construct the three altitudes of
each triangle. What can you say about how the altitudes intersect?

Quadrilaterals 97
Definition orthocenter of a triangle
The altitudes of a triangle are concurrent. Their common point is called orthocenter of the
triangle.
Since the position of the altitudes of a A
triangle depends on the type of triangle, the
position of the orthocenter relative to the
K
triangle changes. In the figure opposite, the ha
hb hc
orthocenter K is in the interior region of the
triangle. Later in this chapter we will look
B H C
at two other possible positions for the K is the orthocenter of DABC
orthocenter.

Once we know how to draw an altitude of a triangle, we can use it to find the area of the
triangle.

Definition area of a triangle


The area of a triangle is half the product of the length of a side (called the base of the
triangle) and the height of the altitude drawn to that base. We write A(ABC) to mean the
area of ABC.

For example, the area of ABC in the figure A


BC  AH a  h
is A( ABC ) = = . Area is usually
2 2
h
expressed in terms of a square unit.

B H C
a

EXAMPLE 87 Find the area of each triangle.


a. b. 2 cm
D c. K
A T

12 cm
6 cm
4 cm 5 cm

B 3 cm H 7 cm C E F M 8 cm N

BC  AH
Solution a. A( ABC ) = (Definition of the area of a triangle)
2
10  4
= (Substitute)
2
= 20 cm2 (Simplify)

98 Geometry 8
FT  DE
b. A( DEF ) = (Definition of the area of a triangle)
2
5  14
= (Substitute)
2
= 35 cm2 (Simplify)
KM  MN
c. A( KMN ) = (Definition of the area of a triangle)
2
6 8
= (Substitute)
2
= 24 cm2 (Simplify)

Definition perpendicular bisector of a triangle


In a triangle, a line that is perpendicular to a side of the triangle at its midpoint is called a
perpendicular bisector of the triangle.
A
In the figure, HN, DN and EN are the
The picture below hangs perpendicular bisectors of triangle ABC. E D
straight when the hook
lies on the perpendicular
Perpendicular bisectors in a triangle are N

bisector of the picture’s always concurrent. B C


top edge. H

Definition circumcenter of a triangle


The intersection point of the perpendicular bisectors of a triangle is called the circumcenter
of the triangle. The circumcenter of a triangle is the center of the circumscribed circle of the
triangle.

The circumscribed circle


of a triangle is a circle
which passes through all
the vertices of the triangle.

EXAMPLE 88 Find the circumcenter of each triangle by construction.


a. b. c.

Quadrilaterals 99
Solution First we construct the perpendicular bisector of each side of the triangle. Their intersection
point is the circumcenter of the triangle.
a. b. c.

Activity Perpendicular Bisector of a Triangle


There are three main faculties on
a university campus. The university
wants to build a library on the
campus so that it is the same
distance from each faculty.
1. Make a geometric model of the
problem.
2. Find the location of the library in
the picture opposite.

As an exercise, draw three more triangles on a piece of paper and construct their
circumcenters. Check that each circumcenter is the center of the inscribed circle.

Check Yourself 3
1. Name the auxiliary element shown in each triangle using a letter (n, h or V) and a vertex
or side.

a. b. c. d. e. f.
A M X K N M P J

N S H M

B C N P Y W Z L M N K L

100 Geometry 8
2. In a triangle MNP, the altitude NT of side MP and the median MK of side NP intersect at
the point R.
a. Name all the triangles in the figure formed. b. Name two altitudes of MTN.
3. In a triangle DEF, EM is the median of side DF. If DE = 11.4, MF = 4.6 and the perimeter
of DEF is 27, find the length of side EF.
4. In a triangle KLM, LN is the altitude of the side KM. We draw the angle bisectors LE and
LF of angles KLN and MLN respectively. If the angles between the angle bisectors and the
altitude are 22° and 16° respectively, find m(KLM).

A
5. In the figure, A(ABH) = A(AHC). Find x.
10
8
x
4
B H C

6. Write one word or letter in each gap to D A


make true statements about the figures. R
a. Point O is a(n) __________ . O

b. Segment ________ is a median. E P F B C


c. Point _______ is an excenter. Y
d. Segment ________ is an altitude. N
G
e. Point B is a(n) _____________. V
X Z M
f. Segment ER is a(n) __________ J

___________. S T L

g. Point _________ is a circumcenter. K

h. Line TM is a(n) __________


__________.
i. Point ________ is a centroid.

Answers
1. a. nB b. hp c. Vx d. Vl e. hn f. nL
2. a. MNK, MKP, MNT, NTP, MRT, MNR, RNK, MNP b. NT, TM
3. 6.4 4. 76° 5. 5
6. a. incenter b. ET c. K d. AB (or BC) e. orthocenter (or vertex) f. angle bisector
g. M h. perpendicular bisector i. G

Quadrilaterals 101
EXERCISES 1 .1
A. Parallelogram 3. Complete the flow chart to prove that a diagonal
of a parallelogram divides the paralellogram into
1. Find the measures of the interior angles of each
two congruent triangles. A B
parallelogram, using the information given.
3 1
Given: parallelogram 4
a. D C b. D C
ABCD with diagonal BD
4x + 10° 2
32° Prove: ABD  CDB 5 6
D C
3x – 5° Proof:
A B E A H B AB P CD Ð1 @ Ð5
c. D C a. b.
c. e. DABC @ DCDB
36°
d. f. h.
g.
A B H
Reflexive property
of congruence

4. In the figure, AFCE is a D C


parallelogram, DE = BF
2. Each figure below shows a parallelogram with one E
and points B, F, E and D
or more angle bisectors. Find the perimeter of F
are collinear. Prove that
each parallelogram, using the information given.
quadrilateral ABCD is a A B
a. C D b. A E 6 C parallelogram.

A2 E B D 8 B
5. In the figure below, quadrilaterals KLMA and
c. D 9 C d. D C
ABCD are parallelograms. Points K, A and B are
6 collinear, and points M, A and D are also collinear.
Prove each statement by using either a paragraph
B
A E 3 B A E 2 proof, a flow chart proof, or a two-column proof.
F
a. L  C D C
e. D E C f. D C
b. LM  DC
c. K and C are K A
B
supplementary
A 12 B A 3 E 2 F B L M

102 Geometry 8
6. In the figure, AC and BD C 11. ABCD is a parallelogram with AB > BC. Point E is
are diagonals of the D on side BC such that CE : EB = 3 : 1, and point F
L
quadrilateral ABCD. Points N is the intersection of DE and AC. If AF = 8 cm
P
P and N are the midpoints K and FE = 4 cm, find the sum of the lengths of DF
of sides AD and BC A B and FC.
respectively, and points L and K are the midpoints
of diagonals AC and BD respectively. Prove that
quadrilateral PKNL is a parallelogram. 12. ABCD is a parallelogram and points E and F are
the midpoints of sides BC and CD respectively. AF
and AE intersect the diagonal BD at points M and
7. In the figure, ABCD is a D
N respectively. Prove that DM = MN = NB.
C
20°
parallelogram with BH  AD
and BH = AD. H
? 13. In the figure, ABCD is a D C
If m(HCD) = 20°, find A B
parallelogram, point E is
m(ABH). O
midpoint of side AB and
F
point O is the intersection of
the diagonals AC and BD. If A E B
8. In the figure, line d and C¢
B¢ d OF = 3 cm, find
parallelogram ABCD D¢

have no common points D C FC
a. the ratio . b. the length of BD.
and d is perpendicular FE
O
to each of AA1, BB1, CC1
and DD1. If AA1 = 7 cm, A B
14. In the figure, ABCD is a D C
BB1 = 9 cm and DD1 = 3 cm, find the length of
parallelogram and points
CC1. 4
A, B and E are collinear. 7 F
Point F is the intersection
of DE and BC.
9. In the figure, ABCD is a E D C A B 6 E

parallelogram. Points B, If AD = 7 cm, BE = 6 cm and FC = 4 cm, find


F 6
K, F, E and C, D, E are K P(ABCD).
4ñ3
respectively collinear, and
A B
AC is the diagonal of the
15. In the figure, ABCD is a D 18 C
parallelogram. Given BK = 4ñ3 cm and FK = 6 cm, 
parallelogram and CF
find the length of EF.
bisects C. FE  EC, 10

AF = FE, AD = 10 cm
and DC = 18 cm are A F B
10. A parallelogram ABCD has side lengths a and b and
diagonals with lengths e and f. If a + b = 13 cm given. Find the perimeter E
and a  b = 36 cm, find the value of e2 + f 2. of the right triangle EBF.

Quadrilaterals
103
16. In the figure, ABCD is a D C 21. Calculate the length x in each figure.

parallelogram and DE and 4 E a. b.
D C D C
AE bisect D and A H 2
2 x
respectively. If EH  AD, A F B p 9 10
8 3
AH = 2 cm and DH = 4 cm, find the length of CF.
A B A B
2 x
p

c. D C |DP| = x
17. In the figure, ABCD is a D E C 3
 P
parallelogram such that
12
4
2  DE = 3  EC, EF  BC K N
A B
and AC = 40 cm. Find the
A F B
length of KN.

18. In the figure, ABCD is a D E F C



parallelogram and AF and
K b
BE are the bisectors of
22. Complete the paragraph to prove that if the
A and B respectively.
A a B diagonals of a parallelogram are congruent then
If AB = a and BC = b, show
the parallelogram is a rectangle.
that EF = 2b – a.
Given: ABCD is parallelogram D C
and DB = CA.

Prove: ABCD is a rectangle.


B. Rectangle
Proof: A B
19. One side of a rectangle measures 12 cm and its
Opposite sides of a parallelogram are congruent,
diagonal measures 13 cm. Find the perimeter of
so DA  a. . Also, AB  BA by the reflexive
this rectangle.
property of congruence. Since DB = CA (given),
DAB  CBA by b. . DAB  CBA
because c. , and DAB and CBA
are supplementary because they are
20. The length of the longer side of a rectangle is
d. angles.
twice the length of its shorter side. If the perimeter
DAB and CBA are right angles because
of the rectangle is 36 cm, find
e. . Hence CDA and DCB are also
a. the lengths of the sides of the rectangle. right angles because f. . So ABCD is
b. the length of the diagonal. a rectangle by g. .

Geometry 8
104
23. In the figure, ABCD is a D C 28. In a rectangle ABCD, D ? E 1 C
rectangle. If CD = EC and 15° point E is on side DC,
m(ADE) = 15°, find the AB = AE, AD = 5 cm and 5
measure of CEB. EC = 1 cm. Find the
? A B
A E B length of DE.

29. In a rectangle ABCD, D E ? C


24. In the figure, ABCD is a D C
30° points E and F are on
rectangle and point O is 5
O sides DC and AB
the intersection of the
? respectively.
diagonals AC and BD. A ? F 2 B
A E B If FB = 2 cm and
If AE = OB and
BC = 5 cm, find the lengths of AF and EC.
m(BDC) = 30°, find m(EOB).

30. In a rectangle ABCD, point E D 5 E C


25. ABCD is a rectangle with AB > BC. H is a point on 
is on side DC and point F is
diagonal AC, and BH is perpendicular to AC. BH 6 F
the intersection of BE and
also divides AC into two line segments with the diagonal AC. If AC  BE,
A ? B
lengths 9 cm and 16 cm. Find the perimeter of DE = 5 cm and AD = 6 cm,
ABCD. find the length of AB.

26. In the figure, point O is the 31. In a rectangle ABCD, points D C


D E C 
intersection of the diagonals E and F are on the diagonal E
?
4 F
DB. Given AE  DB, CF  DB, 3
of the rectangle ABCD. If F
DE = EC, DF = 4 cm and
O
60° AB = 4 cm and AD = 3 cm,
A 4 B
m(OBC) = 60°, find find the length of EF.
A B

a. the length of the diagonal.


b. the lengths of sides.
C. Rhombus
32. In the figure, ABCD is a D

27. In the rectangle ABCD in the rhombus with m(A) = 100°


D C E
figure, CE is the bisector of and m(BEC) = 55°. Find
C and point E is the m(DBE). A 100° 55° C
6ñ2
midpoint of side AB. If ?
CE = 6ñ2 cm, find the A E B
perimeter of the rectangle. B

Quadrilaterals
105
33. In the figure, ABCD is a 110° 38. In the figure, ABCD is a A
D
rhombus and ABE is an C rhombus and point E is on
E ?
equilateral triangle. If the diagonal AC. 9
7
m(D) = 110°, find If AE = 7 cm, EC = 3 cm
B D
m(BCE). and AD = 9 cm, find the ?
length of DE. E
A B
3

34. In the figure, ABCD is a D E C


39. In the figure, ABCD is a D
rhombus, point E is on
rhombus. Given FE  AD, E
side DC and AE is the
30°
m(DAB) = 120°, EA = 5 cm F
bisector of DEB. If 5 2ñ3
?
and EF = 2ñ3 cm, find
m(DAE) = 30°, find ?
the length of FC. A 120° C
m(ABE). A B

B
35. Find the perimeter of a rhombus whose diagonals
measure 24 cm and 32 cm.
D. Square
40. In the figure, ABCD is a D C
square with AE = BC and 25°
36. Quadrilateral ABCD in the D C m(CDE) = 25°.
4 E
figure is a rhombus and BE E
Find m(EAB).
is perpendicular to AD. If 6
AE = 6 cm and ED = 4 cm, ?
A B
find the lengths of A B

diagonals of the rhombus.

41. In the figure, ABCD is a square E


and ADE is equilateral. Find
?
m(BFD) and m(BED). F
37. In the figure, ABC is a right A D
A
?
triangle and the quadrilateral D F
BEFD is a rhombus.
If AB = 5 cm and AC = 12 cm, B E C
find the length of one side of the rhombus. B C

Geometry 8
106
42. In the figure, ABCD is a E
45. In the figure, ABCD is a B A 6 E
 ? square, AE = 6 cm and
square, points B, D and E D C
are collinear, and BD = EC. ? BD = 8ñ2 cm. Find the
8ñ2 ?
Find m(DEC) and length of DE.
m(DCE).
C D
A B

43. Complete the two-column proof to prove that in a


square, any point taken on a diagonal is equidistant
from the vertices on either side of the diagonal.
46. In the figure, ABCD is a D C
square and DE bisects ADB.
Given: ABCD is a square and D C
If AE = 2 cm, find the
point P is on diagonal DB.
perimeter of the square.
Prove: AP = CP K

A 2 E B
P
A B
Proof:
Statements Reasons
AB @ CB a.
ÐABP @ b. c.
BP @ BP reflexive property of congruence 47. In a square ABCD, point E is D C
d. @ e. SAS congruence postulate on side BC and DH is 1
AP @ CP f. perpendicular to AE. ? E
AP = CP g. ?
If BE = 3 cm and CE = 1 cm, 3
H
find the lengths of DH and
HE.
A B
44. In the following squares, P is any point. Calculate
x in each figure, using the given lengths.
a. b.
12 x
9 8
P P
x 3 10
2 48. In a square ABCD, point E is D C
c. x P d. P on the diagonal AC.
2 x 4 If AE = 2 cm and BE = 10 cm,
x ?
10
8 6
find the length of one side
of the square. 2
E 10

A B

Quadrilaterals
107
49. In the figure, ABCD is a D C E. Trapezoid
square and and point E is on
2x–2 E ? 53. In the following trapezoids, the bases are shown
the diagonal AC.
by parallel lines. Calculate x in each figure.
If DE = 2x – 2, EB = x + 4 x+4 a. D C b. C
and AB = 8ñ2 cm, find the D

length of EC. 7x–25°


A 8ñ2 B
A 2x
3x+5°
A B 70°

B
c. D

50. In the square ABCD in the D ? C C



figure, points C, F, E and
2ò13 x+35° 3x–5°
B, A, E are respectively
A B
F
collinear. Given that
?
DF = 2FA and FC = 2ò13,
E A B
find the lengths of EF and

51. In the figure, ABCD is a D C



square and EF  AB.
5
If AE = DE = EF = 5 cm, 5
F
find the perimeter of the E
square. 5
54. Each figure shows the lengths of the bases and
A B the median of a trapezoid. Calculate x in each
case.
a. 6 b. x

x 5

3
52. In the figure, ABCD is a D C 9

square and points E, A 3
c. d.
and B are collinear. F 4
Given m(EDF) = 90°, 7
x 6x
6
CF = 3 cm and BF = 6 cm, 4
2x
find the length of EF. E A B

Geometry 8
108
55. Each figure shows the median and diagonals of a 59. In the quadrilateral D 4 C
trapezoid. Find the unknown lengths in each ABCD in the figure, 6
case, using the information given. AB  PN  DC, and AP ? P N

a. 4 b. c
and DP are the bisectors
A 14 B
of A and D respectively.
x y z 4 x If DC = 4 cm, PN = 6 cm and AB = 14 cm, find
2
the length of AD.
10 a

c.
60. In each trapezoid below, point O is the intersection
x
of the diagonals and KP is the line segment passing
a
3 through this point which is parallel to the bases.
5 Find x in each figure.
y
a. 8 b. P
5
O
56. The median of a trapezoid divides the trapezoid O 3 x
K P K
into two new trapezoids. If the lengths of the
medians of the new trapezoids are 8 cm and 4
KP = x
12 cm, find the lengths of the bases of the
original trapezoid. c. 3
K P
O 2

57. In the figure, AD  BC  PN. D x


Given that AD = 12 cm, P

BC = 4 cm, AN = 2 cm and
12 C
NB = 6 cm, find the length of ?
4 61. In the trapezoid opposite, D P C
PN.
B
A 2 N
6 AB  DC, DH  AB,
O
PT  AB and points P, O
and T are collinear.
If AB = 12 cm, DC = 4 cm A H T B
58. In a trapezoid ABCD, D C
and DH = 6 cm, find the lengths of OP and OT.
AB  DC and DF and CE
bisect D and C K
?
respectively. If AD  CE ?
130°
?
and m(DFB) = 130°, A E F B 62. ABCD is a trapezoid with AB  DC.

find the measures of A, B and CKF. Prove that AC2 + DB2 = BC2 + AD2 + 2  AB  DC.
Quadrilaterals
109
63. In a trapezoid ABCD, D 6 C 67. The figure shows a trapezoid D C

AB  DC and AC and BD ABCD with AB  DC and


are diagonals. AB = 12 cm, 8 9 AD = BC. Decide whether O
f
e
BC = 9 cm, AD = 8 cm each statement is true or
and DC = 6 cm are given. false. A H B
A 12 B
If AC = e and BD = f,
a. ABCD is an isosceles trapezoid.
find the value of e2 + f 2.
b. A  B and D  C
(Hint: Use the formula from question 62.)
c. m(A) + m(D) = 180° and
m(A) + m(C) = 180°
d. AC = DB
e. DO = OC and AO = BO
64. In a trapezoid ABCD, D 3C f. OAB  OBA and DAO  CBO
AB  DC. 16 30
g. ADB  BCA, AOD  BOC and ACD  BCD
Given AB = 37 cm,
AB  DC
BC = 30 cm, A B h. HB =
2
AD = 16 cm and 37

DC = 3 cm, find the height of the trapezoid.


68. In the figure, ABCD is an D C

isosceles trapezoid with H


AB  DC. If BH is the
bisector of angle B and
65. Prove that in a trapezoid, the angle formed by the BH  AD, find the A B

bisectors of any two interior angles that share the measures of the interior
same leg is a right angle, and the intersection of angles of the trapezoid.
these bisectors lies on the median of the trapezoid.

69. Prove that if the diagonals of an isosceles



trapezoid are perpendicular to each other, then
the height of the trapezoid is equal to half the
66. In the figure, ABCD is a trapezoid D 4 C sum of the lengths of the bases.

with AB  DC. AN, BK, CK and
DN are the bisectors of A, B, 70. In the figure, ABCD is an D C
E F
C and D respectively. K N
isosceles trapezoid with 3 E
Given AB = 12 cm, DC = 4 cm, AB  DC and AC  BD. 5
BC = 10 cm and AD = 8 cm, A 12 B If DE = 3 cm and AE = 5 cm,
A H B
find the length of KN. find the length of CH.
(Hint: Use the theorem from question 65.)
Geometry 8
110
71. In an isosceles trapezoid with base lengths 13 cm 75. The diagonals of a right trapezoid are perpendicular
and 5 cm, each diagonal is perpendicular to a leg. to each other. Given that the bases measure 6 cm
Find the height of this trapezoid. and 24 cm, find the height of the trapezoid.

D C
76. In the figure, ABCD is a
right trapezoid with 8 ?

72. Find the lengths of the diagonals of an isosceles AB  DC and BD is the


A 16 B
trapezoid whose base lengths are 6 cm and 18 cm, bisector of B.
given that one leg measures 10 cm. If AD = 8 cm and AB = 16 cm, find the length of
BC.

77. In the right trapezoid C

ABCD shown opposite,


73. In the figure, ABCD is D 8 C ?
AD  BC and DE  EC. If
an isosceles trapezoid ?
E AD = 2 cm, AE = 6 cm D
with AB  DC, and AF 3 ? and BE = 3 cm, find the 2
F
and DF are bisectors of lengths of BC and DC. A 6 E 3 B
A and D respectively.
A 12 B
Given EF  AD,
EF = 3 cm, DC = 8 cm and AB = 12 cm, find the 78. In the figure, ABCD is a right D
13
length of BC. trapezoid with AD  BC, and CE
C
bisects BCD.
If AE = 2 cm, DC = 13 cm and E 10
2
BC = 10 cm, find the length of AB. A B
?

74. Find the length x in each trapezoid.


a. 3 b.
x
79. In the right trapezoid ABCD D 4 C

4
x 9 in the figure, AB  DC.
3 Given AP = PD, PC  BC, P
6 8
DC = 4 cm and AB = 12 cm, ?
c. 10 d. find A 12 B
x
x
a. the length of AD.
x 13
4 b. the length of BC.
2
5

Quadrilaterals
111
80. In the right trapezoid ABCD, D 2 C 84. In the figure, ABCD is a D
 kite with AB = AD = x
AB  DC and AE  BC. x y
and CB = CD = y.
Given AB = BC, DC = 2 cm A C
60° 120°
and AE = 4 cm, find the E m(A) = 60° and
length of AB.
4 m(C) = 120° are given. x y
Find the ratio of x to y.
B
A ? B

F. Kite
85. In the figure, ABED is a A
81. In the figure, ABCD is a D kite with AB = BE and D
kite with AB = AD and DE = 4 cm.
115° ?
CB = CD. A ? ? C
4
If 3  BE = 2  EC, find
If m(A) = 3  m(C) B E C
? the length of DC.
and m(D) = 115°, find
B
the measures of A, B
and C.

86 In the figure, ABCD is a kite A



82. In the figure, ABCD is a kite A with AB = AD and CB = CD. ?
Given m(BAD) = 90°, E
with AB = AD and CB = CD. 48° B D

If m(BAC) = 48° and B


?
D CD = ò10 cm and AC = 4 cm,
?
find the length of AB. ò10
m(ACD) = 26°, find
m(DBC) and m(BDA). C
26°
C

87. In the figure, ABCD is D


83. In the figure, ABCD is a kite A  F
a kite with AB = AD and
with AB = AD and CB = CD. 4
CB = CD. A, E, B and A ? C
Given m(B) = 105° and B 105° D D, F, C are respectively
E
AD = 4 cm, find the length of collinear, and the B
DC. ? diagonals of the kite measure 9 cm and 6 cm. If
FC = 2  DF and AE = 2  EB, find the length of
C EF.
Geometry 8
112
Mixed Problems 92. In the figure, ABCD is a F
 E
kite, AHD is an equilateral
88. In the figure, ABCD is a F
D C triangle and CEFD is a
rhombus and BEFC is a
square. D
square. 108°
45°
If m(BCD) = 40° and
If m(DCB) = 108°, find E A H 40° C
m(CDH) = 45°, find
m(AEF). A B
m( HBE).
B

89. D 4 C
6ñ3
93. In the figure, ABCD is a D C
60°  5
30°
?
rectangle. Given EH  BD, H
A B 8
AE = EH, DH = 5 cm and
In the figure, ABCD is a trapezoid with AB  DC, HB = 8 cm, find P(AEHD). A E B

BC = 6ñ3 cm, DC = 4 cm, m(A) = 60° and


m(B) = 30°. Find the length of AB.

90. In the figure, ABCD is a D C 94. In the figure, ABCD is a D 5 C



parallelogram and DE and trapezoid and ADCE is a
E E
AE are the bisectors of the 5
? kite. Given AB  DC,
4 DC = CE = 5 cm and ?
D and A respectively.
A B H AB = 12 cm, find the
Points A, B and H are
length of EB. A 12 B
collinear. If CH  AH, AE = 4 cm and BC = 5 cm,
find the length of CH.

95. In the figure, ABCD is a D C


91. Beyza and Rana are designing 
parallelogram and AK, K
a kite to look like the one at
BK, CP and DP bisect T S b
the right. Its diagonals will
A, B, C and D P
measure 44 cm and 60 cm,
respectively. AB = a A a B
and the students will use ribbon
and BC = b are given.
to connect the midpoints of
the sides. How much ribbon a. Prove that TPSK is a rectangle.
will Beyza and Rana need? b. Prove that TS = PK = a – b
Quadrilaterals
113
C. The Triangle Proportionality 16. In the figure, MNP is M
?
Theorem and Thales’ Theorem a triangle and K
2
J MN  KS, T
13. In the figure, MN  KL, JM = 5
and MK = 2. Find each ratio. KN  TS,
5
JN JN NS = 3 cm,
a. b. N 3 S 6 P
NL JL M N SP = 6 cm and
NL NL 2 KT = 2 cm.
c. d.
JN JL K L
What is the length of MK?

17. Determine whether or P


14. Find the value of x in each figure by using the
not MN  PS in the M
information given.
figure, given each set
a. A b.
K of extra information.
3 x x
E P
D R N S

8 8
6 a. PR = 18 MR = 6
SR = 24 NR = 8
B C M 12 R 3 N
b. PR = 12 MP = 8
c. X d. M
SR = 16 NR = 12
6 8 6
P Z T c. MR = 5 MP = 4
x
8 K 24
6 T RN = 6 NS =
P
x
6 5
B d. PR = 15 MR = 12
N
RN = 16 NS = 4

15. In the figure, A

AD = 8 cm, 8 18. ABCD in the figure is a D F C


DB = 6 cm, D E
parallelogram with 2
?
EF  AB and DE  BC. 6
CE = 2 cm, E
Find each ratio. B F C EB = 4 cm and
H 4
AC AC BF FC FH  AD.
a. b. c. d.
AE EC FC BC What is the length of FH? A B

Geometry 8
114
19. In the figure, DC  EF  AB. D C D. Further Applications
Find the value of x. 10 x
23. Find the length x in each figure by using
E F
Menelaus’ Therom.
12 9
A M

A B 3 3
4
4 K
E S
F 2
5 6
x T
x
6
B 6 D 4 C N P

A A
20. Write a proof of the Converse of the Triangle x
3
3 E
Proportionality Theorem in two-column form. E 18
1
K D 2
4 x

B D C B T C
(Hint: Draw a line parallel
to DE through B.)

21. A point on the hypotenuse of a right triangle



divides the hypotenuse into two segments of
lengths 12 and 16. Given that the point is
equidistant to the legs of the triangle, find the
lengths of the legs of the triangle.
24. Find the length x in each figure by using Ceva’s
Theorem.
a. A b. M
2
5 2
3 T
F K
5
E
x
22. In the triangle C 2 6 P
x
 S
ABC at the right, D E 6 4 4
B D C N
m(A) = 90°, 1

CD  DE, c. x
S
3 N
DE = 1, 9
10
AD = 3 and T M
AB = BE. A B
12
Find the length of CE. 10
P
8
(Hint: Draw the perpendicular EH  AB.)
K
Quadrilaterals
115
CHAPTER REVIEW TEST 1A
1. What is the sum of the measures of the interior 5. In the quadrilateral ABCD D
F
angles of a polygon which has 20 diagonals? in the figure, diagonals AC 6

and BD are perpendicular A C


E
A) 720° B) 900° C) 1080° D) 1800° E) 2160°
to each other, AB = 7 cm, 7 9
BC = 9 cm, AD = 6 cm
and DF = FC. Find the B

length of EF.

2. The measure of an interior angle of a regular A) ò15 cm B) ò17 cm C) ò19 cm


polygon is equal to four times the measure of an D) ò23 cm E) 2ò17 cm
exterior angle. How many sides does this polygon
have?
6. The diagonals of a quadrilateral ABCD are
A) 17 B) 15 C) 12 D) 10 E) 8 perpendicular to each other. Which shape is
formed by joining the midpoints of the sides of
this quadrilateral?

A) a square B) a trapezoid C) a kite


D) a parallelogram E) a rectangle
3. In the figure, ABCDE is a D F
H
regular polygon, points B, C ?

and F are collinear and E C 7. In the figure, ABCD and H F


D C
FE  AE. What is the BEFH are parallelograms
measure of HFC? and points A, B and E are ?
A B E
collinear. A B
A) 60° B) 54° C) 48° D) 40° E) 8
If m(DCB) = 110° and m(CBH) = 30°, what is
m(BEF)?

A) 110° B) 100° C) 90° D) 95° E) 80°

4. In a quadrilateral ABCD, D 8. In the figure, ABCD is a D 6 C

m(A) = m(C) = 90°, parallelogram with 120°


6 H
AD = 6, CD = 9, AB = x 9 HB  AD, m(D) = 120°
and BC = y. A and DC = 6 cm. Find the
If x + y = 9, what is the
C length of BH. A B
x
y
value of x – y? B A) 3 cm B) 4 cm C) 3ñ3 cm
A) 5 B) 6 C) 7 D) 8 E) 9 D) 4ñ3 cm E) 6 cm

116 Geometry 8
9. In the polygon ABCDE C 13. In the figure, ABCD is a D C
at the right, A, B trapezoid with AB  DC. ?
45° E
10
and E are right angles 6 EF  BC, EF = 4 cm,
E D 6 4
and m(C) = 45°. BC = 10 cm and EA = 6 cm
2 A F B
If AE = 2, 10
are given. What is the
A B
length of ED?
AB = 10 and
BC = 6, what is the perimeter of ABCDE? A) 9 cm B) 12 cm C) 15 cm
D) 16 cm E) 18 cm
A) 24 + 4ñ2 B) 28 + 4ñ2 C) 28ñ2
D) 30 + 2ñ2 E) 4ñ2 + 32
14. In the figure, ABCD is a D x+1 C
right trapezoid with
4x – 2
AB  DC. 3x – 1
10. In the figure, ABCD is a D C AB = 3x + 1,
rectangle. Given AD = CE, 66° BC = 4x – 2, A 3x + 1 B
DE = DC and m(ADE) = 66°,
AD = 3x – 1 and DC = x + 1 are given. What is the
find m(EBA). E
perimeter of this trapezoid?
A B

A) 5° B) 6° C) 9° D) 10° E) 12° A) 16 B) 20 C) 27 D) 30 E) 32

15. In the figure, ABCD is an D ? C


isosceles trapezoid with
11. In the figure, ABCD is a D C 15 15

square and AED is an AB  DC, AD  BD,


isosceles right triangle. AB = 25 cm and A 25 B
E
If AB = 4 cm, what is AD = BC = 15 cm. What is the length of DC?
the length of EC?
A 4 B A) 3 cm B) 5 cm C) 6 cm D) 7 cm E) 10 cm
A) 5ò15 cm B) 12 cm C) 10 cm

D) 2ò10 cm E) 3ñ8 cm 16. In the figure, ABCD is a D


kite and AB = AD. 30

Given that AB  BC, 16 K ?


A C
AE = EB, AK = 16 and
12. The diagonals of a rhombus measure 20 cm and E
AD = 30, find the length
48 cm respectively. What is its perimeter?
of KC. B

A) 52 cm B) 78 cm C) 104 cm 27 43
A) B) C) 18 D) 21 E) 32
D) 125 cm E) 208 cm 2 2

Chapter Review Test 1A 117


CHAPTER REVIEW TEST 1B
1. Three interior angles of a polygon measure 80°, 5. In the figure, ABCD is a D C
115° and 135°, and all the other interior angles square with m(CEA) = 120° 3

measure 165°. How many sides does this polygon and CE = 4ñ3 cm. Find the
have? perimeter of the square. E
120°

A) 9 B) 10 C) 12 D) 13 E) 15
A B
A) 18 cm B) 24 cm C) 30 cm
D) 36 cm E) 48 cm
2. In the figure, ABCDE is a F
D 85°
pentagon.
Given that m(A) = 110°, E
130° ? C 6. The ratio of the lengths of two consecutive sides
m(E) = 130°, of a rectangle is 3 : 4. If the perimeter of the
110° 65°
rectangle is 42 cm, how long is its diagonal?
m(CBH) = 65° and A B H
m(FDC) = 85°, find m(BCD). A) 9 cm B) 12 cm C) 13 cm
D) 14 cm E) 15 cm
A) 70° B) 75° C) 80° D) 85° E) 90°

7. In the figure, ABCD is an D C

3. In the figure, ABCDEF... is a B isosceles trapezoid with


A 5 5
regular polygon. C AB  DC, and diagonal AC
If m(CEF) = 135°, what is is the bisector of A.
D A 11 B
the measure of one interior If AB = 11 cm and
angle of the polygon? 135°
AD = BC = 5 cm, what is the length of AC?
E
F
A) 6 cm B) 8 cm C) 3ñ5 cm
A) 150° B) 145° C) 140° D) 135° E) 130° D) 4ñ5 cm E) 6ñ5 cm

8. In the figure, ABCD is a D 6 C


4. In the figure, ABCD is a D C
right trapezoid with
parallelogram and DE and 125° H
AB  DC, DH  AC, 8
CF are the bisectors of E
F AB = 15, BC = 8 and
D and C respectively.
A B DC = 6. How long is DH? A 15 B
If m(DEF) = 125°, what
is m(EFC)? 48 36 32 24 20
A) B) C) D) E)
17 17 17 17 17
A) 135° B) 140° C) 145° D) 150° E) 155°

118 Geometry 8
9. In the figure, ABCD is a E 13. In the figure, ABCD is a E

parallelogram and points A, D square and points A, C and ?

and E are collinear. CE and AF ? E are collinear. If AC = CE D C


bisect C and A respectively. F and AB = 6 cm, how long is
D C
If DF = FC and BC = 6 cm, ED?
6
what is the length of ED? A 6 B

A B A) 12 cm B) 10 cm C) 6ñ5 cm
A) 14 cm B) 12 cm C) 8 cm
D) 4ñ3 cm E) 6 cm D) 4ñ5 cm E) 5ñ2 cm

10. In the figure, ABCD is a D C 14. In the figure, ABCD is a kite D


square and AE is the bisector with AB = AD and DC = CB. C
2
?
of CAB. If EB = 2 cm,
If HB  AD, AD  DC, H
what is the length of CE? E ?
AH = 3 and HD = 2, how 3
2 A B
long is BC?
A B 5 7
A) 5 B) C) 32 D) 12 E)
A) ñ2 – 1 cm B) ñ2 cm C) ñ2 + 1 cm 2 2
D) 2ñ2 cm E) 2ñ2 – 1 cm

11. In the figure, ABCD is a D C 15. In the figure, ABCD is a D C

rectangle. rhombus, point O is the


12
midpoint of the diagonal BD 4
Given that DE  EB, O
A F B and OH  BC. If AB = 10 cm H
DF = FB, AD = 12 cm and 9
and OH = 4 cm, what is the A 10 B
FE = 9 cm, find the length E
length of BD?
of DC.
A) 2ñ5 cm B) 5 cm C) 6 cm
A) 15 cm B) 22 cm C) 24 cm
D) 8 cm E) 4ñ5 cm
D) 25 cm E) 28 cm

12. In the figure, ABCD is a D H 8 C 16. In the figure, ABCD is a D 6 C


parallelogram. trapezoid with AB  DC.
12 8 ?
Given that EF  AD, Given that AE = ED, E
E EC  CB, DC = 6 cm,
BH  DC, AE = 5, 5
?
A 14 B
DE = 12 and HC = 8, A F B EC = 8 cm and
find the length of EF. AB = 14 cm, find the length of BC.

75 25 56 A) 9 cm B) 12 cm C) 15 cm
A) B) C) 16 D) E) 28
8 3 5 D) 16 cm E) 18 cm

Chapter Review Test 1A 119


CHAPTER REVIEW TEST 1C
1. The difference between the measures of an 5. In the figure, ABCD is a D C

interior angle and an exterior angle of a regular parallelogram and DE and ?


polygon is 132°. What is the measure of one CE are the bisectors of D 10 12

interior angle of this polygon? and C respectively.


A E B
If AD = 10 and DE = 12,
A) 108° B) 120° C) 140° D) 144° E) 156°
how long is EC?

A) 9 B) 12 C) 16 D) 18 E) 20
2. In the figure, ABCDE is a E
D
regular pentagon, DB is a
diagonal and AF is the
?
F 6. In the figure, ABCD is a right D 5 C
bisector of A. What is A
trapezoid with AB  DC.
m(AFB)? C
If AE = DC = 5, AB = DE and
B 13ñ2
BC = 13ñ2, what is the length
E
A) 54° B) 56° C) 60° D) 72° E) 76° of AB?
5

A B
A) 18 B) 15 C) 13 D) 12 E) 9
3. In polygon ABCDE, EK D
and BK are the bisectors 70°
of E and B respectively.
If m(A) = 130°, E
K 120° C 7. In the figure, ABCD is a D C
? ?
m(C) = 120° and square. CE = BF, AF = FE
130°
m(D) = 70°, what is and m(EAB) = 20° are
A B E
the measure of EKB? given.
F
What is the measure of
A) 125° B) 120° C) 110° D) 105° E) 100° 20°
DCF?
A B

A) 20° B) 30° C) 45° D) 50° E) 55°


4. In the figure, ABCD is a D E C
rhombus, AC is its diagonal ?

and AE is the bisector of 8. The radius of the circumscribed circle of a square


124°
DAC. is 5. What is the radius of the inscribed circle of
A B
If m(B) = 124°, what is this square?
m(AEC)?
5 2
A) 2ñ5 B) C) 5ñ2 D) 10 E) 12
A) 124° B) 130° C) 138° D) 143° E) 146° 2

120 Geometry 8
9. In the figure, ABCD is a D E C 13. In the figure, ABCD is a D F 2 C
rectangle and EB bisects rectangle.
AEC. 6 ? Given DE  EF,
A B
If DC = 10 and AD = 6, DE = 2  EF, AE = 8 cm 8 E
A B
what is the length of BE? and FC = 2 cm, find the length of BC.
3 10
A) ò10 B) 2ò10 C) 3ò10 D) E) 7 A) 4 cm B) 5 cm C) 6 cm D) 8 cm E) 10 cm
2

14. In a quadrilateral ABCD, D

10. In the figure, ABCD is a ? AB  AD and 60°


D C 10
trapezoid and ADCE is a m(C) = m(D) = 60°. 8
kite. Given that AB  DC, E If AD = 8 cm and 60° C
DC = CE, EB = 7 and ?
7
DC = 10 cm, what is the A B
AB = 11, find the length length of BC?
of CD. A 11 B
A) 2ñ3 cm B) 3ñ2 cm C) 6 cm
A) 5 B) ñ5 C) 4 D) 3 E) ñ2 D) 5 cm E) 3ñ3 cm

11. In the figure, ABCD is a D G C 15. In the figure, ABCD is an D C


square and DEFG is a isosceles trapezoid with
?
rectangle. If DE = 2  EF and AB  DC.
AC =12ñ2, what is the If AD = DC = CB, A 10 B
E F
perimeter of the rectangle
m(ACB) = 90° and
DEFG?
A B AB = 10, what is the length of AC?
A) 12 B) 18 C) 20 D) 24 E) 32
A) 2ñ3 B) 3ñ2 C) 4ñ2 D) 5ñ3 E) 6ñ2

12. In the figure, ABCD is a D 3 C


16. In the figure, ABCD is a D C
trapezoid with AB  DC. 60°
parallelogram. F
?
If m(C) = 60°, 4
If AE = 2  EB, AF = 3  FD K
AB = 5, BC = 4 A B and KB = 4, what is the
5
A E B
and DC = 3, what is the length of AD? length of FK?

A) 5 B) 2ñ7 C) 6 D) 2ò10 E) 7 A) 9 B) 11 C) 15 D) 16 E) 18

Chapter Review Test 1A 121


a. The Pythagorean Theorem
The Pythagorean Theorem is one of the most famous theorems in Euclidean geometry, and
almost everyone with a high school education can remember it.

Theorem Pythagorean Theorem

In a right triangle ABC with m(C) = 90°,


A
the square of the length of the hypotenuse
is equal to the sum of the squares of the
lengths of the legs, i.e. b c

c2 = b2 + a2.

C a B

Proof There are many proofs of the Pythagorean


A
Theorem. The proof we will give here uses
the dissection of a square. It proves the
Pythagorean Theorem for the right triangle c
b
ABC shown opposite.

C a B

Imagine that a large square with side length b a


a + b is dissected into four congruent right
a
triangles and a smaller square, as shown in c c b
the figure. The legs of the triangles are a and
b, and their hypotenuse is c. So the smaller b c c
a
square has side length c.
a b

124 Geometry 8
We can now write two expressions for the area S of the larger square:
 ab 2 2
S= 4  + c and S = (a + b) .
 2 
Since these expressions are equal, we can write

 ab 
4 2
 + c = ( a + b)
2

 2 
2ab + c2 = a2 + 2ab + b 2
c2 = a2 + b 2 .
This concludes the proof of the Pythagorean Theorem.

EXAMPLE 1 In the figure, ST  SQ. Find x and y. S

13
6
x
Solution First we will use the Pythagorean Theorem
in SKT to find x, then we can use it in y
T 12 K Q
SKQ to find y.

 SK 2 + KT 2 = ST2 (Pythagorean Theorem in SKT)


2 2 2
x + 12 = 13 (Substitute)
2
x + 144 = 169
x2 = 25 (Simplify)
x = –5 is not an answer x=5 (Positive length)
because the length of a
segment cannot be negative.  SK2 + KQ2 = SQ2 (Pythagorean Theorem in SKQ)
So the answer is x = 5. 2 2 2
From now on we will 5 +y =6 (Substitute)
always consider only
y2 =36 – 25 (Simplify)
positive values for lengths.
y = ò11

Pythagorean Theorem 125


EXAMPLE 2 In the figure,
K

PT = TS = KS,
PM = 4 cm and KM = 3 cm. Find ST. S

P T M
Solution Let PT = TS = KS = x.
So SM = KM – KS = 3 – x and TM = PM – PT = 4 – x.
In TMS, TS2 = TM2+ MS2 (Pythagorean Theorem)
Quadratic formula 2 2 2
The roots x1 and x2 of
x = (4 – x) + (3 – x) (Substitute)
the quadratic equation 2 2 2
x = 16 – 8x + x + 9 – 6x + x (Simplify)
ax2 + bx + c = 0 are
2
x – 14x + 25 = 0
–b b2 – 4ac
x1,2 = .
2a x1, 2 = (7 ò24) cm (Quadratic formula)
Since 7 + ò24 is greater than 3 and 4 which are the lengths of the sides, the answer is
x = |ST| = 7 – ò24 cm.

EXAMPLE 3 In the figure, K

m + k = 3  n.
Given A(KMN) = 30 cm2, n
m
find the value of n.

M k N

Solution  m + k = 3  n (1) (Given)


 A(KMN) = 30 cm 2
(Given)
km
= 30 (Definition of the area of a triangle)
2
k  m = 60 (2)
 In KMN, n2 = k2 + m2 (Pythagorean Theorem)
2 2
n = (k + m) – 2km (Binomial expansion: (k+ m)2 = k2 + 2km + m2)
n2 = (3n)2 – 2  60 (Substitute (1) and (2))
2
8n = 120 (Simplify)
2
n = 15
n = ò15 cm.

Theorem Converse of the Pythagorean Theorem

If the square of one side of a triangle equals the sum of the squares of the other two sides,
then the angle opposite this side is a right angle.

126 Geometry 8
Proof We will give a proof by contradiction. A A

Suppose the triangle is not a right triangle,


and label the vertices A, B and C. Then there c
b c b
are two possibilities for the measure of angle
C: either it is less than 90° (figure 1), or it is
C a B C a B
greater than 90° (figure 2). figure G figure ?
Let us draw a segment DC  CB such that
DC = AC.
D D
By the Pythagorean Theorem in BCD, A
A
BD2 = a2 + b2 = c2, and so BD = c. c c
c b
So ACD is isosceles (since DC = AC) and b b c
b
ABD is also isosceles (AB = BD = c). As a
result, CDA  CAD and BDA  DAB. C a B C B
figure k figure J
However, in figure 3 we have
m(BDA) < m(CDA) and m(CAD) < m(DAB), which gives m(BDA) < m(DAB) if
CDA and CAD are congruent. This is a contradiction of BDA  DAB. Also,
in figure 4 we have m(DAB) < m(CAD) and m(CDA) < m(BDA), which gives
m(DAB) < m(BDA) if CAD and CDA are congruent. This is also a contradiction.
So our original assumption must be wrong, and so ABC is a right triangle.

EXAMPLE 4 In the triangle ABC opposite, K  AC and


H
B
AN is the interior angle bisector of A.
AB = 16 cm, AN = 13 cm, AK = 12 cm and 16
N
NK = 5 cm are given. Find the area of
ABN. 13 5

A K C
Solution Let us draw an altitude NH from the vertex 12
N to the side AB.
NH  AB
To find the area of ABN we need to find NH, because A( ABN ) = and AB is given
2
as 16 cm.
132 = 122 + 52, so m(NKA) = 90°. (Converse of the Pythagorean Theorem)
Also, NH = NK = 5 cm. (Angle Bisector Theorem)
NH  AB 5  16
So A( ABN ) = = = 40 cm 2 . (Substitution)
2 2

Pythagorean Theorem 127


5
A
EXAMPLE Find the length x in the figure. 2
x N

T
P 2x
4

B 4 K 6 C

Solution AT2 + BK2 + CN2 = AN2 + BT2 + CK2 (Carnot’s Theorem)


x2 + 42 + (2x)2 = 22 + 42 + 62 (Substitute)
5x2 = 40 (Simplify)
x2 = 8
x = 2ñ2

Check Yourself
1. Find the length x in each figure.
a. D b. A c. P
A Pythagorean triple is a
set of three integers a, b x x 17
and c which satisfy the 8 7 10
Pythagorean Theorem.
The smallest and E 17 F B 24 C M S x N
6
best-known Pythagorean
triple is (a, b, c) = (3, 4, 5). C
K
d. e. f. M
8
7
6 N x
E 15
15 x
L 5
9
x
M A B N ò19 K 12 J

2. In the figure, TN = NK, ST = 12 cm S

and SN = ò69 cm. Find the length of TK.


12
ò69

T N K

3. In a right triangle ABC, m(A) = 90°, AB = x, AC = x – 7 and BC = x + 1. Find AC.

Answers
1. a. 15 b. 25 c. 9 d. 5ñ3 e. 20 f. 10 2. 10 cm 3. 5 cm

128 Geometry 8
Properties 7
1. The length of the median to the hypotenuse of a right triangle is equal to half of the length
of the hypotenuse.
2. a. In any isosceles right triangle, the length of the hypotenuse is ñ2 times the length of
a leg. (This property is also called the 45°-45°-90° Triangle Theorem.)
b. In any right triangle, if the hypotenuse is ñ2 times any of the legs then the triangle is
a 45°-45°-90° triangle. (This property is also called the Converse of the 45°-45°-90°
Triangle Theorem).
3. In any 30°-60°-90° right triangle,
a. the length of the hypotenuse is twice the length of the leg opposite the 30° angle.
b. the length of the leg opposite the 60° angle is ñ3 times the length of the leg opposite
the 30° angle. (These properties are also called the 30°-60°-90° Triangle Theorem.)
4. In any right triangle,
a. if one of the legs is half the length of the hypotenuse then the angle opposite this leg
is 30°.
b. if one of the legs is ñ3 times the length of the other leg then the angle opposite this
first leg is 60°. (These properties are also called the Converse of the 30°-60°-90°
Triangle Theorem.) A
5. The center of the circumscribed circle of
r
any right triangle is the midpoint of the
B C
hypotenuse of the triangle. r O r

EXAMPLE 6 In the figure, C


m(BAC) = 90°, 60°
m(C) = 60° and D
6x – 1
BD = DC. 2x + 3
Find BC if AD = 2x + 3 and AC = 6x – 1.
A B

Pythagorean Theorem 129


Solution  Since AD is a median and the length of the median to the hypotenuse of a right triangle
1
is equal to half the length of the hypotenuse, AD =  BC.
2
 By the Triangle Angle-Sum Theorem in ABC, m(B) = 30°.
1
 By the 30°-60°-90° Triangle Theorem, AC =  BC because m(B) = 30° and BC is the
2
hypotenuse.
 So by the transitive property of equality, AC = AD, i.e. 6x – 1 = 2x + 3 and so x = 1.
 Finally, BC = 2  AC = 2  AD = 2  (2x + 3) = 10.

EXAMPLE 7 In the figure at the right, find m(ADC) if A


m(BAC) = 90°,
m(BAD) = 2x, 2x

m(ACB) = 3x and
3x
BD = DC.
B D C

1
Solution  BC.
Since AD is a median, by Property 7.1 we have AD =
2
So AD = BD = DC. Hence DCA and BDA are isosceles triangles.
Since DCA is isosceles, m(DAC) = m(ACD) = 3x.
Additionally, m(BAC) = m(BAD) + m(DAC) by the Angle Addition Postulate.
So 2x + 3x = 90° and x = 18°.
By the Triangle Angle-Sum Theorem in DCA, m(ADC) + 3x + 3x = 180°.
So m(ADC) = 180° – (6  18)°, i.e. m(ADC) = 72°.

EXAMPLE 8 One of the acute angles in a right triangle measures 16°. Find the angle between the
bisector of the right angle and the median drawn from the same vertex.

130 Geometry 8
Solution Let us draw an appropriate figure. We need A
to find m(NAT).
45°
According to the figure,
 AN is the angle bisector, AT is the 16°
B N T C
median, and m(BAC) = 90°.
 m(ACB) = 16° by Property 5.3.
 Since AT is median to hypotenuse, AT = CT = BT.
Property 5.3:
In any triangle ABC, if  So ATC is isosceles.
m(B) > m(C) or
m(B) < m(C) then  Therefore, by the Isosceles Triangle Theorem, m(TAC) = m(ACT) = 16°.
ha < na < Va.
 Since AN is an angle bisector and m(BAC) = 90°, m(NAC) = 45°.
 So m(NAT) = m(NAC) – m(TAC)= 45° – 16° = 29°.

EXAMPLE 9 In the figure, AB  AC and AH  BC. A


Given m(C) = 30° and BH = 2 cm, find the
length of HC.

30°
B 2 H ? C

Solution In ABC, since m(C) = 30°,


A
m(B) = 60°.
In ABH, since m(B) = 60°, 60°
4 30°
m(BAH) = 30°.
60° 30°
In ABH, by Property 7.3,
B 2 H C
AB = 2  BH = 2  2 = 4 cm.
8
This set square is in the
form of a 30°-60°-90° In ABC, again by Property 7.3,
triangle.
BC = 2  AB = 2  4 = 8 cm.
So HC = BC – BH = 8 – 2 = 6 cm.

Pythagorean Theorem 131


EXAMPLE 10 Find the value of x in the figure. A

60°
10
x

Solution Let us draw an altitude from C to AB. 45°


 In BHC, B 6ñ2 C

BC = ñ2  BH (45°-45°-90° Triangle
Theorem)
A
6ñ2 = ñ2  BH (Substitute)
BH = 6. (Simplify) H 60°
This set square is in the
form of 45°-45°-90° right
 AB = AH + HB (Segment Addition
triangle.
Postulate) 45°
10 = AH + 6 (Substitute) B 6ñ2 C
AH = 4. (Simplify)

 In AHC,
AC = 2  AH (30°-60°-90° Triangle Theorem)
AC = 2  4 (Substitute)
AC = x = 8. (Simplify)

Objectives
1. In an isosceles right triangle, the sum of the lengths of the hypotenuse and the altitude
drawn to the hypotenuse is 27.3. Find the length of the hypotenuse.

2. In the figure, ABC is a right triangle with C


m(ABC) = 90° and CF = FE, and CE is the
D
angle bisector of C. If m(ADB) = 102°, find
102° F
the measure of CAB.

A E B

3. One of the acute angles in a right triangle measures 48°. Find the angle between the
median and the altitude which are drawn from the vertex at the right angle.
4. In a triangle ABC, m(B) = 135°, AC = 17 cm and BC = 8ñ2 cm. Find the length of AB.
5. In a right triangle, the sum of the lengths of the hypotenuse and the shorter leg is 2.4.
Find the length of the hypotenuse if the biggest acute angle measures 60°.

132 Geometry 8
6. In the figure, A
m(C) = 60°,
D
HC = 4 cm and x
DH = 2ñ3 cm. Find the length AC = x. 2ñ3
60°
B H 4 C

7. ABC in the figure is an equilateral triangle with A


x
DH  BC, D
BH = 5 cm and
HC = 3 cm.
Find the length AD = x.

B 5 H 3 C

8. The distance from a point to a line k is 10 cm. Two segments non-perpendicular to k are
drawn from this point. Their lengths have the ratio 2 : 3. Find the length of the longer
segment if the shorter segment makes a 30° angle with k.
9. CAB is a right triangle with m(A) = 90° and m(C) = 60°, and D is the midpoint of
hypotenuse. Find the length of the hypotenuse if AD = 3x + 1 and AC = 5x – 3.
10.The hypotenuse of an isosceles right triangle measures 18 cm. Find the distance from the
vertex at the right angle to the hypotenuse.
Answers
1. 18.2 2. 22° 3. 6° 4. 7 cm 5. 1.6 6. 5 cm 7. 2 cm 8. 30 cm 9. 14 10. 9 cm

Pythagorean Theorem 133


The word trigonometry is derived from the Greek words trigon (which means ‘triangle’) and
metry (which means ‘measurement’). So trigonometry is the study of triangle measurement.
In this chapter we will study trigonometry for right triangles.
Remember that a right triangle is a triangle with one 90° angle and two acute angles. Let us
begin by looking at the basics of trigonometry in a right triangle.

A. TRIGONOMETRIC RATIOS OF ACUTE ANGLES


A trigonometric ratio is the ratio of the lengths of any two sides of a right
triangle. We will learn three basic trigonometric ratios: sine, cosine, and tangent. We abbre-
viate them as sin, cos, and tan respectively.
Let DABC be a right triangle. Then the basic trigonometric ratios are defined as
follows:

A
length of the side opposite A
sinA = (hypotenuse)
length of hypotenuse
c
b (side opposite ÐB)
length of the side adjacent to A
cos A = (side adjacent to ÐA)
length of hypotenuse B
a C
length of the side opposite A
tanA = (side opposite ÐA)
length of the side adjacent to A

Therefore, in the right triangle above,


a b a b a b
sinA= , cos A= , tan A= , and sin B= , cos B= , tanB= .
c c b c c a

Note
We sometimes use tg
as the abbreviation of 1. Draw at least four non-congruent right triangles containing an angle of 30°.
tangent. 2. Make a table with six columns and as many rows as the number of triangles.
3. Measure the length of each side of each triangle in millimeters. Write the lengths in the
first three columns of the table.
4. Use a calculator to find the following values for each triangle:
length of the side opposite 30°
a.
length of hypotenuse .

134 Geometry 8
length of the side adjacent to 30°
b.
length of hypotenuse
.

length of the side opposite 30°


c.
length of the side adjacent to 30°
.
Write the results in the last three columns of your table.
5. What can you say about the numbers in the last three columns of your table?
6. Repeat steps 1-5 for triangles containing an angle of 53°.

EXAMPLE 11 Find the trigonometric ratios in the given tri- A


angle.
a
5
a. sin a 3

b. cos a b
B C
4
c. tan a

a
Solution opposite side 4 5
3
a. sin  = =
hypotenuse 5 b
B C
4

a
adjacent side 3 5
b. cos  = = 3
hypotenuse 5
b
B C
4

a
opposite side 4 5
c. tan  = = 3
adjacent side 3
b
B C
4

Pythagorean Theorem 135


EXAMPLE 12 Find the trigonometric ratios in the triangle. A

a. sin a b. cos a c. tan a a


5
3
b
B C
4

Solution In order to find all the ratios we need to find |BC|. A

Remember the Pythagorean Theorem:


13
5
|AC|2 + |BC|2 = |AB|2
a
52 + |BC|2 = 132 B
12 C

|BC| = 169 – 25 
2

 a. 5
So |BC| = 12 and  sin  = .
13
|BC|2 = 144 
 b. 12
|BC| = 12  cos  = .
13

c. 5
tan  = .
12

B. RECIPROCAL TRIGONOMETRIC RATIOS


Note We have looked at the basic trigonometric ratios sine, cosine, and tangent. Now we can
We sometimes use define three new trigonometric ratios. They are cosecant, secant, and cotangent, which we
ctg as the abbrevia- abbreviate as cosec, sec, and cotan respectively. They are defined as follows:
tion of cotangent.

length of hypotenuse c 1 A
cosec A = = =
length of side opposit e A a sinA
c
b
length of hypotenuse c 1
sec A = = =
length of side adjacent t o A b cos A B C
a
length of side adjacent to A b 1
cot A = = =
length of side opposi te A a tanA

136 Geometry 8
EXAMPLE 13 Write the ratios for the triangle in the figure.
A
a. cot a b. sec a c. cosec a
10
6
a
B C
8

Solution a. 1 1 8 8
cot  = = = 1 =
tan  6 6 6
8

b. 1 1 10 10
sec  = = = 1 =
cos  8 8 8
10

c. 1 1 10 10
cosec  = = = 1 =
sin  6 6 6
10

C. TRIGONOMETRIC RATIOS OF COMPLEMENTARY


ANGLES
Since the sum of the acute angles in a right triangle is 90°, these angles are
complementary.
a+ + 90° = 180° (sum of the angles in a triangle)
A a+ = 180° – 90°

a
a+ = 90°
c
b
So a = 90° – or
b
B C = 90° – a.
a
In the right triangle opposite,

a b a
sin  = = cos , cos  = = sin , tan  = = cot , and
c c b
b c c
cot  = = tan , sec  = = cosec , cosec  = = sec 
a b a

Pythagorean Theorem 137


Therefore,
sin a = cos b = cos (90° – a),
cos a = sin b = sin (90° – a),
tan a = cot b = cot (90° – a),
cot a = tan b = tan (90° – a),

For example,
sin 53° = cos (90° – 53°) tan 17° = cot (90° – 17°) cos 29° = sin (90° – 29°)
= cos 37° = cot 73° = cot 61°.

EXAMPLE 14 Find sin b if


A
4
cos a = and a + b = 90°. 10
5 6
a
B C
8

Solution Since a + = 90°, = 90° – a.


4
We can write sin = sin (90° – a) = cos a =
5

D. BASIC TRIGONOMETRIC IDENTITIES


Look at the right triangle in the figure.
A
a c a
b
In the triangle, sinA = and sin C = . So tan A = .
c b b c

1. If we divide the top and bottom of the fraction for the tangent by b we get
B a C
a
b sinA a c
tanA =  (since sin A and 
 cos A ).
c cos A b b
b

138 Geometry 8
Property

sin 
tan  = ( cos   0 )
cos 

2. If we apply the definition of cotangent we get

1 1 cos A
cot A =   .
tanA sin A sin A
cos A

Property
cos 
cot  = ( sin   0 )
sin 

cot
  

3. tan a × cot a = tan a × 1 =1
tan  or
tan a × cot a = sin  cos  = 1.

cos  sin 
 
tan  cot 

Property
tan a × cot a = 1

4. Look at the triangle on the left.

We can write a b
 sin  and
 cos  .
c c
A

a a b a 2 b2 a 2 b 2 .
c Now sin2 a + cos2 a = ( )2 ( )2  2 2 2 
b c c cc c
 c2
b By the Pythagorean Theorem, we know that a2 + b2 = c2.
B a C

(a 2  b2 ) c2
Therefore we have sin2 a + cos2 a =   1.
c2 c2

Pythagorean Theorem 139


Conclusion
sin2 + cos2 = 1, cos2 = 1 – sin2 , and sin2 = 1 – cos2 .
We can use the identities we have found to simplify trigonometric
expressions.

EXAMPLE 15 Simplify 1 + tan2 x.

Solution sin x
We know tan x  .
cos x
1
 
2
sin x 2 1 sin x cos x + sin 2 x
2
1 1 2
So 1+( ) = + = = =( ) = sec 2 x.
cos x 1 cos x
2
cos x2 2
cos x cos x
(cos2x)

EXAMPLE 16 Simplify sin x × cot x.

Solution cos x
sin x  cos x

sin x

cot x

EXAMPLE 17 Simplify cos x + tan x × sin x.

Solution 1

sin x cos x sin x cos x sin x cos x sin 2 x
2 2 2 2
cos x  sin x     
cos x
 1 cos x cos x cos x cos x
(cosx)
tan x 1
  sec x
cos x

140 Geometry 8
EXAMPLE 18 Simplify
(cot x – tan x) × (sin x × cos x).

Solution cos x sin x cos 2 x – sin 2 x


( – )  sin
x  cos x x  cos x cos 2 x – sin 2 x
 sin
sin x cos x sin x  cos x
 
cot x tan x
= (1 – sin2 x) – sin2 x

= 1 – 2sin2 x

or

= cos2 x – (1 – cos2 x)

sin2 x
= cos x – 1 + cos2 x
2

= 2cos2 x – 1

EXAMPLE 19 Simplify tan   cot 


.
cosec sec 

Solution  1

tan  cot 
  2 2
sin  cos  sin  cos  sin   cos  1
+  sin   cos  sin   cos 
cos  sin  cos  sin 
= = = =1
1 1 1 1 1

sin  cos  sin   cos  sin   cos  sin   cos 
 
cosec  sec 

Pythagorean Theorem 141


EXAMPLE 20 Simplify tan   cot 
.
cosec sec 

Solution  1

tan  cot 
  2 2
sin  cos  sin  cos  sin   cos  1
+  sin   cos  sin   cos 
cos  sin  cos  sin 
= = = =1
1 1 1 1 1

sin  cos  sin   cos  sin   cos  sin   cos 
 
cosec  sec 

EXAMPLE 21 Simplify sin3 x + cos2 x × sin x.

Solution sin x  sin2 x + cos2 x  sin x = sin x (sin2 x + cos2 x) = sin x


   
sin3 x 1

EXAMPLE 22 Simplify
cot x 1 .
cos x    tan x
sec x 1– sin2 x

Solution cot x

cos x
sin x 1 sin x cos x 1 sin x
cos x    cos x   cos x   1
1 1– sin 2
x cos x sin x cos 2
x cos x
    
cos x cos 2 x tan x

sec x

142 Geometry 8
EXAMPLE 23 Simplify tan x + cot x.

Solution  1

2 2
sin x cos x sin x cos x sin x cos x sin x cos 2 x
2
     
cos x sin x cos x sin x sin x  cos x sin x cos x sin x cos x
  (sin x ) (cos x )
tan x cot x

1 1 1
   cosec x  sec x
sin x  cos x 
sin x 
cos x
cosec x sec x

EXAMPLE 24 Verify that tan x  sin x.


sec x

Solution tan x

sin x
cos x sin x cos x
   sin x
1 cos x 1
cos x

sec x

EXAMPLE 25 Verify that

(1 – sin x)(1 + sin x) = 1


.
sec 2 x

Solution 1 1
(1 – sin x)(1 + sin x) = 12 – sin2 x = 1 – sin2 x = cos2 x = =
1 1 2
( )
1 1 cos2 x cosx
 (a
 , a  0)
2
sec x 1
a

Pythagorean Theorem 143


EXAMPLE 26 Simplify
(1 – sin2 18°)× tan 35 sec 12° .
 sin 78°
sin 72° cot 55°
2

Solution 1
cos12

 
2 tan 35° sec 12° tan 35° 1 1
(1 sin 18° ) 2  sin 78°  cos 2 18°    cos 12°
 sin 72° cot 55  cos 2 18° cos 12° tan35°
cos 2 18 ° 
     cos12°
cos2 18 ° tan 35°
1

E. FINDING A TRIGONOMETRIC RATIO FROM A GIVEN


RATIO
Sometimes we are given one trigonometric ratio and we need to find another trigonometric
ratio in the same triangle. Look at the steps we can use for
problems like this.

Property
1. Draw a right triangle and assign the angle in question to any one of the acute angles.
2. Use the given trigonometric ratio to write the lengths of the sides of the
triangle.
3. Use the Pythagorean Theorem to find the length of the missing side.
4. Write the desired ratio by using the side lengths of the triangle.

EXAMPLE 27 3
Find sin  given tan   .
4

144 Geometry 8
Solution Follow the steps.
1. Draw the triangle opposite. Let us say assign mB = a. A
2. 3 | AC| 3
tan = ,  5
3
4 |BC| 4
a
So |AC| = 3 and |BC| = 4. B C
4
2 2 2
3. AB| = |AC| + |BC|
|AB|2 = 32 + 42
|AB|2 = 9 + 16
|AB|2 = 25. So |AB| = 5.
4. | AC| 3
sin = 
| AB| 5

EXAMPLE 28 Find tan x + cot x if cos x = 5


.
13

Solution |AC|2 + |BC|2 = |AB|2


|AC|2 + 52 = 132
|AC|2 = 169 – 25
|AC|2 = 144
|AC| = 12
5
So tan x  12 and cot x  .
5 12

12 5 144 + 25 169
Therefore, tan x + cot x = + = = .
5 12 5  12 60
( 12 ) (5)

Pythagorean Theorem 145


EXAMPLE 29 4 sin x  3 cos x
2 sin x – cos x
 5 is given.

Find the ratios.


a. tan x b. cot x
c. sin x d. cos x

Solution First let us simplify the equation.

4 sin x  3 cos x 5
=
2 sin x – cos x 1
1 × (4 sin x + 3 cos x) = 5 × (2 sin x – cos x)
4 sin x + 3 cos x = 10 sin x – 5cos x
3 cos x + 5 cos x = 10 sin x – 4sin x
8 cos x = 6 sin x

a. 8 cos x 6 sin x sin x 8 sin x


  8 
6  
cos x cos x cos x 6 cos x

8 4
So tan x  .
6 3

b. 1 1 3
cot x  
tan x 4 4
3
c. Let us draw the triangle and find |AB|. A
2 2 2
|AB| = 3 + 4
|AB|2 = 25 5 4

|AB| = 5
x
B C
So sin x  4 .
3
5

d. 3
cos x 
5

146 Geometry 8
EXAMPLE 30 Find sin x if sin 2x =
3
5

Solution The question gives us a ratio for a right triangle with an angle 2x. We need to make a right
triangle with an angle x. Look at the first figure.
Let us apply the Pythagorean Theorem to DADC:
|AD|2 = |AC|2 + |DC|2
A
52 = 32 + |DC|
|DC| = 4. 3
5
Now let us draw [BD] which is congruent to [AD] as
2x
shown in the second figure (points B, D, and C are D 4 C
collinear).
If we draw [AB], then ABD = mBAD = x.
Apply the Pythagorean Theorem again to ABC:
|AB|2 = |BC|2 + |AC|2
A
|AB|2 = 92 + 32
x
|AB|2 = 90 3ò10
5 3
|AB| = ò90
x 2x
|AB| = 3ò10. B 5 D 4 C

3 1 10
So sin x  = = .
3 10 10 10
( 10 )

Pythagorean Theorem 147


F. RATIOS IN A 30°-60°-90° TRIANGLE

Look at the lengths of the sides of the triangle on the left.


We can write, sin 30° = cos 60° = a 1

2a 2
sin 60° = cos 30° = a 3 3

2a 2
tan 30° = cot 60° = a 1 3
 
a 3 3 3
cot 30° = tan 60° =
a 3
 3.
a

A Objectives
60°
After studying this section you will be able to give the trigonometric ratios of some common
2a
a angles, and use them to solve problems.
30°
B añ3 C

G. RATIOS IN A 45°-45°-90° TRIANGLE


A Similarly, by using the triangle on the left we can write,

45°
sin 45° = cos 45° = a 1 2
 
a 2 2 2
añ2
a
tan 45° = cot 45° = a
 1.
45° a
B a C This gives us the values of the trigonometric ratios of some common angles.

30° 45° 60°

sin 1 ñ2 ñ3
2 2 2

cos ñ3 ñ2 1
2 2 2

tan ñ3
1 ñ3
3

cot ñ3 1 ñ3
3

148 Geometry 8
EXAMPLE 31 Evaluate
4  sin 30  tan 60  .
tan 30   cos 45 

Solution Let us use the values from the table.

1/ 2 3
 
4  sin 30 tan 60  4 3 2 12 12 2
  3    6 2
tan 30 cos 45  2 3 2 2 2
  ( 2)
3 2
3 2

EXAMPLE 32 Simplify
4 + 2  sin 30°
.
cot 30°

Solution Let us use the values from the table.

1/
 2 4+ 2  1
4 + 2 sin 30° 2 5 5 3
=  
cot
30
  3 3 3
( 3)
3

EXAMPLE 33 Simplify

sin 52°tan 43°


3 – 4sin 60°cos 60°.
cot 47°cos 38°

Solution 3  sin52°  tan43° 3 1


– 4  3– 3
cot
 47°
 cos
38°
 2 2
tan43° sin 52°

Pythagorean Theorem 149


EXAMPLE 34 Evaluate
tan 1°×tan 2°×...×tan 88°×tan 89°.

Solution Remember that tan a × cot a = 1.


By using complementary angles we have
tan 89° = cot 1°, tan 88° = cot 2°, ... , tan 46° = cot 44°.
So we have
tan 1° × tan 2° × ... × tan 88° × tan 89
= tan 1° × tan 2° × ... × tan 44° × tan 45° × cot 44° × ... × cot 2° × cot 1°
= (tan 1° × cot 1°) × (tan 2° × cot 3°) × ... × (tan 44° × cot 44°) × tan 45°
  
1 1 1

= tan 45°
= 1.

Activity
1. Simplify the ratios.

tan 30°cot 60° 3+ 2 sin 30°


a. b.
sin 30° 1 3  tan60°
2. sin 30° sin 60°+cos 30° cos 60°
Evaluate .
tan 30° tan 60° + tan 45°

3. Evaluate cot 1° × cot 2° × cot 3° × ... × cot 88° × cot 89°.

150 Geometry 8
EXERCISES 2 .1
1. Write the ratios for the triangle in the figure. A
tan 27° (sin 2 13° cos 2 13° )
5. Simplify .
a. sin  b. cos  (sec 2 5° – tan 2 5° ) cot 63°
9
c. tan  d. cot 
a sin 5°  sin 10° sin 15° sin 20°
e. sec  f. cosec  B C 6. Simplify .
12 cos 70°  cos 75°  cos 80° cos 85°
g. sin (90° – ) h. cot (90° – )
7. Simplify cot 5°  cot 10°  cot 15°  ...  cot 85°.
2. Simplify the ratios.
5  5 cos 2 x
8. Simplify .
a. sin x cot x
2 2
tan2 x  cos 2 x
b. cot2 x sec2 x sin x
9. Find the values.
c. (sin x + cos x cot x) tan x
a. sin 45°  cos 30°  tan 60°  cot 45°
d. cot x (tan x + cot x)
b. tan 60°  cot 60° + sin2 60° + cos2 60°
2
e. 1 tan x c. sec 30° + cot 45° + cos 30°
tan2 x
10. Find sin , cos , tan if cot  = 24 .
2
f. 1– cosec x
1– sin2 x 7

g. 1 sec   cos  4
– cot x 11. Find given sin  = .
sin x  cos x tan   cosec  5

3. Find the value of x in each equation. 12. Find cot x if tan 2x = 2.


a. tan x = cot 73° 13. Find the ratios using a trigonometric table.
b. sin 2x = cos 66° a. cos 17° b. tan 46° c. sin 78°
c. cos (x – 10°) = sin 70° 14. Use a table of trigonometric ratios to find the
d. cot (5x + 5°) = tan 15° approximate measure of A.
a. sin A = 0.743 b. cot A = 1.304

4. Verify each equation. c. cos A = 0.346 d. tan A = 2.426

a. sin (cosec  – sin ) = cos2  15. How tall is the building on


b. sin tan  + cos  = sec  right?

c. (sin – cos )2 + (sin + cos )2 = 2


2 42°
d. 1+ cot x  cot 2 x 103 m
1 tan2 x
sin x cos x 16. A plane makes an angle of depression of 33° with
e.  1

cosecx sec x a runway. Its altitude is 5200 m. Find the horizon-
tal distance from the plane to the runway.

Pythagorean Theorem 151


1. Distance Between Two Points
Let us use x0, x1, x2, ... and y0, y1, y2, ... to denote the abscissas and the ordinates of
respective points in the coordinate plane.

Theorem distance between two points


The distance between two points A(x1, y1) and B(x2, y2) is

AB = ( x2  x1 )2 +( y2  y1 )2 .

Proof In the figure, ABC is a right triangle.


AC = x2 – x1
BC = y2 – y1
By the Pythagorean theorem,
AB2 = AC2 + BC2
AB2 = (x2 – x1)2 + (y2 – y1)2
and so AB = ( x2  x1 )2 +( y2  y1 )2 or

AB = ( x1  x2 )2 +( y1  y2 )2 .

EXAMPLE 35 Find the distance between A(3, 0) and B(–2, –3).


Solution AB = ( x2 – x1 )2 +( y2 – y1 ) 2 = (–2 – 3) 2 +(–3 – 0) 2 = (–5)2 +(–3) 2 = ò34 units.

EXAMPLE 36 Show that ABC with the vertices A(–2, 2), B(1, 5), and C(4, –1) is an isosceles triangle.
Solution Let us find the length of the sides of ABC. y
B
AB = (1  2)2  (5  2)2 = 9  9 
3 2 W

AC = (4  2)2 +( 1  2)2 = 36+9 = 3 5


A 2
2 2
BC = (4  1) +( 1  5) = 9+ 36 = 3 5 4
x
–2
AC = BC, so two sides of the triangle have the same length. –1
C
Therefore, ABC is isosceles.

152 Geometry 8
EXAMPLE 37 A(a, 2), B(3, 4), and C(–2, 1) are given. If A is at the same distance from the points B and C, find a.
Solution We are given AB = AC. By the theorem for the distance between two points,

(3  a)2 + 2 2 = ( a + 2)2 +12

9  6a + a2 + 4 = a2 + 4a + 4+1
10 a = 8

4
a= .
5

EXAMPLE 38 Find the ordinate of the point on the y-axis which is equidistant to the points A(– 4, 0) and
B(9, 5).

Solution The point is on the y-axis, so its x-coordinate is 0. y


Let us call the point P(0, k). Now, from the diagram, P(0( k)
PA = PB
( 4)2 + k2 = 92 +( k  5)2 W B(9( 5)
2 2 2 2
(– 4) + k = 9 + (k – 5)
16 + k2 = 81 + k2 – 10k + 25 x
A(–4( 0) O 9
10k = 90
k = 9. Therefore, the point is P(0, 9).

Check Yourself
1. Find the distance between the points A(2, –1) and B(–2, 2).
2. Find the lengths of the sides of the triangle MNP with vertices at the points M(–1, 3),
N(–2, –3), and P(5, 1).
3. The points K(2, 1) and L(–6, a) are given. If KL = 10 cm, find the possible values of a.
4. A is a point on the y-axis with ordinate 5 and B is the point (–3, 1). Calculate AB.
5. Find the point on the y-axis which is equidistant to the points A(–3, 0) and B(4, –1).
Answers
1. 5 2. ò37, 2ò10, ò65 3. a  {–5, 7} 4. 5 5. (0, –4)

Pythagorean Theorem 153


2. Midpoint of a Line Segment
Theorem midpoint of a line segment
Let the points A(x1, y1) and B(x2, y2) be the endpoints of a line segment AB, and let C(x0, y0)
be the midpoint of AB. Then,
x + x2 y + y2
x0 = 1 and y0 = 1 .
2 2

Proof Let us take point C on AB such that AC = CB.


From the figure, CAK  BCD. y

So AK = CD and CK = BD.
B
y2
Now, x0 – x1 = x2 – x0 and y0 – y1 = y2 – y0
y0 C
2x0 = x1 + x2 and 2y0 = y1 + y2 D

x1 + x2 y1 + y2 y1 A
x0= and y0= . K
2 2
x
x1 + x2 y1 + y2 O x1 x0 x2
So C(x0, y0) = ( , ).
2 2

EXAMPLE 39 A(–1, –2) and B(–5, 4) are given. Find the coordinates
of the midpoint of AB.

x1 + x2 1  5 y + y2 2+ 4
Solution x0 = = = 3, y0 = 1 = =1
2 2 2 2
So C(–3, 1) is the midpoint of AB.

EXAMPLE 40 A triangle ABC with vertices A(–2, –2), B(1, 8), and C(6, 2) is given. If the points D and E are
BC
midpoints of AB and AC respectively, show that ED = .
2

Solution First, let us find the coordinates of D(a, b) and E(c, d). Points D(a, b) and E(c, d) are the
midpoints of AB and AC, so their coordinates are
x1 + x2 2+1 1
a= = = 
2 2 2 1
 D(  , 3),
y + y2 2+8  2
b= 1 = =3 
2 2

154 Geometry 8
x1 + x2 2+6 y
c= = =2
2 2 
 E(2, 0). 8
B(1( 8)
y + y2 2+ 2 
d= 1 = = 0
2 2

Now, let us find the length of ED and BC by using the


distance formula, and then compare their lengths: D(a( b)
C(6( 2)
2
1
ED = (2+ )2 +(0  3) 2 –2
2 x
O E(c( d) 6
25 61 61 –2
= 9=  A(–2( –2)
4 4 2

BC = (6  1)2 +(2  8) 2

= 25  36 = 61.

BC
Hence, ED = .
2

Rule
Let the points A(x1, y1), B(x2, y2), C(x3, y3), and D(x4, y4) D(x4( y4) C(x3( y3)
be vertices of a parallelogram ABCD, and let P(x0, y0) be
the intersection point of the diagonals.
P(x0( y0)
Since P(x0, y0) is the midpoint of the diagonals,
x1 + x3 x + x4
x0 = and x0 = 2 , so x1 + x3 = x2 + x4. A(x1( y1) B(x2( y2)
2 2
y1 + y3 y  y4
y0 = and y0 = 2 , so y1 + y3 = y2 + y4.
2 2
As a result, for any parallelogram ABCD with given vertices the following rules are valid:
x1 + x3 = x2 + x4 and y1 + y3 = y2 + y4 .

EXAMPLE 41 KLMN is a parallelogram with vertices K(2, a), N M

L(1, 4), M(b, 3), and N(3, 2). Find a – b.

Solution The midpoint of KM is also the midpoint of NL, so


2+b=1+3 and a+3=4+2 K L
b=2 a = 3.

Therefore, a – b = 3 – 2 = 1.

Pythagorean Theorem 155


Check Yourself
1. A(a + 1, 4 – 2b) and B(3 – a, 2b – 3) are given. Find the coordinates of the midpoint of AB.
2. A triangle ABC with vertices A(2, 5), B(–2, 3), and C(4, –1) is given. Find the length of
the median passing through A.
3. The points A(–2, –3), B(3, –2), C(x, y), and D(–1, 3) are the vertices of a parallelogram
ABCD. Find the coordinates of C.
Answers
1
1. (2, ) 2. ò17 3. (4, 4)
2

Properties 4 Triangle Inequality Theorem


In any triangle ABC with sides a, b and c, the following inequalities are true:
|b – c| < a < (b + c),
|a – c| < b < (a + c),
|a – b| < c < (a + b).
The converse is also true. This property is also called the Triangle Inequality Theorem.

EXAMPLE 42 Is it possible for a triangle to have sides with the lengths indicated?
a. 7, 8, 9 b. 0.8, 0.3, 1 c. 1 , 1 , 1
2 3
Solution We can check each case by using the Triangle Inequality Theorem.
a. |9 – 8| < 7 < (8 + 9) b. |0.8 – 0.3| < 1 < (0.8 + 0.3) c. This is impossible,
|8 – 9| < 8 < (7 + 9) |1 – 0.3| < 0.8 < (1 + 0.3) since
1 1
|7 – 8| < 9 < (7 + 8). |1 – 0.8| < 0.3 < (1 + 0.8). 1< + .
2 3
This is true, so by the This is true, so by the
Triangle Inequality Triangle Inequality Theorem
Theorem this is a this is a possible triangle.
possible triangle.

EXAMPLE 43 Find all the possible integer values of x in the A


figure.
10
5

B C
x
4
7
D

156 Geometry 8
Solution In ABC, |10 – 5| < x < (10 + 5) (Triangle Inequality Theorem)
5 < x < 15. (1)
In DBC, |7 – 4| < x < (7 + 4) (Triangle Inequality Theorem)
3 < x < 11. (2)
The possible values of x are the elements of the common solution of inequalities (1) and (2),
i.e. 5 < x < 11.
So x  {6, 7, 8, 9, 10}.

EXAMPLE 44 Find the greatest possible integer value of m A


in the figure, then find the smallest possible
integer value of n for this case. 9
n
m
B
6
D 8 C

Solution In ABD, |9 – 6| < m < (9 + 6) (Triangle Inequality Theorem)


3 < m < 15.
So the greatest possible integer value of m is 14.
In ADC, |8 – m| < n < (m + 8) (Triangle Inequality Theorem)
|8 – 14| < n < (14 + 8) (m = 14)
6 < n < 22.
So when m = 14, the smallest possible integer value of n is 7.

EXAMPLE 45 In a triangle ABC, m(A) > 90°, c = 6 and b = 8. Find all the possible integer lengths of a.

Solution Since m(A) > 90°, b2 + c2 < a < (b + c) by Property 5.1.

Substituting the values in the question gives 82 +6 2 < a < (8 + 6), i.e.

10 < a < 14. So a  {11, 12, 13}.

Pythagorean Theorem 157


Check Yourself
1. Two sides of a triangle measure 24 cm and 11 cm respectively. Find the perimeter of the
triangle if its third side is equal to one of other two sides.
2. Determine whether each ratio could be the ratio of the lengths of the sides of a triangle.
a. 3 : 4 : 5 b. 4 : 3 : 1 c. 10 : 11 : 15 d. 0.2 : 0.3 : 0.6
3. The lengths of the sides DE and EF of a triangle DEF are 4.5 and 7.8. What is the greatest
possible integer length of DF?
4. The base of an isosceles triangle measures 10 cm and the perimeter of the triangle is an
integer length. What is the smallest possible length of the leg of this triangle?
5. In an isosceles triangle KLM, KL = LM = 7 and m(K) < 60°. If the perimeter of the
triangle is an integer, how many possible triangle(s) KLM exist?
6. In a triangle ABC, AB = 9 and BC = 12. If m(B) < 90°, find all the possible integer
lengths of AC.
Answers
1. 59 cm 2. a. yes b. no c. yes d. no 3. 12 4. 5.5 cm 5. six triangles
6. AC  {4, 5, 6, 7, 8, 9, 10, 11, 12, 13, 14}

158 Geometry 8
H. EUCLIDEAN RELATIONS
Theorem
The altitude to the hypotenuse of a right triangle divides the right triangle into two smaller
right triangles which are similar to the original triangle, and therefore also similar to each other.

Proof Look at the first figure. A

Given: ABC is a right triangle and AH is the


altitude to the hypotenuse.
Prove: ABC  HBA  HAC
B H C
We will give the proof in paragraph form.
Let m(BCA) = x.
A A
Then, m(ABC) = 90° – x, m(HAB) = x
x
and m(HAC) = 90° – x. 90° – x

So each smaller triangle is similar to the


90° – x x
larger triangle by the AA Similarity Theorem,
B H H C
and therefore the two smaller triangles are
Remember! A
also similar to each other.
The geometric mean of
two numbers a and b is
a positive number x
a x
such that = , 90° – x
x b x
i.e. x = a  b. B H C

This theorem leads us to two more useful theorems.

Theorem Euclidean theorems

In any right triangle, when the altitude to the A


hypotenuse is drawn, the following two
statements are true: c
b
h
1. The length of the altitude is the geometric
mean of the lengths of the two segments B p H q C
of the hypotenuse formed by the altitude a
(AH2 = BH  CH in the figure).
2. The length of each leg is the geometric mean of the length of its adjacent hypotenuse
segment and the length of the hypotenuse. (CA2 = CH  CB in the figure).

Pythagorean Theorem 159


Proof Let us draw an appropriate figure (shown at the right).
Given: ABC is a right triangle and AH is the altitude to the hypotenuse.
Prove: AH2 = BH  CH (1) and
For any right triangle
ABC, the relations BA2 = BH  BC and CA2 = CH  CB (2)
h2 = p  q, c2 = p  a and We will write the proof of (1) in paragraph form.
b2 = q  a are also called
Euclidean relations. By the theorem at the beginning of this
A section, AHB  CHA.
c b By the definition of similarity, corresponding sides are proportional:
h
BH AH
p q
= , i.e. AH2 = BH  CH, as required.
B H C AH CH
a
Now let us prove (2). By the same theorem, HBA  ABC. So by the definiton of
similarity, corresponding sides are proportional:
BH BA
, i.e. BA = BH  BC.
2
=
BA BC
CH CA
, i.e. CA = CH  CB.
2
By a similar argument, HAC  ABC. So =
CA CB

EXAMPLE 46 Find the lengths a, c and x in the figure. B

Solution Since ABC is a right triangle and BH is an


c a
altitude, we can use the Euclidean relations: 6
h2 = p  q; 62 = x  3; x = 12,
a2 = 3  (3 + 12) = 3  36; a = 6ñ3, A x H 3 C

c = 12  (12 + 3) = 180; c = 6ñ5.


2

EXAMPLE 47 Prove that


1 1
= +
1
h2 b2 c2
in the figure. A

b c
h

B k H p C
a

160 Geometry 8
A. THE CONCEPT OF AREA
1. Basic Definitions
Definition polygonal region
The union of a polygon and its interior region is called a polygonal region.

We name polygons by their vertices. For example, ABC is the name of a triangle with
sides name vertices at points A, B and C, and ABCD is the name of a quadrilateral with vertices at points
3 triangle
A, B, C and D. We use extra notation to refer E
4 quadrilateral
5 pentagon to a polygonal region: (ABC) is a triangular F
region, and (ABCDE) is a pentagonal region. D
6 hexagon
7 heptagon In the figure, (ABCDEF) is the union of the
8 octagon A
hexagon and its interior region. Since
9 nonagon
10 decagon ABCDEF is a hexagon, we can say that
. . B
. . (ABCDEF) is a hexagonal region. C
. .

Definition square unit


The interior region of a square with side length one unit is called a square unit. We write
unit2 to mean a square unit.
In the figure opposite, each side of the square
measures 1 unit and so its area is 1 square
1 unit
unit, or 1 unit2. We can also use metric units
for lengths and areas: a square with side 1 cm
has area 1 cm2, and a square with side 1 m
1 unit
has area 1 m2, etc.

Definition area
The area of a closed plane figure is the total number of non-overlapping square units and part
units that cover the surface of the polygonal region. The area of a figure is always a positive
real number.
We use the letter A to mean the area of a polygon: the area of ABC is A(ABC), and the area
of the pentagon ABCDE is A(ABCDE).
If the sides of a figure are not natural numbers or if the polygon is very big, it is difficult to
find its area by counting the individual unit squares. In this book we will learn a set of
formulas and methods to find the area of any geometric figure by calculation.

170 Geometry 8
Definition altitude, height
An altitude is a line segment between a vertex of a polygon and a line containing a side of the
polygon, which is perpendicular to this line. The length of an altitude is called a height of the
polygon. We write ha, hb, etc. to mean the altitudes to sides a, b, etc. of a polygon.

Definition base
The side of a polygon from which we draw an altitude is called the base. We can use any side
of a polygon as a base.

altitude

altitude
altitude

altitude
base
or

base altitude base base


rectangle triangle parallelogram

Note
In an isosceles triangle, the congruent sides are called the legs of the triangle and the third
side is the base.

Postulate area congruence postulate


If two figures are congruent then their areas are the same.

Two polygons are


congruent if their
corresponding sides and
angles are the same.

2. Area of a Rectangle
Postulate
The area of a rectangle is the product of the
D C
lengths of two consecutive sides:

A(ABCD) = a  b . a

A b B

EXAMPLE 1 Two sides of a rectangle measure 14 cm and 20 cm.


What is the area of this rectangle?

Solution Let the sides of the rectangle be a = 14 cm and b = 20 cm. Then


A = a  b = 14  20 = 280 cm2.

171 Geometry 8
EXAMPLE 2 A rectangle has area 84 cm2 and one of its sides measures 7 cm. What is this perimeter of this
rectangle?
Solution Let us write a = 7 cm and A = 84 cm2. So A = a  b gives us 84 = 7  b, i.e. b = 12 cm.
So the perimeter of the rectangle is 2(a + b) = 2(7 + 12) = 38 cm.

EXAMPLE 3 In the figure, ABCD is a rectangle with D C


AC = 10 cm and
AB = 8 cm. Find the area of this rectangle. 10

A 8 B

Solution We can use the Pythagorean Theorem or


special right triangles to find the length of
BC = b. By the Pythagorean Theorem,
AB2 + BC2 = AC2
82 + b2 = 102
b = 6 cm.
So A(ABCD) = a  b = 6  8 = 48 cm2.

EXAMPLE 4 A rectangle has area 108 cm2 and one of its sides measures three times the other side. Find the
perimeter of this rectangle.

Solution Let a = x and b = 3x be the side lengths since one side is three times as long as the other
side.
Then A = ab gives us 108 = x  3x = 3x2, so x2 = 36 and x = 6.
So a = 6 cm and b = 18 cm.
So the perimeter of ABCD is 2  (a + b) = 2  (6 + 18) = 48 cm.

Rule
If we connect a point on one side of a
D E C
rectangle to the endpoints of the opposite
side then the area of the triangle obtained is
half the area of the rectangle: in the figure, b

A( ABCD ) a  b .
A( ABE) =  A a B
2 2

Areas of Quadrilaterals 172


EXAMPLE 5 In the figure, ABCD is a rectangle with sides D E C
AB = 8 cm and AD = 6 cm, and E is a point
on side DC.
6
Find the combined area of the shaded
regions.
A 8 B

A( ABCD )
Solution By the previous rule, A( ABE) = .
2
A( ABCD )
So the sum of the shaded areas will also be .
2

So the combined area is

A( ABCD ) 6  8
= = 24 cm 2 .
2 2

Check Yourself
1. A rectangle has perimeter 40 cm and one side is 4 cm longer than the other side. Find
the area of this rectangle.

2. In the figure, ABCD is a rectangle with BD = 8 cm D C


and m(ABD) = 30°. Find A(ABCD).
8

30°
A B

3. A rectangle has area 48 cm2 and perimeter 28 cm. Find the lengths of the sides of this
rectangle.

4. One side of a rectangle is twice as long as another side. Given that the perimeter of this
rectangle is 30 cm, find its area.
Answers

1. 96 cm2 2. 16ñ3 cm2 3. 6 cm, 8 cm 4. 50 cm2

173 Geometry 8
B. AREA OF A TRIANGLE
Theorem base-height formula
The area of a triangle is half the product of
A
the length of one base and the height of the
triangle from that base: in the figure,
ah . h
A( ABC ) =
2

B H C

Proof Look at the figure.


B¢ A C¢
In ABC, BC = a.
We draw a line parallel to BC through A, and
In a triangle, the side
opposite vertex A is from B and C we draw perpendiculars BB' h
called a, the side
opposite B is called b,
and CC' to the parallel line.
and the side opposite C
We can say that BCCB, BHAB and AHCC B H C
is called c.
are rectangles, and also BB = CC = AH = h.
Also, ABH  BAB since ABH  BAB, BAH  BBA and AB is a common side.
By the Area Congruence Postulate we can write A(ABH) = A(ABB) = X.
By similar reasoning we have
Area Congruence A(AHC) = A(ACC') = Y.
Postulate: If two figures
are congruent then their So A(BCCB) = a  h = 2X + 2Y
areas are the same. ah
= 2  (X + Y), i.e. X + Y = .
2
ah
Finally, A(ABC) = X + Y = .
2
ah
So A( ABC ) = , as required.
2

Note
a  ha b  hb c  hc
We can use any side of a triangle as a base, so A( ABC ) = = = .
2 2 2

Areas of Quadrilaterals 174


EXAMPLE 6 In the figure, A
BC = 7 cm and
AH = 4 cm.
Find A(ABC). 4

Solution In the figure, BC = a = 7 and


B H C
AH = h = 4.
7
ah 7 4
So A( ABC ) = = =14 cm 2.
2 2

EXAMPLE 7 In the triangle opposite, A


AH = 8,
D
BC = 12 and 10
AC = 10. 8
Find the length of BD.
B H C
Solution BC = a = 12,
12
AC = b = 10,
AH = ha = 8 and we need to find hb.
a  ha b  hb 12  8 10  hb 48
We have = , so = and so hb = .
2 2 2 2 5

EXAMPLE 8 The base of an isosceles triangle measures 12 cm and the other sides are each 10 cm long.
Find the area of this triangle.

Solution The figure shows the triangle with


A
BC = a = 12 cm.
We draw the altitude AH = ha. Because the 10 10
triangle is isosceles, H will be the midpoint of
side BC.
By the Pythagorean Theorem in AHC, B 6 H 6 C

ha2 + 62 = 102
ha2 = 100 – 36
ha2 = 64
ha = 8 cm.
a  ha 12  8
So A( ABC )    48 cm 2 .
2 2

175 Geometry 8
EXAMPLE 9 In the triangle ABC opposite, A
BC = 14 cm,
AC = 8 cm and 8
m(ACB) = 60°.
Find A(ABC). 60°
B 14 C

Solution We know BC = a = 14.


Let us draw AH = ha. A

Then AHC is a right triangle, and so we can


use the properties of a 30°-60°-90° triangle: if 8
A ha
the hypotenuse AC measures 8 cm then the
2x 60° 60°
x side opposite the 60° angle measures
B H C
8 3
30° = 4 3  ha .
C xñ3 B 2
a 30°H60°H90° triangle a  ha 14  4 3
So A( ABC ) = = = 28 3 cm 2 .
2 2

EXAMPLE 10 In the figure, AB = AD = 6 cm, A


BD = 8 cm and
DC = 4 cm. 6
Find A(ADC).

B 8 D 4 C

Solution Let us draw the altitude AH to side BC and write AH = h.


ABD is isosceles, so BH = HD = 4 cm.
Now we can use the Pythagorean Theorem in ABH to find h:
h2 + 42 = 62 and h2 = 20, i.e. h = 2ñ5 cm.
AH  DC 2 5  4
So A( ADC ) = = = 4 5 cm 2 .
2 2

Check Yourself
1. Find the area of the triangle with the given base and altitude.

a. a = 4, ha = 7 b. b = 3, hb = 8

2. The sides of ABC are a = 6 cm, b = 8 cm and c = 10 cm, and A(ABC) = 24 cm2. What
are the three heights of this triangle?

Areas of Quadrilaterals 176


3. In a triangle, a = 6 cm and c = 12 cm. Find hc if ha = 10 cm.

4. In the isosceles triangle opposite, A

AB = AC, m(A) = 120° and the length of 120°


the base is BC = 18. Find A(ABC).
B 18 C
Answers
24
1. a. 14 b. 12 2. ha = 8 cm, hb = 6 cm, hc = cm 3. 5 cm 4. 27ñ3
5

Theorem area of a right triangle


The area of a right triangle is half the
A
product of its legs: in the figure,
b
ac . c
A( ABC ) =
2

B a C

Proof Let us draw a line from A parallel to BC, and A C¢

let the foot of the perpendicular from C to


this line be C. c

Then ABCC is a rectangle, because


B a C
AC BC, m(ABC) = 90° and
m(ACC) = 90. So m(BAC) = 90°.
Also, ABC is congruent to CCA by the ASA Congruence Theorem, since
m(BAC) = m(ACC), m(ACB) = m(CAC) and AC is a common side.
So we can write A(ABC) = A(CCA) = X.
Since ABCC is a rectangle, A(ABCC) = a  c = A(ABC) + A(CCA) = X + X = 2X.
A( ABCC ) a  c
So X = A(ABC) =  , as required.
2 2

177 Geometry 8
EXAMPLE 11 In the triangle opposite, m(C) = 90°, A
AB = 13 cm and AC = 5 cm.
13
Find A(ABC). 5

Solution AB = c = 13 cm and AC = b = 5 cm are


B a C
given, and we need to find BC = a to find the
area. By the Pythagorean Theorem in ABC,
a2 + 52 = 132 so a2 = 169 – 25 = 144, a = 12 cm.
a  b 12  5
So A( ABC ) = = = 30 cm 2 .
2 2

EXAMPLE 12 The hypotenuse of an isosceles right triangle measures 7ñ2 cm. Find the area of this triangle.

Solution We know from basic trigonometry that if the sides of an isosceles


right triangle measure x units then the hypotenuse measures A
xñ2 units.
45°
So xñ2 = 7ñ2, which gives x = 7, i.e. a = c = 7 cm. xñ2
x
a  c 7  7 49
So A( ABC ) = = = cm 2 .
2 2 2 45°
C x B
an isosceles right triangle

EXAMPLE 13 In the given figure, ABCD is a rectangle with D F C


m(AFD) = 30°, m(BEC) = 45°, 30°
12
AF = 12 and AE = 8.
Find the area of quadrilateral AECF. 45°
A 8 E B

12
Solution ADF is a 30°-60°-90° triangle so AD = = 6 and DF = 6ñ3.
2
Since ABCD is a rectangle, BC = AD = 6.
EBC is an isosceles right triangle (because m(BEC) = m(ECB) = 45°), so EB = BC = 6.
So in rectangle ABCD, AD = a = 6 and AB = b = AE + EB = 8 + 6 = 14.
6  6 3 6 6
Finally, A(AECF) = A(ABCD) – (A(ADF) + A(EBC)) = 6  14 – ( + )
2 2
= 84 – (18ñ3 + 18) = 66 – 18ñ3.

Areas of Quadrilaterals 178


Theorem area of an equilateral triangle
The area of an equilateral triangle with side
A
length a is one-fourth of the product of a2
and ñ3: in the figure,
a a
a2 3 .
A( ABC ) =
4

B a C

Proof Let us draw the altitude AH in ABC. A

Since ABC is equilateral,


30°
a 3
m(C) = 60° and so AH = ha = . a
2 añ3
2
a 3
a
a  ha a2 3
So A( ABC ) = 2  . 60°
2 2 4 B H a C
2

EXAMPLE 14 One side of an equilateral triangle measures 6 cm. Find the area of this triangle.

a2 3 6 2 3
Solution By the theorem above, A =   9 3 cm 2 .
4 4

EXAMPLE 15 The height of an equilateral triangle is 10 cm. Find its area.

a 3 20
Solution The height is h =  10 , so a = cm.
2 3
20 2
( ) 3
a2 3 400 3 100 3
So A( ABC ) = = 3 = = cm 2.
4 4 12 3

EXAMPLE 16 The area and perimeter of an equilateral triangle have the same value. Find the length of one
side of this triangle.
a2 3
Solution Let the side measure a, then the area is and the perimeter is 3a.
4
a2 3
If the area and the perimeter have equal values then = 3a and so
4
12
a= = 4 3 is the length of one side of the triangle.
3

179 Geometry 8
Check Yourself
1. The hypotenuse of a right triangle measures 25 units and one of its legs measures
24 units. Find the area of this triangle.
2. An isosceles right triangle has a hypotenuse of 10 units. Find its area.
3. In the figure, ABCD is a rectangle, D C
EB = 8, m(BEC) = 45° and
EC = DC. Find the area of CDE.

45°
A E 8 B
4. Find the area of the equilateral triangle with the given side length.
a. 4 b. 10
5. Find the area of the equilateral triangle with the given height.
a. 14ñ3 b. 8 c. 2ñ3
Answers
1. 84 2. 25 3. 32ñ2 4. a. 4ñ3 b. 25ñ3 5. a. 196ñ3 b. 64 3 c. 4ñ3
3

The figure shows two triangles. We cut the first triangle along the lines shown
and rearrange the parts to get the second triangle.
When we do this, we can see that there is an empty space in the second
triangle. Where is the missing square?

?
Answer
If we draw the figures accurately and with a large scale, we will see that the slopes of the red and blue
triangles are different. This means that the bigger shapes are not triangles. So we cannot calculate the areas
directly by using the area formula, and in fact the areas are not equal.

?
Theorem Heron’s Formula
If a triangle has sides a, b and c and perimeter A
2u then the area of the triangle is the square
root of the product of u, u – a, u – b and
c b
u – c: in the figure,
abc
u=
2 B a C

A( ABC ) = u( u  a)( u  b )( u  c) .

Proof Look at the figure. Let us draw the altitude A


AH to BC and let AH = h, BH = x,
HC = a – x and A(ABC) = A. b
c
By the Pythagorean Theorem in ABH and h
AHC respectively we get
x2 + h2 = c2, h2 = c2 – x2, (1) B x H a–x C
Heron of and (a – x)2 + h2 = b2, h2 = b2 – (a – x)2. (2)
alexandriA
From (1) and (2) we have h2 = c2 – x2 = b2 – (a – x)2
(10-75 AD)
c2 – x2 = b2 – (a2 – 2ax + x2) (expand the binomial)
c2 – x2 = b2 – a2 + 2ax – x2 (simplify)
a 2  b2  c2
2ax = a2 – b2 + c2, i.e. x = . (3) (rearrange)
2a
Heron of Alexandria was a Greek
a 2  b2  c2 2 ( a2  b2  c2 )2
mathematician and inventor who Let us use (3) in (1): h2 = c2 – ( ) , i.e. h2 = c2 – .
lived in Alexandria in Egypt. 2a 4 a2
Heron was mainly interested in
the practical study of mechanics (2 ac)2  ( a2  b2  c2 )2
and engineering. He also studied Equalizing denominators gives us h2 = .
geometry, optics, astronomy and
4a 2
architecture. In geometry, he So 4a2h2 = (2ac – (a2 – b2 + c2))  (2ac + a2 – b2 + c2). (4) (difference of two squares)
found the area of a triangle by
using square roots and the lengths ah a2 h2
of the sides. He invented many
We know that A( ABC ) = A  , so A 2 = , i.e. 16A2 = 4a2h2. So
2 4
machines and devices such as
fountains and syphons, and he 16A2 = (b2 – (a2 – 2ac + c2))  (a2 + 2ac + c2 – b2) (from (4))
also invented the first steam
powered device.
16A2 = (b2 – (a – c)2)  ((a + c)2 – b2) (use a2 ± 2ac + c2 = (a ± c)2)
Heron wrote around fifteen books 16A2 = (b – a + c)(b + a – c)(a + c – b)(a + c + b). (5) (difference of two squares)
about mathematics, engineering We know that the perimeter of ABC is 2u, i.e. a + b + c = 2u.
and astronomy, including
Katoptirikos (about optics), Let us substitute this in (5):
Automata, Mechanica, Geometrica 2 u  2( u  a)  2( u  b)  2( u  c)
and Stereometrica. The proof of 16A2 = (2u – 2a)(2u – 2c)(2u – 2b)(2u), i.e. A 2 =
the formula presented here 16
appears in his book Metrica. and so A = u  ( u  a )  ( u  b )  ( u  c), as required.

181 Geometry 8
EXAMPLE 17 Find the area of the triangle with side lengths 4 cm, 5 cm and 7 cm.

a + b + c 4+5+7
Solution We can use Heron’s Formula with u = = = 8:
2 2

A( ABC ) = 8(8  4)(8  5)(8  7) = 8  4  3  1=4 6 cm 2.

EXAMPLE 18 The sides of a triangle ABC are a = 7, b = 9 and c = 12. Find hc.

7  9  12
Solution By Herons’s Formula with u =  14 we have
2

A( ABC ) = 14  (14  7)  (14  9)  (14  12) 


14 5.

c  hc 12  hc 7 5
Also, A( ABC ) = . So 14 5 = and hc  .
2 2 3

Theorem trigonometric formula for the area of a triangle


The area of a triangle is half the product of
A
any two sides and the sine of the angle
between these two sides: in the figure,
b
c
1
A( ABC ) = ab sin C
2
1 B a C
= ac sin B
2
1
= bc sin A .
2
A
Proof Look at the figure.
Let us draw the altitude from the vertex A to
side BC, so AH = ha. From the figure,
c b
h ha
sin C = a , i.e. ha = b  sin C.
b
a  ha 1
So A( ABC ) =  ab  sin C.
2 2 B H C
The proofs for the other pairs of sides are
a
similar.

Areas of Quadrilaterals 182


EXAMPLE 19 In the figure, AB = c = 3 cm, A
BC = a = 6 cm and m(B) = 60°.
What is A(ABC)? 3

Solution We know that a = 6, c = 3 and m(B) = 60°. 60°

1 B 6 C
Since A( ABC ) = ac  sin B, we have
2
1 9 3
A( ABC ) =  6  3  sin60°  cm 2 .
2 2

EXAMPLE 20 In the figure, AD = 5 cm, A


BD = 6 cm,
AE = 8 cm and 5
8
EC = 2 cm. D

A( ADE)
What is ? 6 E
A( ABC )
2
Solution We can find both of the areas using the B C
trigonometric formulas we have just seen.
We know AD = 5, AE = 8, AB = 11 and AE = 10, so
1
 AD  AE  sin A
A( ADE) 2 AD  AE 5 8 4
= = = = .
A( ABC ) 1 AB  AC 11 10 11
 AB  AC  sin A
2

EXAMPLE 21 In the figure,


A
AB = 6 cm,
BC = 8 cm,
6 3ñ2
AC = 3ñ2 cm and
m(C) = 45°.
a 45°
Find .
B 8 C
Solution We have AB = c = 6, BC = a = 8 and
2
AC = b = 3ñ2, and we know sin 45°= .
2
1 1 2
Since A( ABC ) = ab  sin C  ac  sin , we have 3 2  6  sin which gives us
2 2 2
1
sin  , i.e. = 30° or = 150°.
2
Since m(C) = 45°, = 150° is impossible, and so = 30°.

183 Geometry 8
EXAMPLE 22 The sides of PQR are p = 5, q = 12 and r = 15. Find sin R.

Solution Let us find A(PQR) by using both Heron's Formula and the trigonometric formula for the
area of a triangle.
5+12+15
By Heron’s Formula with u = =16,
2
A( PQR ) = 16  (16  5)  (16  12)  (16 15) = 8ò11.
1 1
By the trigonometric formula, A( PQR ) =  p  q  sin R =  5  12  sin R  30  sin R.
2 2
8 11 4 11
So 30  sin R = 8ò11, and so sin R = = .
30 15

Check Yourself
1. A triangle has sides of 13 cm, 14 cm and 15 cm. Find its area.
2. The sides of a triangle are a = 3, b = 5 and c = 6. Find the three heights of this triangle.
3. AB = 8 cm and AC = 6ñ3 cm are two sides of a triangle ABC. Find A(ABC) if
a. m(A) = 30°. b. m(A) = 60°. c. m(A) = 90°.
4. The sides of a triangle are a = 8, b = 7 and c = 9. What is the sine of angle B?
Answers
4 14 4 14 2 14 5
1. 84 cm2 2. ha =  , hb  , hc 3. a. 12ñ3 cm2 b. 36 cm2 c. 24ñ3 cm2 4.
3 5 3 3

Definition incircle, inscribed circle, incenter, inradius


The incircle (or inscribed circle) of a triangle is a circle inside the triangle which is tangent to
each of its sides. The center of the incircle is called the incenter. It lies at the intersection
point of the angle bisectors of the triangle. The radius of the incircle is called the inradius (r).

Theorem area of a triangle by its inradius


The area of a triangle is the product of half
A
its perimeter and its inradius: in the figure,
if a + b + c = 2u then

A(ABC) = u  r . c b
r r
I

B a C

Areas of Quadrilaterals 184


Proof Look at the figure. Let I be the incenter of
A
ABC and let r be its inradius. If we connect
I and the vertices A, B and C we can write
A(ABC) = A(AIB) + A(BIC) + A(AIC) c b
r I
r
cr ar br
= + + r
2 2 2
B a C
a+ b+ c
= r
2
= u  r.

EXAMPLE 23 A triangle with perimeter 20 cm has inradius 6 cm. Find the area of this triangle.

Solution If the perimeter is 2u then 2u = 20, i.e. u = 10.


By the theorem we have just seen, A = u  r = 10  6 = 60 cm2.

EXAMPLE 24 The sides of a triangle measure 7, 8 and 9 units. Find the radius of the incircle of this
triangle.

7+8+9
Solution u= =12
2

A = u( u  a)( u  b )( u  c) = 12(12  7)(12  8)(12  9) (Heron's Formula )

= 12  5  4  3 
12 5
Also, A = u  r = 12  r = 12ñ5, so r = ñ5.

Definition circumcircle, circumscribed circle, circumradius


The circle which passes through all three vertices of a triangle is called the circumcircle (or
circumscribed circle) of the triangle. The radius of the circumcircle is called the
circumradius (R).

185 Geometry 8
Theorem area of a triangle by its circumradius
The area of a triangle is equal to the ratio of
A
the product of the sides to four times its
circumradius: in the figure,
c b
abc .
A( ABC ) =
4R R
B a C

a b c
Proof The law of sines tells us that = = = 2 R.
sin A sin B sin C
a
So a = 2R  sin A, i.e. sin A = .
2R
1 1 a abc
Substituting this in A( ABC ) =  b  c  sin A gives us A( ABC ) =  b  c  = , as
2 2 2R 4R
required.

EXAMPLE 25 The sides of a triangle measure 8 cm, 10 cm and 12 cm. Find the circumradius R of this
triangle.

8+10+12
Solution Let u = =15, then by Heron’s Formula the area of the triangle will be
2
A = 15  (15  8)  (15  10)  (15  12) = 15  7  5  3 =15 7 cm 2.

abc 8  10  12 16 7
Using the formula A( ABC ) = gives us 15 7 = , i.e. R = cm.
4R 4R 7

EXAMPLE 26 The sides of a right triangle measure 6 cm and 8 cm. Find the sum of the circumradius and
the inradius of this triangle.

Solution From geometry we know that if the vertices of a right triangle all lie on the same circle then
the hypotenuse is the diameter of this circle. So the hypotenuse is the diameter of the
circumcircle (2R). By the Pythagorean Theorem, 62 + 82 = (2R)2. So 2R = 10, i.e. R = 5 cm.
ac 68
Also, if the triangle is a right triangle then its area is A = = = 24 cm 2 .
2 2
a + b + c 6+8+10
Let u = = =12, then since A = u  r we have 24 = 12  r, i.e. r = 2 cm.
2 2
So the sum of the circumradius and inradius is R + r = 5 + 2 = 7 cm.

Areas of Quadrilaterals 186


EXAMPLE 27 A circle has radius 8 cm. Find the area of the equilateral triangle whose vertices lie on this
circle.

Solution We can write the area of the equilateral triangle in two ways:
a2 3 aaa
A= and A = .
4 4R
a2 3 a  a  a
If we equate these expressions using R = 8 cm, we get = and a = 8ñ3.
4 48
a2 3 (8 3)2  3 192 3
So A = = = = 48 3 cm 2.
4 4 4

Check Yourself
1. The legs of a right triangle measure 5 cm and 12 cm. Find the inradius of this triangle.
2. In ABC, m(A) = 90° and AB = 3ñ2. If m(C) = 45°, find the circumradius of ABC.
3. The sides of a triangle measure 12, 9 and 7 units. Find the lengths R and r of the
circumradius and inradius of this triangle.
4. One of the legs of a right triangle measures 9 units and the diameter of its circumcircle
is 15. Find the inradius of this triangle.
5. An equilateral triangle has side length 6. Find the sum of the inradius and
circumradius of this triangle.
6. The base of an isosceles triangle measures 24 units and the other sides are each 13 units
long. Find the inradius r and circumradius R of this triangle.
Answers
27 5 12 169
1. 2 cm 2. 3 
3. R  ,r 5 4. 3 5. 3ñ3 
6. r  ,R
10 5 10

C. PROPERTIES OF THE AREA OF A TRIANGLE


So far we have learned several formulas for the area of a triangle. However, sometimes these
formulas may not be enough to solve a problem, or using them can make the solution longer.
In this section we look at some properties of triangles that can help us to solve problems more
directly.

187 Geometry 8
Property 1
In a right triangle, the product of the legs is A
equal to the product of the hypotenuse and
the length of the altitude to the hypotenuse: b
c ha
in the figure,

a  ha = b  c . B H C

a  ha bc
Proof We can write the area of a right triangle in two ways: A( ABC ) = and A( ABC ) = .
2 2
Equating and simplifying gives us a  ha = b  c.

EXAMPLE 28 The legs of a right triangle measure 7 cm and 24 cm. Find the height drawn to the
hypotenuse of this triangle.

Solution Let the hypotenuse be a. By the Pythagorean Theorem we have


a2 = 72 + 242 = 49 + 576 = 625, i.e. a = 25 cm.
168
Now using a  ha = b  c gives us 25  ha = 7  24, i.e. ha = cm.
25
As an exercise, try solving this problem using only the formulas we studied in the previous
section. Can you do it?

Property 2
If the base lengths and heights of two triangles are the same then their areas are equal.

base1  height1
Proof Let the area of the first triangle be A1 = , and the second area be
2
base1  height1
A2 = (because the bases and heights are equal).
2
Then A1 = A2, as required.

EXAMPLE 29 In the figure, A

A(ABC) = 112 cm2,


DE = 12 cm and 8
AH = 8 cm are given.
Find BD = EC = x.
B x D H E x C

12

Areas of Quadrilaterals 188


Solution In the figure, AH = ha = 8 cm is the common height of ABD, ADE and AEC.
The base lengths BD = EC = x are the same and their heights are also equal,
so by Property 2 we have
A(ABD) = A(AEC) = S.
DE  AH 12  8
Also, A( ADE) = = = 48 cm 2 .
2 2
From the figure we can say
A(ABC) = A(ABD) + A(ADE) + A(AEC)
112 = S + 48 + S
2S = 64, S = 32 cm2.
x  ha
Now, since A( ABD) = S = 32 = , we have 64 = 8  x, i.e. x = 8 cm.
2

Property 3
A median of a triangle divides the area of A
the triangle into two equal parts: in the
figure, if BD = DC then
A(BAD) = A(DAC).

B D H C

Proof Let AD be the median and AH be the altitude, as in the figure. So BD = DC.
We can say that AH is the altitude of both of the triangles ABD and ADC, so
BD  AH DC  AH
A( ABD ) = and A( ADC) = .
2 2
BD  AH DC  AH
But since BD = DC we can write = , which means A(ABD) = A(ADC).
2 2

Conclusion
A
By applying Property 3 repeatedly, we can see that a
triangle can be divided into equal parts by taking
successive medians.
S1 S2 S3 S4
For example, if BD = DE = EF = FC in the figure
opposite then S1 = S2 = S3 = S4. B D E F C

189 Geometry 8
EXAMPLE 30 The figure shows a triangle ABC. BD is the A
median of side AC in ABC, DE is the
median of side BC in BCD, and EF is the
median of side BD in BDE. Given that D
A(DEF) = 5 cm2, find A(ABC).
F

B E C
Solution In BDE, BF = FD so
A(BEF) = A(DEF) = 5 cm2 and so A(BDE) = 10 cm2.
In BCD, BE = EC so A(CDE) = A(BDE) = 10 cm2 and so A(BCD) = 20 cm2.
In ABC, AD = DC so A(ABD) = A(BCD) = 20 cm2 and so A(ABC) = 40 cm2.

Property 4
If the heights of two triangles are the same
A
then the ratio of their areas is the same as
the ratio of their base lengths: in the figure,
A( ABD ) m
= .
A( ADC ) n

B m D n C

Proof Triangles ABD and ADC in the figure both have common altitude AH = ha.
AH  BD
A( ABD ) 2
So = .
A( ADC ) AH  DC
2
A( ABD) BD m
Canceling common terms gives us = = .
A( ADC ) DC n

Note
We can use the letter S to mean the common multiplier in triangle ratio problems which use
this property: A(ABD) = mS and A(ADC) = nS.

Areas of Quadrilaterals 190


EXAMPLE 31 In the figure, A(ABC) = 70 cm2, A
BD = 4 cm and DC = 10 cm are given.
Find A(ABD).

Solution By Property 4 we can write


B 4 D 10 C
A(ABD) = 4S and A(ADC) = 10S. Since
A(ABC) = A(ABD) + A(ADC), we have 70 = 4S + 10S, i.e. 14S = 70 and so S = 5 cm2.
So A(ABD) = 4S = 4  5 = 20 cm2.

EXAMPLE 32 In the figure, AE = 5 cm, EB = 3 cm, A


BD = 6 cm and DC = 5 cm. The area of
5
AED is 15 cm2. What is the area of ABC?
E
3
Solution In ABD, AE = 5 and EB = 3 so we can write
A(AED) = 5S and A(EBD) = 3S. B 6 D 5 C

So A(AED) = 5S = 15, i.e. S = 3 cm2.


So A(ABD) = A(AED) + A(EBD) = 5S + 3S = 8S = 8  3 = 24 cm2.
Since BD = 6 and DC = 5 we can write A(ABD) = 6X and A(ADC) = 5X.
So A(ABD) = 6X = 24, i.e. X = 4 cm2.
So A(ABC) = A(ABD) + A(ADC) = 6X + 5X = 11X = 11  4 = 44 cm2.

EXAMPLE 33 In the figure, ABCD is a rectangle.


D E C
DE = EC, AF = FG = GB and
A(EFG) = 6 cm2 are given.
Find A(ABCD).

A F G B
Solution Let us draw EA and EB. By Property 4,
A(AEF) = A(EFG) = A(EGB) = 6 cm2.
So A(EAB) = 6 + 6 + 6 = 18 cm2.
A( ABCD )
We also know that A( EAB) = by the properties of a rectangle.
2
So A(ABCD) = 2  A(EAB) = 2  18 = 36 cm2.
191 Geometry 8
EXAMPLE 34 In the figure, ABCD is a rectangle. DC is
D E F G H C
divided into five equal parts and AB is
divided into four equal parts. Given that
A(ABCD) = 180 cm2, find A(IJGF).

Solution Let us divide IJGF into two triangles and find A I J K B


the area of each triangle. We know that if we
connect any point on one side of a rectangle to the two non-adjacent vertices then the area of
the triangle formed will be half the area of the rectangle. Let us draw the lines GI, DI and CI.
A( ABCD ) 180
By the properties of a rectangle we have A( DIC ) = = = 90 cm 2.
2 2
The base DC of DIC is divided into five equal parts, so by Property 4 we can write
A(DIC) = 5S = 90 cm2.
90
So A(IFG) = S = = 18 cm2.
5
In the same way we can draw GA and GB to get
A( ABCD ) 180
A( ABG) =   90 cm 2.
2 2
Since AB is divided into four equal parts, we have
90 45
A( IGJ ) =  cm 2 .
4 2
45 81
Finally, A(IJGF) = A(IFG) + A(IGJ) = 18 + = cm2.
2 2

Property 5
If the bases of two triangles are the same A
then the ratio of their areas is the same as D
the ratio of their heights: in the figure,
A( ABC ) h1 h1
= . h2
A( DBC ) h2

B H K C

Proof BC is a common base, so


BC  h1
A( ABC ) 2 .
=
A( DBC ) BC  h2
2
A( ABC ) h1
Canceling common terms gives us  , as required.
A( DBC ) h2

Areas of Quadrilaterals 192


EXAMPLE 35 In the figure, BC = 8 cm and A
DH = 4 cm.
Given that A(ABDC) = 20 cm2, find AD.

Solution Let AD = x, then AH = x + 4.


4
We have A(ABC) = A(ABDC) + A(DBC).
B H C
A( ABC ) AH
By Property 5, = , so
A( DBC ) DH
48
20+
2 = x+ 4 ,
48 4
2
36 x + 4 9 x+ 4
= , i.e. = , x + 4 = 9, x = 5 cm. So AD = 5 cm.
16 4 4 4

Check Yourself
1. The legs of a right triangle measure 5 cm and 12 cm. Find the length of the altitude to
the hypotenuse of this triangle.
2. Two parallel lines are given. Two points A and B lie on one of the lines and points C, D and
E lie on the other line. What can you say about A(ABC), A(ABD) and A(ABE)?

3. In the figure, ABC is an isosceles triangle with A

AB = AC. Given that AD = DE = EF = FG = GC D

A( FGH ) E
and AH is the altitude to side BC, find .
A( ABC ) F

B H C

4. In the figure, BC is divided into four equal parts and A


AC is divided into three equal parts.
G
If A(EGH) = 4 cm2, find A(ABC).
H

B D E F C

193 Geometry 8
5. In the figure, ABCD is a rectangle. AB is divided into D H I C
four equal parts and DC is divided into three equal
parts. If A(ABCD) = 120 cm2, find A(EGIH).

A E F G B
Answers
60 1
1. cm 2. they are equal 3. 4. 24 cm2 5. 50 cm2
13 10

Property 6
The ratio of the areas of the two triangles A
formed by the bisector of an angle in a
triangle is the same as the ratio of the
lengths of the two sides separated by the
b
bisector: in the figure, c

A( ABN ) c
= .
A( ANC ) b
B N C
Proof In the figure, let us draw perpendiculars from
the point N to the sides AB and AC and let the intersection points of these perpendiculars and
the sides be D and E respectively.
Since a point on the bisector of an angle is the same distance from the two sides of the angle,
we can write ND = NE = x.
AB  ND
A( ABN ) 2 AB  x AB c
So = = = = , as required.
A( ANC ) AC  NE AC  x AC b
2

EXAMPLE 36 In the figure, CN is the bisector of C. A


Given that AC = 4 cm,
N
BC = 9 cm and 4
A(ANC) = 12 cm2,
find A(ABC).
B 9 C
A( ANC ) AC
Solution By Property 6 we have = , so
A( BNC ) BC
12 4
= and so A(BNC) = 27 cm2.
A( BNC ) 9

So A(ABC) = A(ANC) + A(BNC) = 12 + 27 = 39 cm2.

Areas of Quadrilaterals 194


Property 7
The medians of a triangle together divide
A
the area of the triangle into six equal parts.

S S
F E
G
S S
S S

B D C
Proof Look at the figure. Let D, E and F be
endpoints of the medians to sides BC, AC and AB, respectively. So G is the centroid of ABC.
If AD is a median then by the properties of a centroid we can write GD = x and GA = 2x.
The centroid of a triangle
divides each median in Then by Property 4 we have
BGA, the ratio 1:2:
A(BDG) = S and A(BGA) = 2S.
A
In BGA, GF is a median so

2x E A(AFG) = A(BFG) = S.
So we have
x
B D C
A(AFG) = A(BFG) = A(BDG) = S. (1)
Since AD is a median, we have A(ABD) = A(ADC).
By similar reasoning to the above we can get
A(AGE) = A(EGC) = A(GDC) = S. (2)
Combining (1) and (2) shows that all six areas are equal to each other.

EXAMPLE 37 In the figure, G is the centroid of ABC. A


Given that BD = DE = EC and
A(GEC) = 8 cm2, find A(ABC). G 4

B D E C
Solution By Property 4,
A(GBD) = A(GDE) = A(GEC) = 8 cm2.
So A(BGC) = A(GBD) + A(GDE) + A(GEC) = 8 + 8 + 8 = 24 cm2.
By Property 7, A(BGC) = 2S = 24 so S = 12 and A(ABC) = 6S.
So A(ABC) = 6S = 6  12 = 72 cm2.

195 Geometry 8
Property 8
Let ABC be a triangle with inradius r and altitudes, ha, hb and hc. Then
1 1 1 1
= + + .
r ha hb hc

a  ha b  hb c  hc 2A 2A 2A
Proof We know A( ABC ) = A = = = , so we have ha = , hb = and hc = .
2 2 2 a b c
1 a 1 b 1 c
Rearranging these gives us = , = and  .
ha 2 A hb 2 A hc 2 A
If we add these terms we have
1 1 1 a b c a + b + c 2u u
+ + = + + = = = .
ha hb hc 2 A 2 A 2 A 2A 2A A

u u 1 1 1 1 1
Since A = u  r we can write = = . So = + + , as required.
A ur r r ha hb hc

EXAMPLE 38 The lengths of the altitudes of a triangle are 4 cm, 6 cm and 8 cm. What is the inradius of
this triangle?

1 1 1 1
Solution By Property 8 we have
= + + .
r ha hb hc
1 1 1 1 6+ 4+ 3 13 24
So = + + = = , i.e. r = cm.
r 4 6 8 24 24 13

Property 9
If two triangles are similar then the ratio of their areas is equal to the square of the ratio of
similarity.

Proof Look at the figure. Let ABC  DEF.


A D
a h
Then = k and a = k, where k is the ratio
d hd
of similarity. ha hd

a  ha
A( ABC ) a  ha a ha B H C E K F
So = 2 = = 
A( DEF ) d  hd d  hd d hd a d
2
 k  k = k2 , as required.

Areas of Quadrilaterals 196


EXAMPLE 39 In the triangle opposite, D and E are the A

midpoints of sides AB and AC


A( ADE)
respectively. Find A( ABC ) .
D E

Solution If D and E are the midpoints of the sides then


B C
DE BC and so ADE  ABC.
AD 1
The ratio of similarity is k = = . (D is the midpoint of AB)
AB 2
A( ADE) 1 1
So by Property 9, = k2 = ( ) 2 = .
A( ABC ) 2 4

Rule
When we connect the midpoints of the
A
sides of a triangle, the area of the triangle is
divided into four equal parts: in the figure,
S
F E
A(AFE) = A(BDF) = A(DEC) = A(DEF) = S. S
S S

B D C

Rule
If we divide two sides of a triangle into equal
A
lengths and connect the dividing points with
parallel lines, the areas of the parts are D
S
E
proportional to the numbers S, 3S, 5S, 7S... . 3S
F G
5S
H I
7S
J K
9S
.
B . C
.

197 Geometry 8
EXAMPLE 40 In the figure, the sides AB and BC are each A
divided into four equal parts.
F
Given that A(AFIC) = 35 cm2,
E
find A(ABC).
D
Solution By the previous rule,
A(ABC) = S + 3S + 5S + 7S = 16S. B G H I C

Also, A(AFIC) = 7S = 35, so S = 5.


As a result, A(ABC) = 16S = 16  5 = 80 cm2.

Check Yourself
A
1. In the figure, AD is an angle bisector,
AB = 8 cm, AE = 7 cm, DE = 4 cm and
A(ABC) = 36 cm2 are given. 7
8
Find DC if DE  AC.
E
4

B D x C

2. A triangle ABC has centroid G, and A(AGC) = 8 cm2. Find A(ABC).


A
3. In the figure, G is the centroid of ABC. Given that G1
is the centroid of ADC and the area of DGG1 is
3, find A(ABC). G
G1

B D C

4. The legs of a right triangle measure 6 cm and 8 cm. Find the inradius of this triangle.

A
5. In the figure, G is the centroid of ABC.
Given that E is the midpoint of AC,
FE BC and A(EFG) = 5, find A(ABC). F E
G

B D C
6. Two points D and E lie respectively on sides AB and AC of a triangle ABC. Given that
DE BC, AE = 2 cm, EC = 3 cm and A(BCED) = 42 cm2, find A(ABC).
Answers
1. 5 cm 2. 24 cm2 3. 54 4. 2 cm 5. 120 6. 50 cm2
Areas of Quadrilaterals 198
EXAMPLE 41 In the figure, AB is divided into four equal A
parts and BC is divided into five equal parts. F
If A(ABC) = 180 cm2, find A(DHIF). E

D
Solution First let us draw FH and AH, then we can use
Property 4. In ABC the side BC is divided
B G H I J C
into five equal parts, so
180
A( ABH ) = 2  = 72 cm 2 .
5
72
In ABH, AB is divided into four equal parts, so A( DFH ) = 2  = 36 cm 2 .
4
Now let us draw FC.
180
In ABC side AB is divided into four equal parts, so A( BCF ) = 3  =135 cm 2 .
4
135
In BCF side BC is divided into five equal parts, so A( FHI ) = = 27 cm 2 .
5
Finally, A(DHIF) = A(DFH) + A(FHI) = 36 + 27 = 63 cm2.

EXAMPLE 42 In the figure, ABCD is a rectangle. Given that


D C
CE  BD, BE = 9 cm and DE = 16 cm,
16
find A(ABCD).

Solution By using the metric relations in a right E 9


triangle in CDB we have
A B
M etric relations in a CE2 = DE  BE, i.e.
right triangle: CE2 = 16  9 and CE = 12 cm.
A
A( ABCD )
b Also, A( BDC ) = .
c 2
h
12  (16+9)
So A(ABCD) = 2  A(BDC) = 2  = 300 cm2.
B p H q C 2
1. h2 =p× q
2. c2 =p× a
3. b2 = q × a

EXAMPLE 43 Prove that A(ABC) = 2R2 sin A  sin B  sin C, where R is the circumradius of ABC.

a b c
Solution By the law of sines,    2 R , i.e. a = 2R sin A, b = 2R sin B and c = 2R sin C.
sin A sin B sin C
a  b  c 2R sin A  2 R sin B  2 R sin C
So A( ABC ) = = = 2R2sin A  sin B  sin C, as required.
4R 4R

199 Geometry 8
EXAMPLE 44 In the figure, ABC and CDE are two
E
triangles and mD = 90°. Find A(ABC).

5
Solution By the Pythagorean Theorem in CDE,
A 8
CE2 = 52 + 122 = 169, i.e. CE = 13. C 12 D
ACB and DCE are vertical angles so they
13
are equal.
5 B
So sin(ACB) = sin(DCE) = .
13
Finally, by the trigonometric formula for the area of a triangle we can write
1 1 5
A( ABC ) =  AC  BC  sin( ACB) = 8 13  = 20.
2 2 13

EXAMPLE 45 In the figure, ABC is a right triangle with A


mC = 90°. Given that AD is an angle
bisector, AB = 6 cm and DC = 3 cm,
find A(ABD).
6
Solution Let AC = h and BD = x.
By using the bisector theorem we have

B isector theorem: AB AC 6 h B D 3 C
= , i.e.  , x 
h 18.
A BD DC x 3

BD  AC x  h 18
c
Finally, A( ABD) = = = = 9 cm 2.
na b 2 2 2

B m N n C

1. c = b
m n
2. na = (b × c) – (m × n)

EXAMPLE 46 In the figure, ABC is a right triangle. A


Given that A(ABD) = 1 cm2,
A(ADC) = 9 cm2 and AD  BC, find the x
length AC = x. h

B m D n C

Areas of Quadrilaterals 200


Solution Let AD = h, BD = m and DC = n.
By the metric relations in a right triangle we have h2 = m  n.
mh nh
Also, A(ABD) = 1 = and A(ADC) = 9 = ,
2 2
which give us m  h = 2 and n  h = 18.
Multiplying these equations gives us (m  h)  (n  h) = 2  18, i.e. m  n  h2 = 36.
But we know h2 = m  n, so h2  h2 = 36, h = 6 cm.
18
Using this in n  h = 18 gives us n  = 3 6 cm.
6
Finally, the Pythagorean Theorem in ADC gives us h2 + n2 = x2, i.e. (ñ6)2 + (3ñ6)2 = x2,
which means x2 = 6 + 54 = 60. So x = ò60 = 2ò15 cm.

EXAMPLE 47 In the figure, AF = 4 cm, A


FB = 3 cm, BD = 4 cm, 3
DC = 2 cm, CE = 5 cm and 4 E
EA = 3 cm. F
5
A( DEF )
Find . 3
A( ABC )
B 4 D 2 C
Solution We can write
A(  DEF ) A(  ABC )  A(  AFE)  A(  BDF )  A( CDE )
=
A(  ABC ) A(  ABC )

A(  ABC ) A(  AFE) A(  BDF ) A( CDE)


=   
A(  ABC ) A(  ABC ) A(  ABC ) A(  ABC )

A(  AFE) A(  BDF ) A(  CDE)


=1 –   .
A(  ABC ) A(  ABC ) A(  ABC )
By the trigonometric formula for the area of a triangle,
1
 AF  AE  sin A
A( AFE) 2 AF  AE 4  3 3
=  = = .
A( ABC ) 1 AB  AC 7  8 14
 AB  AC  sin A
2
A( BDF ) BD  BF 4  3 2
Similarly, = = = and
A( ABC ) BC  BA 6  7 7
A( CDE) CE  CD 5  2 5
   .
A( ABC ) CA  CB 8  6 24
A( DEF ) 3 2 5 119 49 7
So =1– – – =1 – = = .
A( ABC ) 14 7 24 168 168 24

201 Geometry 8
EXAMPLE 48 The hypotenuse of a right triangle measures 20 cm and its two acute angles are 15° and 75°.
What is the area of this triangle?

Solution Look at the figure. ABC is the right triangle.


15° A
Let us draw the median AD to the hypotenuse.
By the property of a median to the hypotenuse, 10 5
The median to the 30°
AD = BD = CD = 10 cm. If AD = BD then 15°
hypotenuse of a right
B D H C
triangle is half the length m(ABD) = m(BAD) = 15° , which means
of the hypotenuse. 10 10
A m(ADC) = 30°.
Now let us draw the altitude AH to the hypotenuse. In ADH, m(ADH) = 30° and
m(AHD) = 90°.
AD 10
B D C By the properties of a 30°-60°-90° triangle in AHD, AH = = = 5 cm.
2 2
AH  BC 5  20
So A(ABC) = = = 50 cm2.
2 2

EXAMPLE 49 In the figure, DE BC, D is the midpoint of


A1
AB and L is the midpoint of BC. F
Given that AF = 1 cm, FE = 3 cm and 3

A(EKLC) = 40 cm2, find A(ABC). D E


K

Solution If DE BC and D is the midpoint of AB then


E must be the midpoint of AC. So EC = 4 cm. B L C

In the figure, KE LC so FKE and FLC are


similar triangles. By Property 9,
A( FKE) FE 2 3 9 A( FKE) 9
=( ) = ( )2 = . So = , i.e. A(FKE) = 9 cm2.
A( FLC ) FC 7 49 A( FKE)+ 40 49
Now let us draw AL. In ALC, FC = 7 cm and AF = 1 cm. So A(FLC) = 7S and A(AFL) = S.
We know A(FLC) = A(FKE) + A(EKLC) = 9 + 40 = 49 = 7S, so A(AFL) = S = 7 cm2.
So A(ALC) = 8S = 56 cm2. L is the midpoint of BC, so A(ALC) = A(ABL) = 56 cm2.
So A(ABC) = A(ABL) + A(ALC) = 56 + 56 = 112 cm2.

EXAMPLE 50 ABC is a triangle with sides a = 6 cm, b = 7 cm and c = 5 cm. A circle is drawn which is
centered on AC and tangent to the sides AB and BC. Find the radius of this circle.

Solution The figure illustrates the problem. Let us draw the radii from point O to the points of
tangency on sides BC and AB. If D and E are the points of tangency then we can say that
OD  BC and OE  AB, since a line passing through the center of a circle is perpendicular
to any tangent line at the point of tangency.
Areas of Quadrilaterals 202
AB  OE BC  OD 5r 6r 11r
If we draw OB then A( ABC )  A( ABO )  A( BOC )  + = + = .
2 2 2 2 2

A
We can calculate A(ABC) by using Heron's
6+7+5
Formula with u = = 9:
2
E
O
A( ABC ) = 9  (9 – 6)  (9 – 7)  (9 – 5)

= 6 6 cm 2 .

B D C
11r 12 6
So A( ABC ) = = 6 6 and r  cm.
2 11

Check Yourself
1. In the figure, ABCD is a rectangle, AB = 8 cm, D C

AD = 6 cm and CH  BD. Find A(ABH).


6 H

A 8 B

2. In the figure, ABC is an isosceles triangle A

with AB = AC. Given that m(DAC) = 90°,


BD = 3 cm and DC = 5 cm, find A(ABD).

B 3 D 5 C

3. ABCD is a quadrilateral with m(A) = 90°. Given that AD = 12, AB = 16, BC = 20 and
CD = 24, find A(ABCD).
4. In the figure, AE = 5  EC, BC = 5  DC, A
m(B) = 30° and AB = 18 cm. 18
If A(ADE) = 15 cm2, find BD. E
30°
Answers B D C

216
1. cm2 2. 3 cm2 3. 288 4. 16 cm
25

203 Geometry 8
Throughout history, people have developed
different ways of finding the area of a polygonal
region. One of these people was Georg
Alexander Pick. He was born in Vienna in 1859
and died in 1943 in a concentration camp. His
famous theorem helps us to find the area of a
polygonal region whose vertices are points in a
regular square grid, such as the region shown
opposite. The distance between points in the
grid must be one unit.
Pick’s Theorem tells us that if I is the number
of grid points inside the polygon and B is the
number of points on its perimeter, the area is
B
Area I  1 .
2
In other words, the area is one less than the sum of the interior points and half of the points on
the boundary.
Let us look at a simple example. Look at the triangle above right. Its legs are four and five units
long, so using regular geometry we can say that
45
its area is
A  10 square units.
2
D Alternatively, using Pick’s Theorem with
I = 6 and B = 10 gives us
B 10
A  I  1 6   1  10.
E C 2 2
Now look at the figure on the left. Using our
knowledge of geometry, the combined area of
the triangle and rectangle is
A B
A Atriangle  Arectangle

42
  (4  2) 4  8
2
 12 square units.
You can check this with Pick’s Theorem using
I = 5 and B = 10.
Pick’s Theorem is especially useful for finding the
area of complicated shapes. Look at the shape on
the right. We could divide it into rectangles and
triangles and calculate their area, but this would
take a long time. So we can use Pick’s Theorem
with I = 31 and B = 15:
B 15
A  I   1  31   1  37.5 square units.
2 2

Of course, we cannot find the areas of all shapes


using Pick’s Theorem. Remember that the
vertices of a shape must all be points in a square
grid before we can use the theorem. So for
example, we cannot use it to calculate the area of an equilateral triangle, because the three
vertices of an equilateral triangle will never all lie at points on a square grid. We also know that
the area of an equilateral triangle involves square roots, and Pick's Theorem does not use square
roots. Therefore the theorem is only useful in some cases.
EXERCISES 3 .2
A. The Concept of Area 8. In the figure, ABCD is a D E 3 C
rectangle. Given that AE
1. A rectangle has perimeter 80 and one side is four
is the bisector of A, 6
times as long as the other side. Find the area of
AE = 6 and
this rectangle.
EC = 3, find the area of
A B
ABCD.
2. The diagonal of a rectangle measures 20 units. If
one of the sides is 12 units long, find the area of
this rectangle.
9. One side of a rectangle is three times as long as
the other side and its diagonal measures 6 units.
Find the area of this rectangle.
3. In the figure, ABCD is a D C
rectangle. Given that 30°

AC = 14 and 14 10. In the figure, ABCD is D C



m(ACD) = 30°, a rectangle.
find the area of this Given that
A B F
rectangle. CE  BD,
AE = 6 and
A 6 E 2 B
EB = 2,
4. The ratio of the sides of a rectangle is 3 : 5 and its find A(ABCD).
area is 240 square units. Find the perimeter of
the rectangle.
B. Area of a Triangle
11. In the figure, A D C
5. In the figure, ABCD is a D 8 C
AC = 12 and
rectangle. Given that
BD = 5. 5
CD = 8, DE = 5 and 5
Given that BD  AC,
EB = 4, find the area
find A(ABC).
of the rectangle. B
A E 4 B

6. A rectangle has area 72 unit2 and perimeter 34 12. A triangle ABC has sides a = 6 and b = 8.
units. Find the lengths of the sides of this rectangle. If ha = 10, find hb.

7. A rectangle has area 84 unit2. Given that one of 13. The base of an isosceles triangle is 10 units long
the sides is five units longer than the other side, and its other sides are each 13 units long. Find
find the perimeter of this rectangle. the area of this triangle.

Areas of Quadrilaterals 206


14. In ABC, BC = 8, AC = 6 and m(C) = 45°. Find 22. One of the acute angles in a right triangle
A(ABC). measures 22.5° and the length of the hypotenuse
is 12 units. Find the area of this triangle.
15. The sides of a triangle ABC are a = 13,
b = 14 and c = 15. 23. Find the area of the equilateral triangle with the
Given that A(ABC) = 84, find the lengths of given side length.
the three altitudes of the triangle. a. 12 b. 4ñ3 c. 3ñ2

16. A
24. Find the area of the equilateral triangle with the
given height.
a. 3ñ3 b. 4 c. 6ñ5
30°
B 22ñ3 C

In the figure, ABC is an isosceles triangle with 25. The altitude to the hypotenuse of a right triangle
AB = AC. Given that m(C) = 30° and the length divides the hypotenuse into two parts of lengths
of the base is BC = 22ñ3, find A(ABC). 4 and 9 units. Find the area of this triangle.

17. In a right triangle ABC the hypotenuse BC 26. Find the area of the triangle with the given side
is 17 units long and one of the legs is 15 units long. lengths.
Find A(ABC). a. 11, 12 and 15
b. 7, 9 and 10
18. The area of an isosceles right triangle is 16 square
units. Find the hypotenuse of this triangle.
27. The sides of a triangle are a = 12, b = 13 and
c = 15. Find hb.
19. The perimeter of a right triangle is 56 units and
its hypotenuse is 25 units. Find the area of this
triangle. 28. A

120° 10
8
20. A right triangle has hypotenuse AC = 10. If one of
the acute angles of this triangle is 30 find the B C
area of the triangle.
In the figure, m(A) = 120°. Given that
21. In the figure, ABCD is a AB = 8 and AC = 10, find A(ABC).
D C
rectangle.
Given that 29. Two adjacent sides of a triangle are 6 and 8 units
m(BED) = 120° and long. Find the area of this triangle if the angle
E
120°
AB = 10 cm, between these two sides is
A 10 B
find the area of CDE. a. 30°. b. 45°. c. 90°. d. 120°.

207 Geometry 8
30. In the figure, A 38. In the figure, A
5 
BD = 4, AC = 8,
E
DC = 7, BC = 10 and
8
BE = 6 and 6
m(C) = 60°.
EA = 5. Find the inradius r and
60°
If A(BDE) = 9, B 4 D 7 C circumradius R of B C
10
what is A(ABC)? ABC.

31. In the figure, A


1
E C. Properties of the Area of a Triangle
AE = 1,
EC = 5, 5 39. Find the length of the altitude to the hypotenuse
BD = 6 and
in a right triangle with legs 8 and 15 units long.
DC = 4.
B 6 D 4 C
A( ABC )
Find .
A( ABDE) 40. In the figure, A
2
A(ABC) = 85 cm .
Given that
32. The sides of a triangle are a = 10, b = 17 and 10
BD = 9,
c = 11. Find the value of sin C.
AH = 10 and B H D x E x C
DE = EC = x,
33. A triangle has perimeter 24 units. Given that the
find x.
area of this triangle is 60 square units, find its
inradius.
41. In the figure, G is A

34. The legs of a right triangle are 9 and 12 units the centroid of
long. Find the inradius and circumradius of this ABC. Given that
triangle. A(ABGC) = 20, G
find A(BCG).
35. The sides of an isosceles triangle measure 16, 10
and 10 units. Find the sum of its inradius and B C

circumradius.

42. In the figure, A


36. One side of an equilateral triangle is 8 units long. BD = 4,
Find the inradius r and circumradius R of this DE = 7 and
triangle. EC = 8.
Given that
B 4 D 7 E 8 C
37. The circumradius of a triangle is 8 units. Given A(ABC) = 95,
that a = 10, find the value of sin A. find A(ABD) + A(AEC).

Areas of Quadrilaterals 208


43. In the given figure, A 48. In the figure, A
4
BD = 12, DE BC.
E
DC = 5, Given that 5
CE = 6 and 6 AD = 5,
EA = 4. DB = 3 and D E
The area of B 12 D 5 C A(DBCE) = 12, 3
AED is 12. find A(ABC).
What is the area of ABD? B C

49. A
1 2
44. In the figure, BD A D E
bisects B, 2 4
DE  BC, ? F G
1 2
D I
DE = 4 and H
2
1
A(ABD) = 24.
B C
Find the length of B E C
AB. In the figure, AD = FH = HB = 1 cm,
DF = AE = GI = IC = 2 and EG = 4. Given that
A(DFGE) = 12, find A(ABC).
45. In the figure, I is A
50. In the figure, BC and A
the center of the
AC are divided into six I
incircle of the right
and four equal parts J
triangle ABC. 6
respectively. K
The legs of ABC I
are 6 and 8 units A(ABC) = 120 is
given. Find A(EGJI). B D E F G H C
long. Find A(AIC). B 8 C

51. The sides of a triangle measure 12, 14 and 16


46. In the figure, G is A units. Find the inradius r and circumradius R of
the centroid of this triangle.
ABC. D
AB = 4  AD and 52. In the figure, AD is a A
A(ABC) = 84 G median and E and F
are given. are the midpoints of N
F E
Find A(ADG). B C AC and AB
respectively.
If the sum of the B D C
47. The lengths of the altitudes of a triangle are 3, shaded areas is
4 and 6 units. Find the inradius of this triangle. 12, find A(ABC).

209 Geometry 8
53. In the figure, A 57. In the figure, ABC A
AF = FB = BD = 4, 
3 is a right triangle
4
DC = 5, E with m(C) = 90°.
EC = 7 and Given that CD is the D
F
AE = 3. 7 median of side AB, 2
4 E
A( ABC ) CD  AE,
Find . B C
A( DEF ) ED = 2 and
B 4 D 5 C
BC
AE = , find A(ABC).
2

58. In the figure, CD is the A



54. In the figure, point D A median of side AB.
15°
lies on the altitude Given that
D
AH. AB = 10,
Given that D
m(ABC) = 30° and
BC = 10 and m(BAC) = 15°, 30°
AD = 6, find A(ABC). B C
B H C
find A(ABDC).

59. In the figure, ABCD is a D M C



rectangle and points M
and N lie on the sides
CD and AD, respectively.
N
55. An equilateral triangle ABC is given. Point P lies Given that BMN is an

on the base BC such that m(APB) = 75°. If one equilateral triangle and A B
side of the triangle is 12 units long, find A(APC). A(DMN) = 12, find A(ABN) + A(BCM).

60. A point P is taken in the A



interior of a triangle ABC H
G
and through it three
P
lines are drawn parallel I F
56. P is a point in the interior of an equilateral
 to the sides of ABC, as
triangle such that the distances from P to the
in the figure. Prove that B D E C
vertices are 5, 12 and 13 units. Find the area of
this triangle. A( ABC ) = A( HIP ) + A( DEP ) + A( FGP ).

Areas of Quadrilaterals 210


In the previous section we studied the areas of rectangles and triangles. In this section we
will use what we have learned to begin our study of quadrilaterals. First we will study the area
of a general quadrilateral, and then we will look at the areas of special quadrilaterals such as
parallelograms and rhombi.

D. AREA OF A QUADRILATERAL
Rule
By drawing the diagonals of a polygon from D
one of its vertices, we can divide the area of
the polygon into small triangles and then use E
the areas of the triangles to calculate the area
of the given region. C

For example, in the figure, A

A(ABCDE) = A(ABC) + A(ACD) + A(ADE).

EXAMPLE 51 In the figure, AB = 5 cm, D


BC = 12 cm, CD = 14 cm and AD = 15 cm.
15
Find the area of the quadrilateral ABCD if
m(ABC) = 90°. A
14
5

B 12 C
Solution Let us join A and C to get two triangles,
ABC and ACD.
AB  BC 5  12
ABC is a right triangle, so A( ABC ) = = = 30 cm 2 .
2 2
Also, the Pythagorean Theorem in ABC gives
AC2 = 52 + 122 = 25 + 144 = 169, i.e. AC = 13 cm.

211 Geometry 8
13  14  15
Now let us use Heron’s Formula in ACD with u =  21:
2
ACB = 21(21 – 13)(21 – 14)(21 – 15) = 21  8  7  6 = 84 cm2.

So A(ABCD) = A(ABC) + A(ACD)


= 30 + 84
= 114 cm2.

Theorem area of a convex polygon


The area of a convex polygon is equal to half C
the product of the lengths of diagonals and
If all the interior the sine of the angle between the diagonals: D
angles of a polygon in the figure,
are smaller than K a
180° then the AC  BD  sin
polygon is a convex A( ABCD ) = .
polygon. 2
A B

Proof Let K be the intersection point of the C


diagonals.
D
180° – a
aK a
180° – a

From the figure, A B

A(ABCD) = A(ABK) + A(BCK) + A(CDK) + A(ADK). So by the trigonometric formula,


AK  BK  sin(180°  ) BK  CK  sin CK  DK  sin(180°  ) AK  DK  sin

A( ABCD )   
2 2 2 2
We know that sin = sin(180° – ), so we can reduce the above expression to
sin
A( ABCD )   [( AK  BK )  ( BK  CK )  (CK  DK ) ( AK  DK )]
2
sin
  [[( AK  CK )  BK ]  [( CK  AK )  DK )] ]
2
sin
  (( AK  CK )  ( BK  DK ))
2
AC  BD  sin
 . ( AK
 CK AC and BK
 DK BD )
2

Areas of Quadrilaterals 212


EXAMPLE 52 In the figure, AC = 12 cm, C
BD = 15 cm and m(AKB) = 150°.
D
Find the area of the quadrilateral ABCD. K

150°

A B

AC  BD  sin
Solution A( ABCD ) = , so
2
12  15  sin150°
A( ABCD ) =
2
1
6  15 
2
 45 cm 2 .

Note
If the diagonals of a quadrilateral are perpendicular to each other then the formula for its area
becomes
AC  BD
A( ABCD ) =
2

since sin 90° = 1.

EXAMPLE 53 The diagonals of a quadrilateral ABCD are perpendicular to each other with AC = 3  BD.
If the area of ABCD is 48 cm2, find the lengths of the diagonals.

Solution Let BD = x, so AC = 3x.


AC  BD x  3x
Using A( ABCD) = gives 48  .
2 2
So x2 = 32, x = 4ñ2. So the diagonals are BD = x = 4ñ2 cm and AC = 3x = 12ñ2 cm.
213 Geometry 8
Theorem
The diagonals of a quadrilateral ABCD divide
D C
the area of the quadrilateral into four parts as S3
shown in the figure. If S4
M S2

A(ABM) = S1, S1

A(BCM) = S2, A B

A(CDM) = S3 and

A(ADM) = S4, then


S1  S3 = S2  S4 .

Proof We know from Property 4 that if two triangles


are the same height then the ratio of their
areas is the same as the ratio of their bases.
So we can write
A( ABM ) S1 BM
  and
A( ADM ) S4 DM

A( BCM ) S2 BM
  .
A( DCM ) S3 DM

BM S1 S2
So   .
DM S4 S3
Cross multiplying gives us
S1  S3 = S2  S4, as required.

EXAMPLE 54 AC and BD are the diagonals of a quadrilateral ABCD, and M is their point of intersection. If
A(ABM) = 18 cm2, A(CDM) = 12 cm2 and A(BCM) = A(ADM), find A(ABCD).

Solution Let S1 = 18, S3 = 12, S2 = S4 = x.


By the previous theorem, S1  S3 = S2  S4, so
18  12 = x  x, x2 = 216 cm2, x = 6ñ6 cm2. So
A(ABCD) = S1 + S2 + S3 + S4 = 18 + 6ñ6 + 12 + 6ñ6 = (30 + 12ñ6) cm2.

Areas of Quadrilaterals 214


EXAMPLE 55 In the figure, ABCD is a quadrilateral and K, C
L and M are the midpoints of their respective
sides. Given that KL  LM, KL = 8 cm and D
M
LM = 10 cm, find the area of quadrilateral.
K

8 10

A L B

Solution Let us draw the diagonals AC and BD. In C


ABD points K and L are the midpoints so KL
is a midsegment. By the properties of a D
R M
A midsegment of a
midsegment,
triangle is a line K N
KL BD and BD = 2  KL = 2  8 = 16 cm.
segment that connects P
the midpoints of two Also, if KL BD then
sides of the triangle.
m(LNR) = m(KLN) = 90°. A L B

In ABC points L and M are midpoints so LM


is a midsegment, which gives us AC LM and
AC = 2  LM = 2  10 = 20 cm.
If AC LM then m(ARB) = m(LNR) = 90°.

As a result, AC  BD and so
AC  BD 16  20
A( ABCD ) =  =160 cm 2.
2 2

Check Yourself
1. In the quadrilateral ABCD, m(DAB) = m(BCD) = 90°. If AD = 12 cm, AB = 16 cm and
BC = 10 cm, find A(ABCD).
2. In a quadrilateral ABCD, point E is the intersection point of the diagonals and AC is
perpendicular to BD. Given that DE = AE = EC = 6 cm and AB = 10 cm, find the area
of ABCD.
3. AC and BD are the diagonals of a quadrilateral such that AC = 4  BD and the angle
between them is 30°. If the area of this quadrilateral is 160 unit2, find the lengths of AC
and BD.
4. The diagonals AC and BD of the convex quadrilateral ABCD intersect at point E.
A(ABE) = 12 cm2, A(CDE) = 16 cm2 and A(BCE) = 3  A(ADE) are given. Find
A(ABCD).
Answers
1. (96 + 50ñ3) cm2 2. 84 cm2 3. AC = 16ò10, BD = 4ò10 4. 60 cm2
215 Geometry 8
E. AREA OF A PARALLELOGRAM
Theorem area of a parallelogram
The area of a parallelogram is the product of
D C
the length of any base and the length of the
corresponding altitude: in the figure, hb
A parallelogram is
a quadrilateral ha
whose opposite
A(ABCD) = a  ha = b  hb . b
K
sides are congruent
and parallel to A H B
each other.
a

Proof If we draw the diagonal BD we get two congruent triangles, ABD  CDB. So
A(ABCD) = A(ABD) + A(CDB)
= 2  A(ABD)
1
=2  a  ha
2
= a  ha.
We can use similar reasoning to show A = b hb.

EXAMPLE 56 Two sides of a parallelogram measure 6 cm and


8 cm. The height from the shorter side is 12 cm.
Find the height from the longer side.

Solution Since this is a parallelogram,


A = a  ha = b  hb, i.e. 6  12 = 8  hb.
6  12
So hb = = 9 cm.
8

EXAMPLE 57 In the figure, ABCD is a parallelogram and D E 3 C


BE is perpendicular to DC.
Given that AB = 12 cm, EC = 3 cm and
BC = 5 cm, find A(ABCD). 5

A 12 B
Solution A(ABCD) = AB  ha = AB  BE.
We need to find BE.
By the Pythagorean Theorem in BEC we have BE2 + 32 = 52, BE = 4 cm.
So AB = 12 cm and BE = ha = 4 cm. So A(ABCD) = 12  4 = 48 cm2.

Areas of Quadrilaterals 216


EXAMPLE 58 ABCD is a parallelogram with sides AD = 8 cm and AB = 12 cm. Given that m(ABC) = 150°,
find the area of this parallelogram.

Solution If m(ABC) = 150° then m(A) = 30°. D C


Let us draw the altitude from the vertex D to
AB and let E be the foot of this altitude. 8

150°
In AED, AD = 8 cm so 30°
A E B
DE = 4 cm (since this is a 30°-60°-90° triangle).
12
So A(ABCD) = 12  4 = 48 cm2.

Theorem
If ABCD is a parallelogram with sides a and b separated by an angle A then
A(ABCD) = a  b  sin A.

Proof Let us write AB = CD = a and D a C

BC = AD = b.
Then we draw the altitude DE from vertex D b b
h
to side AB with length h.
h
From the figure, sin A = , i.e. A E B
b
h = b  sin A.
So A(ABCD) = a  h = a  b  sin A.

EXAMPLE 59 The sides of a parallelogram measure 6 cm and 8 cm. Find its area if its interior angles
measure 60° and 120°.

Solution We can use the theorem above with either sin 60° or sin 120°, since we know
3
sin 60° = sin 120° = .
2
3
So A(ABCD) = a  b  sin A = 6  8  sin 60° = 6  8  24 3 cm 2.
2
217 Geometry 8
Theorem
If P is any point inside a parallelogram
D C
ABCD then
A( PAB)  A( PCD )  A( PBC )  A( PDA )
P
A( ABCD )
 .
2
A B

Proof Let us draw the altitudes PE and PF from a


point P to sides AB and CD respectively. D F C
Then PE + PF = ha.
P
a  PE a  PF
Also, A( PAB)  A( PCD
) 
2 2
a  ( PE  PF ) A E B

2 a

a  ha

2
A( ABCD )
 .
2
A( ABCD )
In the same way we can prove that A( PBC )  A( PDA )  .
2

EXAMPLE 60 In the figure, P is a point inside the D C


parallelogram ABCD. Given that P
2
A(PAB) = 13 cm ,
A( PBC ) 2
A(PCD) = 12 cm2 and  ,
A( PDA ) 3
A B
find A(PBC), A(PDA) and A(ABCD).

A( PBC ) 2
Solution We are given  , so we can write A(PBC) = 2S and A(PDA) = 3S.
A( PDA ) 3
By the previous theorem,
A( ABCD )
A(PAB) + A(PCD) = A(PBC) + A(PDA) =
2
13 + 12 = 2S + 3S, i.e. 5S = 25 and S = 5.

Areas of Quadrilaterals 218


So A(PBC) = 2S = 2  5 = 10 cm2,
A(PDA) = 3S = 3  5 = 15 cm2 and
A(ABCD) = 2  (A(PAB) + A(PCD)) = 2  (13 + 12) = 2  25 = 50 cm2.

Properties 10
1. Any diagonal in a parallelogram divides it
into two equal parts. S S

S S

2. The two diagonals of a parallelogram


S
divide its area into four equal parts. S
S
S

3. Three lines drawn from any vertex of a D C


parallelogram to the opposite vertex and S
the midpoints of the two opposite sides
S F
divide the parallelogram into four equal S S
parts.
A E B

4. Any point E on any side of a D E C


parallelogram which is connected
to the two non-adjacent vertices
creates a triangle which has half area of
the parallelogram: in the figure,
A B
A( ABCD )
A( ABE)  .
2

5. Connecting the midpoints of opposite H C


D
sides of a parallelogram creates four
S S
congruent parallelograms. K
E G
S S

A F B

219 Geometry 8
EXAMPLE 61 In the figure, ABCD is a parallelogram and D C
E is the midpoint of side AB.
60°
Given that DE = 6 cm, AD = 8 cm and 8 6

m(ADE) = 60°, find the area of ABCD.


A E B

Solution By the trigonometric formula for the area of a triangle,


1 1 3
A( ADE) =  6  8  sin60° = 6 8  =12 3 cm 2.
2 2 2
By Property 10.3 we can now write
A(ABCD) = 4  A(ADE) = 4  12 3 = 48 3 cm2.

EXAMPLE 62 In the figure, ABCD is a parallelogram and E


D C
is a point on side AB.
Given that DE = 6 cm, CE = 7 cm and
6 7
m(DEC) = 90°, find the sum of the areas
of the shaded regions. A E B

A( ABCD )
Solution By Property 10.4, A( DEC ) = and so the sum of the areas of the shaded regions
2
A( ABCD )
is also .
2
A( ABCD ) 6 7
So the sum of the areas of the shaded regions is 
A( DEC )   21 cm 2.
2 2

EXAMPLE 63 In the figure, ABCD is a parallelogram. Given D G H I C


that AE = EF = FB, DG = GH = HI = IC and
A(ABCD) = 120 cm2, find the area of
quadrilateral EFIG.

A E F B
Solution Let us connect point G to A, B and F.
Then we can find A(EFIG) as the sum of A(EGF) and A(GFI).
A( ABCD ) 120
By Property 10.4 we have A( AGB) = = = 60 cm 2.
2 2
In AGB, the base AB is divided into three equal parts. So
A( AGB) 60
A( EGF ) =   20 cm 2.
3 3
Now let us connect point F to D and C.
Areas of Quadrilaterals 220
A( ABCD ) 120
By Property 10.4 we have A( DFC ) =  60 cm 2.
2 2
In DFC the base CD is divided into four equal parts and so
A( DFC ) 60
A( GFI ) = 2   30 cm 2.
4 2
So the sum of the shaded areas is A(EGF) + A(GFI) = 20 + 30 = 50 cm2.

EXAMPLE 64 In the figure, ABCD is a parallelogram. E is D C


the midpoint of AB and BD is a diagonal.
Given that A(BEF) = 8 cm2, find A(ABCD).
F

A E B
Solution If ABCD is a parallelogram then EB DC.
So m(FBE) = m(FDC), m(FEB) = m(FCD) and m(EFB) = m(DFC).
EB 1
So EFB  CFD and the ratio of similarity is k =  .
DC 2
1 FB 1
If k  then = .
2 DF 2
FB 1
Let us draw the segment DE. In DEB,  and A(BEF) = 8 cm2.
DF 2
So A(DEF) = 2  8 = 16 cm2 and A(DEB) = 8 + 16 = 24 cm2.
Finally, by Property 10.3 we can write A(ABCD) = 4  A(DEB) = 4  24 = 96 cm2.

EXAMPLE 65 ABCD is a parallelogram and E and F are the midpoints of sides AB and BC respectively. Given
that A(DEFC) = 30 cm2, find A(ABCD).
D C
Solution Let us draw the line segment DF.
By Property 10.3 we can write
F
A( ABCD )
A( DFC ) = = A( ADE).
4
A E B
By Property 10.5 we can write
A( ABCD )
A( ABCD ) A( ABCD ) A( ABCD ) 3
A( EBF )  4  , so A( DEF )   A( ABCD ).
2 8 2 8 8
3  A( ABCD ) A( ABCD ) 5  A( ABCD )
Now A( DEFC )  A( DEF )  A( DFC )    = 30 cm 2,
8 4 8
30  8

which gives A( ABCD )  48 cm 2.
5

221 Geometry 8
Check Yourself
1. Two sides of a parallelogram measure 14 cm and 18 cm. Given that the acute angles in this
parallelogram measure 45°, find the area of the parallelogram.

2. In the figure, ABCD is a parallelogram and D 13 C

m(DBC) = 90°.
Given that BC = 5 cm and DC = 13 cm, find 5

A(ABCD). A B

3. In the figure, ABCD is a parallelogram. D 9 C


Given that m(D) = 120°, m(BEC) = 30°, 120°

AE = 5 cm and DC = 9 cm, find A(ABCD).


30°
A 5 E B

4. In the figure, ABCD is a parallelogram and P is a D C


point inside it. The area of the parallelogram is
76 cm2. Find the sum of the areas of the P
shaded regions.
A B

5. In the figure, ABCD is a parallelogram and side D E F G H I C

DC is divided into six equal parts. Given that


A(ABCD) = 150 cm2, find A(BIG).

A B

6. In the figure, ABCD is a parallelogram. Side AD D C


is divided into five equal parts and side BC is E
K
divided into four equal parts. F
Given that A(ABCD) = 200 cm2, find A(GIKE). J
G

H I

A B

Areas of Quadrilaterals 222


7. In the figure, ABCD is a parallelogram, E is the D C
intersection point of its diagonals, and BF is
5 F
perpendicular to AC. Given that BF = 3 cm, E
3
DE = 5 cm and CF = 6 cm, find the area of the
parallelogram ABCD.
A B
Answers
1. 126ñ2 cm2 2. 60 cm2 3. 18ñ3 cm2 4. 38 cm2 5. 25 cm2 6. 90 cm2 7. 60 cm2

F. AREA OF A RHOMBUS
A rhombus is also a parallelogram, so it shares the rules and properties that we have seen for
parallelograms. It also has some additional properties.

Properties 11
If ABCD is a rhombus then the following statements are true.
1. The area is given by A(ABCD) = base × height:

A(ABCD) = a  ha .
A rhombus is a
quadrilateral with
four congruent
2. A(ABCD) = a2  sin A
sides and parallel 3. Since the diagonals of a rhombus are perpendicular to each other,
opposite sides.
AC  BD  sin90  AC  BD

A( ABCD )  .
2 2

EXAMPLE 66 A rhombus has sides of 6 cm. Given that its height is 4 cm, find the area of this rhombus.

Solution A(ABCD) = base × height = 6 × 4 = 24 cm2.

EXAMPLE 67 The diagonals of a rhombus measure 12 cm and 16 cm. Find the length of its altitude.

Solution Let a be the measure of one side of the D a C

rhombus. We know that the diagonals of a 8


6
rhombus bisect each other, so in the figure, E
a
DE = BE = 6 cm and AE = EC = 8 cm. a
8 6
By the Pythagorean Theorem,
62 + 82 = a2, i.e a = 10 cm. A a B
AC  BD 16  12

Also, A( ABCD )   96 cm 2.
2 2
Since A(ABCD) = a  ha we have 96 = 10  ha, i.e. ha = 9.6 cm.

223 Geometry 8
EXAMPLE 68 The acute angle of a rhombus measures 30° and its height is 6 cm. Find its area.

Solution Let a be the side length. In the figure, DE is


D C
the altitude and DE = 6 cm.
DE 1 6 6
sin 30° = , so = , i.e.
AD 2 a 30°
AD = a = 12 cm. A E B

So A(ABCD) = a  ha = 12  6 = 72 cm2.

EXAMPLE 69 The longer diagonal of a rhombus measures 8 cm and the obtuse angle of the rhombus
measures 120°. Find the area of this rhombus.

Solution Look at the figure. We know that the D C


diagonals of a rhombus are perpendicular to 120° 30°
4
each other and bisect each other. Also, the
diagonals bisect the vertex angles. 4 E

Consider the triangle DEC in the figure.


AC = 8 cm so EC = 4 cm, and by using A B
trigonometric ratios,

DE 3 DE 4 3 8 3
tan 30° = ,  , DE  cm and BD 2  DE  cm.
EC 3 4 3 3
8 3
8
AC  BD 3 = 32 3 cm 2 .
So by Property 11.3, A(ABCD) = 
2 2 3

EXAMPLE 70 In the figure, ABCD is a rhombus. Given D C


that BK and CK are angle bisectors and
A(BCK) = 8 cm2, find the area of the
K
rhombus.

Solution We know that A B


m(ABC) + m(BCD) = 180°, so
m(KBC) + m(BCK) = 90° and
m(BKC) = 90°.
We also know that the diagonals of a rhombus are perpendicular to each other, so K is the
intersection point of the diagonals.
A( ABCD )
By Property 10.2 we can write A( BCK ) = and so A(ABCD) = 4  8 = 32 cm2.
4
Areas of Quadrilaterals 224
71
D
EXAMPLE In the figure, ABCD is a rhombus.
Given that
F
AE = 3  EC, ñ3
A C
EF  DC, E
3  DF = 5  FC and EF = ñ3 cm, find the area
of the rhombus.
B

Solution AE = 3  EC so let EC = x and AE = 3x. D


3  DF = 5  FC so let DF = 5y and FC = 3y. 5y
F
Now let us draw the diagonal BD and mark 3y
ñ3
the intersection point O of the diagonals. A
x E x
C
2x O
AC = 4x so OC = 2x.
m(EFC) = 90° = m(DOC) and
B
OCD is a common angle, so
m(FEC) = m(ODC) and EFC  DOC.
EF EC FC x 3y
By similarity, = = , i.e = and so x2 = 12y2.
DO DC OC 8y 2x
By the Pythagorean Theorem in EFC,
(ñ3)2 + (3y)2 = x2 = 12y2, so y = 1 and x = 2ñ3 which gives us
OC = 4ñ3 cm, AC = 8ñ3 cm, DC = 8 cm.
By the Pythagorean Theorem in DOC,
DO2 + OC2 = DC2, so DO2 = 82 – (4ñ3)2 = 64 – 48 = 16, i.e. DO = 4 cm and BD = 8 cm.
AC  BD 8 3  8
Finally, A( ABCD) =   32 3 cm 2.
2 2

Check Yourself
1. One of the diagonals of a rhombus is the same length as one side of the rhombus. Find
the area of the rhombus if one side measures 8 cm.
2. The altitude BH is drawn from vertex B of a rhombus to side CD. Given that DH = 2 cm
and CH = 3 cm, find the area of this rhombus.
3. A rhombus has perimeter 80 cm. Given that one of the diagonals has length 24 cm, find
the area of the rhombus.
4. One side of a rhombus measures 18 cm. Given that an obtuse angle in the rhombus
measures 150°, find the area of the rhombus.

225 Geometry 8
5. The diagonals of a rhombus are 25 cm and 30 cm long. Find its area.
6. The lengths of the diagonals of a rhombus have ratio 3 : 4. Given that the area of this
rhombus is 216 unit2, find the lengths of the diagonals and one side of the rhombus.
D
2
7. In the figure, ABCD is a rhombus and A(ABCD) = 120 cm . E is
the midpoint of BC. Find A(ABF).
O
A C
F

Answers E

1. 32ñ3 cm 2
2. 20 cm 2
3. 384 cm 2
4. 162 cm 2
5. 375 cm 2

2 B
6. diagonals: 24, 18; side length: 15 7. 20 cm

G. AREA OF A SQUARE
A square is a special type of rectangle, so we D a C
can find its area by multiplying the lengths
e e
of its two adjacent sides: in the figure,
a a
A(ABCD) = a  a, i.e.

A(ABCD) = a2 .
A a B

The diagonals of a square are perpendicular


to each other. Also, if we say that the length of a diagonal is e then the area of the square is

e2
A( ABCD ) = .
2
A square is also a parallelogram, so it shares all the properties of a parallelogram.

EXAMPLE 72 Find the area of a square whose perimeter is 28 cm.

Solution The perimeter is 4  a = 28, so a = 7 cm. So the area is a2 = 72 = 49 cm2.

EXAMPLE 73 The length of the diagonal of a square is 12 cm. Find the area of this square.

e2 12 2 144
Solution If e is the length of the diagonal then Asquare = , i.e. A = = = 72 cm 2.
2 2 2

Areas of Quadrilaterals 226


EXAMPLE 74 A square has area 20 cm2. Find the length of its diagonal.

Solution If the area is a2 then 20 = a2 and so a = ò20 = 2ñ5 cm.


The length of the diagonal is therefore e = añ2 = 2ñ5  ñ2 = 2ò10 cm.

EXAMPLE 75 In the figure, ABCD is a square. D C


Given that AE = 3 cm and EB = 5 cm,
find the area of the shaded region.

Solution Since AB = AE + EB, we have A 3 E 5 B

a = 3 + 5 = 8 cm.
83
So the shaded region has area A(ABCD) – A(ADE) = 82   64  12  52 cm 2.
2

EXAMPLE 76 In the figure, ABCD is a square and AEC is D C


equilateral triangle. Given that
A(AEC) = 9ñ3 cm2, find the area of this
square.
A B
Solution Let one side of the square be a and let the E
diagonal AC = e.

e2 3 e2 3 2
The area A of an 
So A( AEC ) ,
i.e. 9 3 , e = 36. So e = 6 cm.
equilateral triangle 4 4
with side length a is
e2 62 36
a2 3 Since ABCD is a square, A( ABCD
)    18 cm 2.
A . 2 2 2
4

EXAMPLE 77 In the figure, ABCD is a square. D C

Given that DE = 4 cm and


4
m(AEC) = 120°, find the area of the
square. E 120°

A B

227 Geometry 8
Solution From the figure we can see m(DEC) = 60°.
Let one side of the square be a, so DC = a.
DC
Also, tan 60° = which gives us
DE
a
3 , a = 4ñ3 cm.
4
So A(ABCD) = a2 = (4ñ3)2 = 48 cm2.

EXAMPLE 78 In the figure, ABCD is a square. Side AB is D C


divided into three equal parts and side DE is
G
divided into four equal parts. Given that the
area of the shaded region is 3 cm2, find H

A(ABCD). I

A E J B
Solution Let us draw the line EC.
A(CHI) = 3 is given, so
A(DEC) = 4  3 = 12 cm2.
A( ABCD )
From the properties of a parallelogram we know that A( DEC )  .
2
So A(ABCD) = 2  12 = 24 cm2.

EXAMPLE 79 In the figure, ABCD is a square. D C


Given that AD = EC, EB = 8 cm and
m(AEB) = 90°, find A(EBC).
E
8

A B
Solution We know that EC = AD = BC, so EBC is an
isosceles triangle. Let us draw the altitude CH in EBC.
Since EBC is isosceles, CH is also a median so EH = HB = 4 cm.
Let m(HBC) = x, then m(EBA) = m(BCH) = 90° – x and m(EAB) = x.
m(CHB) = m(AEB) = 90° and AB = BC, so by the ASA Congruence Theorem we can say
AEB  BHC and CH = EB = 8 cm.
EB  CH 8  8
So A( EBC )   32 cm 2.
2 2

Areas of Quadrilaterals 228


Check Yourself 12
1. A square has area x cm2 and perimeter x cm. What is the value of x?
2. The perimeter of a square is 8 cm. Find the area of this square.
3. In the figure, ABCD and EFGH are two D C
squares. Given that sum of the perimeters H
of these squares is 36 cm and area of the
E
shaded region is 27 cm2, find the areas of G
the two squares.
F
A B
4. The diagonal of a square is 8 cm long. Each side of the square is extended by 1 cm. By
how much does the area of square increase?

5. The perimeter of a square ABCD and the perimeter of an equilateral triangle EFG are
A( EFG )
equal. Find .
A( ABCD )

D G C
6. In the figure, ABCD is a square with side length 12 cm.
The side AB is divided into four equal parts, BC is
H
divided into two equal parts, and DC and AD are each F
divided into three equal parts. Find the area of the
shaded region.
A E B
Answers
4 3
1. 16 2. 4 cm2 3. 36 cm2, 9 cm2 4. (1 + 8ñ2) cm2 5. 6. 77 cm2
9

H. AREA OF A TRAPEZOID
Theorem area of a trapezoid
The area of a trapezoid is the product of the c
D C
height and half the sum of the bases: in the
A trapezoid is a figure,
quadrilateral with h
two parallel sides.
ac .

A( ABCD ) h
2 A H B
a

229 Geometry 8
Proof Look at the figure. Let us draw the altitudes D c C H

DK and BH.
We know AB DC, so DK = BH = h. h h

Now we draw the diagonal BD.


A K B
So A( ABCD)  A( ABD)  A( BCD )
a
AB  DK DC  BH
 
2 2
ah ch
 
2 2
( a  c)  h
 , as required.
2

EXAMPLE 80 In the figure, ABCD is a trapezoid. D 3 C


AB = 13 cm, DC = 3 cm and CH = 6 cm are
given. Find the area of this trapezoid. 6

A H B

Solution We are given a = 13, c = 3 and h = 6. 13

By the formula for the area of a trapezoid,


ac 13  3

A( ABCD )  h , i.e. A( ABCD )   6  8  6  48 cm 2.
2 2

Note
We know that the line which connects the midpoints of the legs of a trapezoid is called a
ac
median, and the length of the median is = .
2
ac
So A(ABCD) =  h = median × height.
2

EXAMPLE 81 The median of a trapezoid measures 10 cm and the height is 14 cm. What is the area of this
trapezoid?

Solution Area = median × height = 10  14 = 140 cm2.

Areas of Quadrilaterals 230


EXAMPLE 82 In the figure, ABCD is a trapezoid. Given that D 8 C
BC = 12 cm, DC = 8 cm, m(DAB) = 45°
12
and m(ABC) = 30°, find the area of ABCD.
45° 30°
A B

Solution First we draw the altitudes DH and CK. 8


D C
In CKB, BC = 12 cm so CK = h = 6 cm and
12
KB = 6ñ3 cm. (30°-60°-90° triangle) 45° h h

In ADH, m(DAH) = 45° and AH  DH so 45° 30°


A H 8 K B
m(ADH) = 45°.
So AH = DH and we know DH = CK = 6 cm.
We also know that DC = HK = 8 cm.
So a = AH + HK + KB = 6 + 8 + 6ñ3 = (14 + 6ñ3) cm, and
c = DC = 8 cm.
ac 14  6 3  8
Finally, A( ABCD )  h  6 (22  6 3)  3 (66 18 3) cm 2.
2 2

EXAMPLE 83 In the figure, ABCD is an isosceles trapezoid.


D 12 C
AB = 24 cm, BC = AD = 10 cm and
DC = 12 cm are given. 10 10
An isosceles trapezoid is a Find the area of this trapezoid.
trapezoid with congruent
legs. A 24 B

Solution First we draw the altitudes DH and CK to


D 12 C
base AB.
So HK = DC = 12 cm. 10 10

Since ABCD is isosceles, AH = KB = x.


A H 12 K B
But AB = AH + HK + KB, which gives
24 = x + 12 + x, x = 6 cm.
In CKB, BC = 10 cm and KB = 6 cm. By the Pythagorean Theorem,
CK2 + KB2 = BC2, h2 + 62 = 102, h = 8 cm.
ac 24  12
So A( ABCD ) = h=  8 =18  8 =144 cm 2.
2 2
231 Geometry 8
EXAMPLE 84 In the figure, ABCD is a right trapezoid. D 4 C
Given that AB = 8 cm, BE = 3 cm,
5
EC = 5 cm and DC = 4 cm, find the area of
AED. E
3

Solution A( 
AED ) A( ABCD )  A( ABE )  A( DCE ) A 8 B

48 8  3 4 5
  (5  3)  
2 2 2
 (6  8)  12  10
 26 cm 2 .

EXAMPLE 85 In the figure, ABCD is a right trapezoid and D C


E is the midpoint of AD. Given that BC = 6 cm
and BE = 5 cm, find the area of the
E
trapezoid. 6

5
Solution Let us draw the median EF.
Then EF AB DC, EF  BC and F is the A B
midpoint of BC. So BF = 3 cm.
EFB is a right triangle so EB2 = EF2 + FB2, i.e. 52 = EF2 + 32, EF = 4 cm.
Finally, A(ABCD) = median  height = EF  BC = 4  6 = 24 cm2.

EXAMPLE 86 An isosceles trapezoid has a diagonal of 10 cm and height 6 cm. Find the area of this
trapezoid.

Solution Let us draw the altitudes DK and CH. D 8–x C


ABCD is an isosceles trapezoid, so let us
10
write AK = HB = x.
6
By the Pythagorean Theorem in AHC we
A x K 8–x H x B
have AC2 = AH2 + CH2, i.e. AH = 8 cm.
If AH = 8 and AK = x then KH = DC = 8 – x, AB = 8 + x.
AB  DC 8  x 8 x
So A( ABCD )   CH   6  8  6  48 cm 2 .
2 2
Areas of Quadrilaterals 232
Theorem
In the figure, ABCD is a trapezoid and O is
D C
the intersection point of its diagonals. If n
A(AOD) = x, A(AOB) = m, A(BOC) = y x O y
and A(COD) = n, then
m

1. x y m  n.
A B
2

2. A( ABCD ) ( m  n) .

Proof 1. ADC and BDC have a common base DC


and two common altitudes, so
A(ADC) = A(BDC). So x + n = y + n and
therefore x = y.
We know that in any quadrilateral,
S1  S3 = S2  S4.
This gives us x  y = m  n. So x  y = x2 = y2 = m  n, i.e. x y m  n.
2. A(ABCD) = x + y + m + n
= mn + mn + m + n (x = y = m  n)
=m+2 mn + n
= ( m  n )2 .

EXAMPLE 87 In the figure, ABCD is a trapezoid and O is the D C


intersection point of its diagonals. Given that
A(DOC) = 9 cm2 and A(AOB) = 25 cm2, O
find the area of ABCD.

A B
Solution A( AOD ) 
A( BOC )
 A( DOC )  A( AOB)
 9  25
 15 cm 2
So A(ABCD) = A(AOD) + A(BOC) + A(DOC) + A(AOB)
= 15 + 15 + 9 + 25
= 64 cm2.

233 Geometry 8
EXAMPLE 88 In the figure, ABCD is a trapezoid and P is D C
the intersection point of its diagonals. Given P
that A(ABD) = 12 cm2 and A(BCD) = 8 cm2,
find the areas of DPC and PCB.

Solution Let A(ADP) = A(PCB) = x, then A B


A(DPC) = 8 – x and A(ABP) = 12 – x.
By the previous theorem, we have x m  n (8  x)  (12  x).
24
Rearranging this equation given us x2 = 96 – 20x + x2, i.e. x A( PCB)  cm 2 .
5
24 16 2
So A( DPC )  8  x  8   cm .
5 5

Theorem
Let ABCD be a trapezoid and let E be the D C
midpoint of leg AD.
E
A( ABCD ) .
Then A( BEC ) 
2
A B

Proof Let us draw the median EF and the altitudes CH and KB of the trapezoids DEFC and EABF
respectively.
h ac
Since EF is the median, CH
 BK
 and EF  .
2 2
ac h D c C

EF  CH 2 2
Now A( ECF
) 
2 2 F K
E
( a  c)  h H
 , and
8
ac h a
 A B
EF  BK 2 2 (a  c )  h .
A( EBF
) 
2 2 8

( a  c )  h ( a  c)  h
So A( EBC )  A( ECF )  A( EBF )  
8 8
ac
h
( a  c)  h 2
 
4 2
A( ABCD )
 , as required.
2

Areas of Quadrilaterals 234


EXAMPLE 89 In the figure, ABCD is a trapezoid and E is D 5 C
the midpoint of leg AD.
Given that AB = 8 cm, CD = 5 cm and E
A(BEC) = 26 cm2, find the height of the
trapezoid.
A 8 B

Solution By the previous theorem we can write A(ABCD) = 2  A(BEC) = 2  26 = 52 cm2.


AB  CD 8 5
But since A( ABCD )   h, we have 52   h, i.e. h 
8 cm.
2 2

EXAMPLE 90 In the figure, ABCD is a trapezoid and F is D C


the midpoint of BC. E
Given that AD = 8 cm,
F
EF = 7 cm and AD  EF, find the area of
the trapezoid.
A B

Solution Let us draw DF and AF. By the previous theorem we can write
AD  EF
A( ABCD ) 2  A( ADF ) 2 
2
 8  7  56 cm 2 .

EXAMPLE 91 ABCD is a trapezoid with bases AB and DC and median EF such that E is on AD and F is on
BC. Given that P is the midpoint of EF and A(APE) = 4 cm2, find A(ABCD).

Solution Let the lengths of the bases be a and c and D c C


let the height of the trapezoid be h.
a+c
ac 4 P
E F
Then EF  ,
2 h
2
ac h
EP  and PH . A H B
4 2
ac h ac a
 h
EP  PH 4 2 2
So A 
APE)  
2 2 8
A( ABCD ) 2
  4 cm .
8
This gives us A(ABCD) = 8  4 = 32 cm2.
235 Geometry 8
EXAMPLE 92 In the figure, ABCD is a trapezoid and
D 6 C
KL AB DC.
Given that A(ABLK) = A(KLCD), K
x
L
AB = 8 cm and DC = 6 cm, find the length
KL = x.
A 8 B

Solution Let KL = x and let us draw line segment MN D 6 C x–6 M


parallel to DA, as show in the figure. h1

So KL = AN = DM = x and NB = 8 – x, K
x
L
CM = x – 6.
h2
Now let us draw the heights h1 and h2 from
A x N 8–x B
point L to the bases DM and AB, respectively.
It is given that A(ABLK) = A(KLCD).
8x 6x h1 8  x
So  h  h1 , i.e.  . (1)
2
2
2 h2 6  x
In CLM and BNL,

m(CLM) = m(BLN), m(LCM) = m(LBN), and

m(LMC) = m(LNB).
h1 CM x  6
This means CLM  BLN, i.e.   . (2)
h2 NB 8  x
h1 8  x x  6
Combining (1) and (2) gives us  , i.e. 64 – x2 = x2 – 36.
h2 6  x 8  x
Rearranging this expression gives x2 = 50, x = 5ñ2 cm.

EXAMPLE 93 In the figure, ABCD is a trapezoid. Base AB D H I C


is divided into four equal parts and base DC
is divided into three equal parts.
Given that AB = 2  DC, find
A( EGIH )
. A E F G B
A( ABCD )
Areas of Quadrilaterals 236
Solution Let DC = x so AB = 2x, and let h be the height of the trapezoid.
Let us draw the line segments EI, ED and EC.
xh
We can say A( DEC )  .
2
A( DEC ) xh
DH = HI = IC is given, so A( HEI
 )  .
3 6
Now let us draw the line segments IA and IB.
2 xh
We can write A( AIB)   xh.
2
A( AIB) xh
AE = EF = FG = GB is given, so A( EIG
 )  .
2 2
xh xh 2xh
So A(EGIH) = A(HEI) + A(EIG) =  .
6 2 3
2 xh
A( EGIH ) 3 4
Finally,   .
A( ABCD ) 3xh 9
2

EXAMPLE 94 In the figure, ABCD is a trapezoid.


D C
Given that AD EC and EB = 2  AE, find
A( AEFD ) F
.
A( ABCD )

A E B

Solution Let AE = x, then EB = 2  AE = 2x, and DC = x because AECD is parallelogram.


DC x 1
Since DC EB, DFC  BFE and
k   . So EF = 2  CF and BF = 2  DF.
EB 2x 2
A( DFC ) 1 2 1
By Property 9,  ( ) , so if A(DFC) = S then A(BFE) = 4S.
A( BFE) 2 4
Property 9: If two triangles
are similar then the ratio If A(BFE) = 4S then A(BFC) = 2S because EF = 2  CF.
of their areas is equal to
the square of the ratio of Now let us draw DE. In DEB, BF = 2  DF and since A(BFE) = 4S, A(DEF) = 2S.
similarity. A( DEB)
In ABD, EB = 2  AE and A(DEB) = 4S + 2S = 6S, so A(ADE) = = 3S.
2
A( AEFD ) A( ADE)  A( DEF ) 3S  2S 5
So    .
A( ABCD ) A( ABCD ) 12S 12

237 Geometry 8
Check Yourself 13
1. The bases of a trapezoid measure 5 cm and 9 cm. Given that the height of this
trapezoid is 6 cm, find its area.
D 8 C
2. In the figure, ABCD is a trapezoid.
120°
Given that AD = 4ñ3 cm, DC = 8 cm, 4ñ3
m(ADC) = 120° and m(ABC) = 45°,
45°
find the area of this trapezoid.
A B

3. The length of the shorter base of a right trapezoid measures 6 cm. The length of the
longer base is 12 cm and one of the base angles measures 60°. Find the area of this
trapezoid.

4. In the figure, ABCD is an isosceles trapezoid. D C


120°
Given that AB = 10 cm, BC = 4 cm and 4

m(BCD) = 120°, find the area of the trapezoid.


A 10 B

5. In the figure, ABCD is a trapezoid and E is a D 5 C

point on AB. Given that AE = 4 cm,


EB = 6 cm and
A( DEBC )
DC = 5 cm, find .
A( ABCD )
A 4 E 6 B

6. The ratio of the lengths of the bases of a trapezoid is 6 : 13. The height of the
trapezoid is 20 cm and its area is 380 cm2. Find the length of the longer base.

15
7. In the figure, ABCD is a trapezoid and EF is its D C
median. Given that AB = 3 cm, DC = 15 cm and
A(ABCD) = 90 cm2, find A(ABFE). E F

A 3 B

8. ABCD is a trapezoid and O is the intersection point of its diagonals.


Given that A(ABC) = 18 cm2 and A(ACD) = 12 cm2, find A(AOD).

Areas of Quadrilaterals 238


9. In the figure, ABCD is a trapezoid and its diagonal BD is D 8 C
divided into three equal parts.
E
Given that AB = 24 cm, DC = 8 cm and A(AEF) = 12 cm2,
find the area of the trapezoid. F

Answers A 24 B

11
1. 42 cm2 2. (66 + 6ñ3) cm2 3. 54ñ3 cm2 4. 16ñ3 cm2 5. 6. 26 cm 7. 30 cm2
15
36
8. cm 2 9. 48 cm2
5

I. AREA OF A KITE D
Recall that a kite is a quadrilateral that has
two pairs of adjacent congruent sides. The
If the diagonals of a diagonals of a kite are perpendicular to each
quadrilateral are
perpendicular to each
other, so the area of a kite is half the product
other then the area of of its diagonals: in the figure,
the quadrilateral is A C
AC  BD AC  BD
A( ABCD )  . A( ABCD )  .
2 2
B

EXAMPLE 95 The diagonals of a kite measure 12 cm and 8 cm. Find the area of this kite.

AC  BD 12  8
SolutionA   48 cm 2 .
2 2

EXAMPLE 96 In the figure, ABCD is a kite. D


Given that BC = 5 cm, DC = 12 cm and
m(BCD) = 90°, find the area of the kite.
12

Solution We draw the diagonal BD to get two A C


triangles ABD and BCD. Since a kite is 5
symmetric about its main diagonal, B

A(ABD) = A(BCD) and m(BAD) = m(BCD) = 90°.


5  12
So A( ABCD)  2  ( BCD)  2   60 cm 2.
2

239 Geometry 8
EXAMPLE 97 In the figure, ABCD is a kite. D
Given that AB = BC = 17 cm,
AD = DC = 25 cm and BD = 28 cm, find the 25
area of the kite.
28

A C
Solution Let us draw the diagonal AC and let the
intersection point of the diagonals be O. 17
Let AO = y and BO = x, then
B
OD = 28 – x.
By the Pythagorean Theorem in ADO we have
y2 = 252 – (28 – x)2 = 172 – x2, i.e. 625 – 784 + 56x – x2 = 289 – x2.
Rearranging this expression gives us x = 8 cm.
Similarly, in ABO we have y2 = 172 – x2 = 172 – 82 = 289 – 64 = 225,
i.e. y = AO = 15 cm and AC = 2  15 = 30 cm.
AC  BD 30  28

So A( ABCD )   210 cm 2.
2 2

Check Yourself
1. A kite has an area of 120 cm2. Given that one of its diagonals measures 24 cm, find the
length of the other diagonal.
D
2. In the figure, ABCD is a kite and O is the intersection
point of its diagonals.
Given that AB = AD, BC = DC, AO = 9 cm and
9 16
A C
OC = 16 cm, find the area of the kite. O

D
3. In the figure, ABCD is a kite.
60° 8
Given that AB = AD = 4 cm, 4

BC = DC = 8 cm and m(ADC) = 60°, find A C


the area of the kite.
Answers
B
1. 10 cm 2. 300 cm2 3. 16ñ3 cm2
Areas of Quadrilaterals 240
The Pythagorean Theorem is one of the oldest and most famous theorems in the history of geometry. Although the
theorem was known by the Babylonians and the Egyptians about 1000 years before the time of Pythagoras (who was
born in around 575 BC), Pythagoras was the first person to publish a deductive proof, which is why the theorem
was given his name.
The Pythagorean Theorem states that the sum of the A

squares of the legs of a right triangle is equal to the length


of its hypotenuse, i.e.
b
2 2 2
a +b =c
C a B
where c is the hypotenuse and a and b are the legs of the
right triangle.
Mathematicians since the time of Pythagoras have studied this theorem, and it has been proved in different ways
in different branches of mathematics. A writer called Elisha Scott Loomis once published a book with over 360
different proofs of the theorem. Here are four popular proofs.

Proof 1: Let us draw a square ABCD with side


D C
length c.
b
In the figure, ABE, BCF, CDG and H b
DAH are congruent right triangles inside
ABCD with sides a, b and c. a– b
a– b G
c
We can see that the triangles create a E a– b
a– b
smaller square EFGH with side length a – b.
b
Now we can write the area of ABCD in two F
b
ways: A(ABCD) = c2 and
A c B
A(ABCD) = 4  A(ABE) + A(EFGH).
AH = DG = CF = BE = a
ab
So c2 4  ( )  ( a  b)2
2
 2ab  a2  2ab  b 2 . b a
D H C
Simplifying this gives c2 = a2 + b2.
c b
Proof 2: Let us draw a square ABCD with side a c
G
length a + b. Then we choose the points E,
F, G and H such that EAF, FBG, GCH
a
and HDE are right triangles with sides a, E c
b and c. b c
We can say that each of these four triangles
A a F b B
has hypotenuse c.
Now we can write A(ABCD) in two different ways:
A(ABCD) = (a + b)2 and A(ABCD) = 4  A(EAF) + A(EFGH).
ab
So A( ABCD ) (a  b )2 
4 ( )  c 2 , i.e. a2 + 2ab + b2 = 2ab + c2.
2
Canceling 2ab from each side gives us a2 + b2 = c2.

Proof 3: Let us draw the right trapezoid ABCD with


bases a and b and height a + b. Then we D b C
connect B and C with point E so that CDE
and EAB are congruent right triangles
with sides a, b and c. a c

As we can see in the figure, BEC is an


isosceles triangle with leg c.
E
We can write the area of the right trapezoid c
in two ways: b

sum of the bases  height A a B


A( ABCD )  ,
2
and A(ABCD) = 2  A(EAB) + A(BEC).
ab a b c c a 2  2ab  b 2 2ab  c 2
So A( ABCD ) ( )  ( a  b ) 2 ( ) , which gives us  .
2 2 2 2 2
Canceling the denominators and 2ab gives us a2 + b2 = c2.

Proof 4: Let us draw the right trapezoid ABCD with


bases a and b and height a. D a C

We can say that


EC = DB = c, EDC  BAD by the SSS b c
Congruence Theorem.
E
From the angles we can get DB  EC. c
a– b
Now we can write the area of the right
trapezoid in two ways:
A b B
sum of the bases height
A( ABCD ) 
2
and A(ABCD) = A(BCDE) + A(ABE).
ab c  c (a  b )  b
So A( ABCD )  a  .
2 2 2
Canceling 2 from both sides and taking ab – b2 to the left-hand side gives us a2 + b2 = c2.
EXERCISES 3 .2
A. Area of a Quadrilateral 6. In the figure, ABCD is a D
convex quadrilateral and
1. In the figure,
E is the intersection point
DC = 6, A
of its diagonals. E
AD = 8,
Given that DC = BC, 20°
AB = 10,
m(DCA) = 20°, C
BC = 12 and
m(DBC) = 25°,
m(ADC) = 90°.
AC = 6 and
Find the area of the quadrilateral ABCD. 25° 10
BD = 10,
find the area of ABCD. B

2. ABCD is a convex quadrilateral and E is the


intersection point of its diagonals. Given that
AE = 2, BE = 5, CE = 6, DE = 10 and BC = 5, 7. ABCD is a convex quadrilateral and E is the
find the area of ABCD. intersection point of its diagonals. Given that
A(ABE) = A(CDE) = 18 and
A(BCE) = 4  A(ADE), find the area of ABCD.
3. ABCD is a convex quadrilateral and E is the
intersection point of its diagonals. DE = 3 cm and
A( ADC )
BE = 12 cm are given. Find .
A( ABCD ) 8. In the figure, ABCD is a C
 E
convex quadrilateral, E is D

the midpoint of DC and F


F
4. In the figure, E is the intersection D is the intersection point
point of the diagonals of ABCD of the diagonals. Given
that AG = 5 cm and A 5 G 7 B
and AC is perpendicular to BD. 8
Given that BC = 4, A( BCF )
GB = 7 cm, find .
DC = 8, AE = EC and A A( DAF )
E C
30°
m(ACB) = 30°, find the
4
area of ABCD.
B
B. Area of a Parallelogram
9. The sides of a parallelogram measure 10 cm and
5. ABCD is a convex quadrilateral with AD = DC = 8, 24 cm. Given that the length of the altitude to the
BD = 14 and m(ADC) = 60°. The angle between longer side is 5 cm, find the length of the altitude
its diagonals is 90°. What is A(ABCD)? to the shorter side.

243 Geometry 8
10. In the figure, ABCD is a parallelogram, 16. One of the diagonals of a parallelogram is 8 units
DE is perpendicular to AB and AF is long and one of its sides is 10 units long. The
perpendicular to BC. Given that AB = 12, AD = angle between this side and this diagonal is 45°.
6 and Find the area of the parallelogram.
AF = 8, find the length DE = x.
17. In the figure, ABCD is D E F G C
a parallelogram. Side
H
DC is divided into
I
11. One of the diagonals of a parallelogram has the four equal parts and
same length as one of its sides. Given that the side BC is divided into A B

longer side of the parallelogram is 6 units long three equal parts. Given that A(ABCD) = 240, find the
and its interior acute angle measures 30°, find the area of the quadrilateral AICF.
area of the parallelogram.
18. In the figure, ABCD D C
12. In the figure, ABCD is D E is a parallelogram
C
a parallelogram and F and E is the mid- 6

BE is perpendicular G 4 point of side AB. 150°


H
to DC. Given that Given that A E 4 B

AH = HG = GF = FD, A ? B EB = 4, BC = 6 and
A(GBH) = 6 and EB = 4, find the length of AB. m(DAB) = 150°, find A(AECD).

19. In the figure, ABCD is D I J K C


a parallelogram. Side
13. ABCD is a parallelogram and H is a point on DC
AB is divided into five
such that BH  DC. Given that BH = 6, BC = 10
equal parts and side
and DH = 8, find the area of ABCD.
CD is divided into A E F G H B
four equal parts.
14. ABCD is a parallelogram and DB is its diagonal. Given that A(ABCD) = 180, find A(FHKJ).
Given that m(DAB) = 30°, AD  BD and BC = 8,
find A(ABCD).
20. ABCD is a parallelogram. E and F are two points
on base AB, and G is a point on CD. Given that
15. In the figure, ABCD is D C 3
EF  AB and A(EFG) = 12, find A(ABCD).
a parallelogram and P P
5
is a point inside the
parallelogram. 21. ABCD is a parallelogram and E and F are the

Given that A B midpoints of sides AB and BC respectively. If G is
A(PBC) = 18, A(PAD) = 12 and the intersection point of lines AF and CE and
2  A(PAB) = 3  A(PCD), find the area of PAB. A(AEG) = 12, find A(ABCD).

Areas of Quadrilaterals 244


22. In the figure, ABCD is D
G H C 30. In the figure, ABCD is a D C
 a parallelogram and rhombus and the diagonal E
6
E F
sides AD and CD are 10 BD is divided into six
each divided into F
equal lengths. G
three equal parts. A B
Given that I
Give that EG  GB, EG = 6 and GB = 10, find the
A(ABCD) = 72, A B
sum of the areas of the shaded regions.
find the sum of the areas of the shaded regions.

C. Area of a Rhombus
31. In the figure, ABCD is D
23. A rhombus has area 80 and a diagonal which is 20 a rhombus. Point E is
units long. Find the length of the other diagonal. on the diagonal BD
such that DE = 41, 41
24. A rhombus has diagonals which are 10 cm and A C
EB = 9 and
22 cm long. Find its area.
m(DCE) = 90°.
E
25. In the figure, ABCD is D C Find the area of this 9

a rhombus. rhombus. B

Given that 8
32. In the figure, ABCD is D 4 E 9 C
AB = BD = 8, find
a rhombus, O is the
A(ABCD). A B
8 intersection point of
O
its diagonals, E is a
26. In the figure, ABCD D C
point on DC, and A B
is a rhombus and BH 120°
OE  DC. Given that DE = 4, EC = 9 and
is an altitude. Given H
8 m(OEC) = 90°, find the area of the rhombus.
that BH = 8 and
m(ADC) = 120°, 33. In the figure, ABCD is D C
A B
find the area of this a rhombus and the
E
rhombus. points A, B and F are 10
collinear. Given that
27. A rhombus has area 200 cm2. Given that the A B 10 F
DF  BC and
length of the altitude to a side is 10 cm, find the
AD = BF = 10, find the area of rhombus ABCD.
measure of the acute angle of this rhombus.

28. In a rhombus ABCD, the lengths of the diagonals 34. In the figure, ABCD is D E C

have the ratio 5ñ2. Given that A(ABCD) = 120ñ2, a rhombus and the 10

find the lengths of the diagonals. points E and F are the F


midpoints of sides DC 13
29. The sum of the lengths of the diagonals of a and BC, respectively.
A B
rhombus is 34 and one side of the rhombus Given that EF = 10
measures 13 units. Find the area of this rhombus. and AF = 13, find A(ABCD).

245 Geometry 8
D. Area of a Square 43. ABCD and BCEF are two squares, and P and Q are
the respective intersection points of their
35. Find the area of the square whose perimeter is
diagonals. Given that AB = 8, find the area of
12ñ5 units.
PBQC.
36. The diagonal of a square is 6 units long. Find the
area of this square. 44. In the figure, ABCD is D F C
a square and E and F S1
37. In the figure, ABCD is a D C are the midpoints of
G
square and A, B and E sides AD and DC E

are collinear. Given that S2


respectively.
A(EDC) = 20, find the Given that
A B
area of the square. A(DEGF) = S1 and
A B E S
A(ABCG) = S2, find 1 .
S2
38. A rectangular floor has side lengths 12 m and 15 m.
We want to cover it with square tiles with side 45. In the figure, ABCD is D C
length 40 cm. How many tiles do we need? a square. Side BC is
E
divided into four G
39. In the figure, ABCD is D C equal parts and AE is
H
F
a square and E is the divided into three I

midpoint of BC. Given equal parts. Find the


E A B
that AE = 5ñ5, find 5ñ5 area of the shaded region if A(ABCD) = 120.
the area of this square.
A B 46. In the figure, ABCD is a D C
square and BEC is an
40. If we lengthen the sides of a square by 40%, by
equilateral triangle. E
what percentage will its area increase?
Given that
41. In the figure, ABC is C A(ABE) = 16,
A B
an equilateral triangle find the area of the
and DEFG is a square. square.
G F
Given that
A(ABC) = 4ñ3, 47. In the figure, ABCD is a D C
find A(DEFG). square with side length
G F
A D E B
12 cm and AEFG is a 12
rectangle. Given that
42. Two opposite sides of a square are shortened by
BF = 3  DF, find
2 units. The area of the rectangle obtained is
A( AEFG ) A E B
35 square units. What was the length of one side .
A( ABCD )
of the original square?

Areas of Quadrilaterals 246


48. In the figure, ABCD is a square D C 53. S 6 R
with side length 10 units. Given 135° 150°
x y
that DE + EC = 12 and 10
E 10
m(DEC) = 90°, find the area
of the shaded region ABCED. P Q
A 10 B
In the figure, PQRS is a trapezoid. Given that
QR = 10, SR = 6, m(PSR) = 135° and
E. Area of a Trapezoid m(SRQ) = 150°, find the area of the trapezoid
PQRS.
49. The two bases of a trapezoid are 12 and 8 units
long. Given that the area of the trapezoid is
100 square units, find the length of its altitude.
54. MN and PQ are the bases of a trapezoid and O is
the intersection point of its diagonals. Given that
50. In the figure, KLMN is N M A(MON) = 90, MN = 12 and PQ = 8, find the
30°
a right trapezoid. area of trapezoid MNPQ.
6
Given that
KL = 4, NK = 6 and
m(LMN) = 30°, find K 4 L 55. In the figure, ABCD is D 25 C

the area of the trapezoid. an isosceles trapezoid.


Given that AD  AC,
AD = AB = BC, A B
7
51. In the figure, ABCD is D 6 C
AB = 7 and DC = 25,
a right trapezoid and E 2
find the area of this trapezoid.
is a point on AD. E
Given that AB = 10, 4
DC = 6, AE = 4 and 56. The diagonals of an isosceles trapezoid are
A 10 B
ED = 2, find the area perpendicular to each other. Given that one
of EBC. diagonal is 20 units long, find the area of this
trapezoid.

52. D 8 C

150° 57. In the figure, ABCD is D C


6 6
a trapezoid and E is
the midpoint of AD. H 20
A B
Given that EH  BC, E 12

In the figure, ABCD is an isosceles trapezoid. EH = 12 and


Given that AD = BC = 6, DC = 8 and BC = 20, find the
m(ADC) = 150°, find the area of the trapezoid. area of ABCD. A B

247 Geometry 8
58. AB and DC are the bases of a trapezoid and E is a 63. ABCD is a trapezoid with bases AB and DC.
point on base AB. Given that AB = 3  DC and 
AB = 26, BC = 16, CD = 6 and AD = 12 are given.
A(DEC) = 12, find the area of the trapezoid. Find the area of the trapezoid.

59. In the figure, ABCD D C 64. ABCD is an isosceles trapezoid with diagonals
is a trapezoid and E, 
12 units long. Given that the angle between a
F and K are the E G H F diagonal and a base is 30°, find the area of the
midpoints of sides trapezoid.
AD, BC and AB, A K B
respectively. Given
that EDG, HFC and GHK are equilateral
triangles and A(GHK) = 4, find the total area of F. Area of a Kite
the trapezoid. 65. The diagonals of a kite are 6 and 8 units long.
Find the area of this kite.

60. In the figure, ABCD is D C


a trapezoid and O is 66. In the figure, ABCD is a D

the intersection point O kite, AD = AB = 10, 10 6


of its diagonals. Given DC = BC = 6 and
A C
that A(AOD) = 12 and m(ABC) = 150°. Find
A B the area of the kite. 10 150° 6
A(DOC) = 4,
find A(ABCD). B

61. In the figure, ABCD is D 7 C 67. In the figure, ABCD is D


a trapezoid and K is a x a kite. Given that 105°
6
point on AD. K
AB = AD = 6,
A C
Given that AB = 13, 10 BC = DC,
DC = 7, AK = 10 and m(BAD) = 90° and 6
A 13 B
A(ABK) = A(KBCD), m(ADC) = 105°, B
find the length KD = x. find the area of the kite.

62. The two bases of a trapezoid measure 5 cm and 68. ABCD is a kite. Given that AB = BC,
15 cm. Given that the diagonals measure 12 cm AD = DC = 12, m(ADC) = 60° and
and 16 cm, find the area of this trapezoid. m(ABC) = 120°, find the area of the kite.

Areas of Quadrilaterals 248

You might also like